You are on page 1of 160

Nguyen Van M~u ici» bien)

ToAN ROI RAe





'" '" ....

vA MOT so VAN DE LIEN QUAN

(Tai lieu b6i dirdng he 2007)

HA. NQI, 05-14 THANG 8 NA.M 2007

S:':., . .:. ~,' - ',. '_' • " ~' .. ~ \ ,

· ~. d "-

LOI nOI au

Tren b6n muoi nam thuc hien "Chuong trmh dao tao va b6i hoc sinh nang khisu toan bac ph6 thong" la mot chang duong cua mot chu trlnh d~c biet g~n vdi su khdi d§.u, trudng thanh va ngay cang hoan thien xu§,t phat til mot rno hinh dao tao nang khieu Toan hoc d~c biet tai D~ hoc T6ng hQP Ha NQi. Huang dao tao miii nhon nay mang tfnh dQt pha cao, da dao tao ra cac th~ he hoc sinh co nang khieu trong linh vue toan hoc, tin hoc va khoa hoc tu nhien: V~t 1y, Hoa hoc, Sinh hoc va khoa hoc su s6ng. Trong di@u kien thieu th6n v@ v~t ch§,t keo dai qua nhieu thap ky va tnii qua nhieu thach thirc, chung ta da tim ra huang di phu hQP, da di len vimg chac va 6n dinh, da tim toi, tich luy kinh nghiem va co nhieu sang tao dang ghi nhan. Cac th~ h~ Th§.y va Tro da dinh hinh va ti~p c~n voi th~ gioi van minh tien ti~n va khoa hoc hien dai, cap nhat thong tin, sang tao phuong phap va t~p duot nghien ciru. G~n vdi viec tich cue d6i mdi phucng phap day va hoc, chucng trlnh dao tao chuyen Toan dang huang tai xay dung h~ thong chuyen d@, dang no hrc va da t6 chirc t.hanh cong Ky thi Olympic Toan quoc t~ l§.n· thir 48, nam 2007 tai Viet Nam, dircc ban be qu6c t~ ca ngoi,

Sau g§.n mra th~ ky hinh thanh va phat tri~n, co th~ noi, giao due miii nhon phB thong (giao due nang' khieu) da thu duoc nhtrng thanh tuu rue rd, duoc Nha nude d§.u tu c6 hieu qua, dUQC xa hoi thira nhan va ban be quoc t~ kham phuc. Cac dQi tuyen quoc gia tham du cac ky thi Olympic quoc t~ c6 b@ day thanh tich mang tfnh Bn dinh va c6 tinh k~ thira. Dac biet, nam nay, cac DQi tuyen Toan va Tin quoc gia tham du thi Olympic quoc t~ da dat diroc thanh tfch nBi bat. DQi tuy~n Toan Viet Nam da vtron len dung thir ba (theo sir s~p xep khong chinh thirc) trong s6 95 dQi tuy~n cac nude tham dir IM048.

Tir nhieu nam nay, cac h~ nang khieu Toan hoc va cac Truong THPT Chuyen thirong S11 dung song song cac sach giao khoa dai tra k~t hop vdi sach giao khoa chuyen biet va sach chuyen d@ eho cac H~ THPT Chuyen, HQc sinh cac ldp nang khieu da ti~p thu t6t cac kien thirc co ban theo thai hrong hien hanh do Be;> GD va DT ban hanh.

Hien nay, chirong trlnh cai each giao due dang biroc VaG giai doan hoan chlnh bQ SGK moi. Thai hrong kien thirc cling nhu trat tl,t ki~n thirc co ban co nhtrng thay d6i dang k~. Cac kien thirc nay dang duoc can nhac d~ n6 van narn trong khuon kh6 hien hanh cua cac kien thirc nang cao d6i vdi cae lop chuyen toano VI le d6, viec ti~n hanhviet h~ thong cac sach chuyen d@ cho cac lop nang khieu c§.n diroc ti~n hanh kh§.n trucng va diroc xem xet toan dien tU phia cac chuyen gia giao due va cac co giao, th§.y giao dang true ti~p giang day cac lop chuyen,

DUQc su cho phep cua BQ GD va DT, Truong D~i HQc Khoa HQc Tl,t Nhien, DHQGHN phoi hop cling voi cac chuyen gia, cac nha khoa hoc, cac co giao, th§.y giao thuoc DHSPHN, DHQG TpHCM, Vien Toan HQc, HQi Toan HQc Ha NQi, Tl?>p Chf Toan HQc va Tu6i Tre, cac Truong THPT Chuyen, Cac SCI GD va DT, ... t6 chirc b6i dircng cac chuyen d@ nghiep vu sau dai hoc nh~m b6i duong hoc sinh gioi cac mon Toan hoc va khoi ki~n thirc khoa hoc tu nhien nhu la mQt tu sach d~c bi~t ph1,lc V1,l b6i duang hQc sinh gioi.

2

Chung toi xin gioi thieu cuon sach cua nh6m cac chuyen gia, cac th§.y giao vdi su tham gia dong dao cua cac d6ng nghiep tham du Truong he 2007 v~ chuyen d~ "T6an fbi rae va mot s6 v§.n d~ lien quan ''.

Cuon sach nay nham cung c§.p mot s6 kien thirc chuyen d~ b§.t dl1ng thirc d mire d9 kh6 v~ t6an fbi f9-C, dai s6, s6 hQC, va giai tich. Day ciing la chuyen d~ va bai giang rna cac tac gia da giang day cho hoc sinh cac d9i tuyen thi Olympic Toan hoc quoc gia va qu6c t~.

Chung Wi cung xin chan thanh cam on cac ban doc cho nhtrng y kien d6ng g6p d@ cuen sach ngay cang hoan chinh.

Thay m~t Ban T6 Chirc

GS TSKH Nguyen Van Mau

3

Mue lue

• •

Loi noi dliu 2

D6 t hi to mau va mot s6 bai toan khong m§.u mire

Dij,ng Huy RuiJ,n . .- 5

Logie hlnh thirc va ap dung

Nguyen Van MiJ,u 18

Cong t hirc tfnh s6 phan tli' cua mot hop cac t~p hop

V'll Dinh. Hoa ...............................................•............. 24

Mang .htoi 0 vuong t ren mat phang

V'll Dl,nh Hoc 30

Nguyen If Dirichlet va mot s6 bai toan ap dung

Nguyen Duy Thai Bon 37

M9t s6 phirong phap giai cac bai toan t6 hop nang cao

Dij,ng Hung Thiing 43

Xay dirng song anh giai mot s6 bai toan t6 hop

Huynh Tan Chau 57

Phuong phap thi~t lap h~ thirc truy h6i trong t6 hop

Huynh Tan Clun: 63

Y tu'dng giai va sv tu'ong minh Wi giai qua mot s6 bai toan t6 hop

L€ Van Quang 69

Gidi t hieu mot s6 bai toan dai s6 co xuat xu til hmh h9C

Nguyen Dang Phat 76

4

B~t bien, don bi~n va tmg dung

'Jlriln Nam Dung ...............................•..•...•..•.......... : .... 117

Mt)t s6 v~n d~ cua Toan rai rae

Nguyfu Van Ti€n 126

, ,

Do thi to mau va mot so bai toan

• •

khong mau rmrc

Dang Huy Ruan

Ly thuyet dB thi n6i chung, d~c biet dB thi to mau dU<;1C van dung d~ giai cac bai to an khong mau muc rfi,t hieu qua.

D~ giai toan thong qua ·dB thi c§.n thuc hien theohai buoc:

D§.u tien xay dung dB thi d~ mo ta cac quan he, di~u kien dU<;1C phat bi~u trong bai toano Sau d6 can cu vao cac khang dinh cua 1y thuyet dB thi d~ suy ra dap an.

Trong phan nay chi d~ cap d~n viec thong qua dB thi to mau d~ giai mot s6 dang iotu: khong mau muc.

D6i vdi moi dang totui khong mau muc d~u dua ra khling dinh tuong irng v~ dB thi to mau d~ c6 th~ van dung giai quyet hang loat bai toan thuoc dang diroc xet.

DB thi duoc goi la d§.y du, n~u moi cap dinh cua n6 d~u diroc n6i bang mot canh.

DB thi d§.y du gBm n dinh,

DB thi d§.y du gorn 3 dinh (4 dinh) vdi cac canh diroc to mau bang cling mot mau dircc goi 1a tam giac (til giac) cling mau,

1 Dang 1

1.1 B~ti toan

Bai toan 1 (D€ thi Olympic Totui QU8c te) MUdi My nhi: btu: h9C viet ihn: cho nhau. M8i ngudi d€u viet this cho tiLt cd ngudi kluic, Ctic tliu chi trao d6i v€ S d€ iiii. Titng cij,p· tihii btu: h9C chi viet thu trao d6i v€ cung mot d€ iai. Chung minh ding chi co it nMtS nhi: btic h9C viet tliu cho nhau trao d6i v€ ciuu; mot van d€.

Bai toan 2 Trong mot cu¢c giJ,p go qu8c te co 17 nha nqoai qiao tham gia. M8i cij,p tihi: ngo(Li giao chi trao d6i true tiep vai nhau Mng mot trong ba ngon ngiJ: Anh, Pluip, Diic.

Chung minh rang luon lutni tun duac ba nha ngo(Li giao, ma ho co the trao d6i true ti€p dU\fc Mng mot. trong ba ngon ngiJ ke tren.

Bai toan 3 M8i ciip d8i iuatiq cho truce chi co m¢t trong ba quem h~: tl, t2, t3. Chung minh rang luot: luan tim duac ba d8i tuanq, ma m6i cij,p trong b¢ ba nay cung co quan h~ ti (1 ~ i ~ 3) dti cho.

5

1. 2 Kh~ng dinh

Sau day xet lop d6 thi c6 chua tam giac cling mau.

Vdi cac day s6 nguyen duong:

al = 2, a2 = 3, , an+! = (n+ 1).an + 1;

b2 = 3, b3 = 6, , bn+l = (bn - 1).n + 2

(1) (2)

c6 cac khang dinh sau day:

Khfutg dinh 1 a) DB thi day du co an + 1 dinh vai n mau camli luim. luon co dB thi con day du K3 vai canh. cung mini (tam giac cung mini}.

b) DB thi day du co bn+l dinh uai n mauconh. luoi: hum co dB thi con day dil K3 uoi canli cung mau.

Chung minh. Khang dinh a) chirng minh bang quy nap theo chi s6 n.

1) Co sd quy nap: n = 1. D6 thi d§,y du tucng irng g6m al + 1 = 2 + 1 = 3 dinh l~p thanh mot chu trmh tam giac, Cac canh cua d6 thi nay duoc t6 b~ng mot mau, nen chu trlnh tam giac l~p nen GI cling mau,

2) Quy nap: Ghi sa khang dinh da dung voi n = k, nghia la, d6 thi d§,y du b§.t ky Gk gam ak + 1 dinh vdi cac canh diroc t6 bang k mau da c6 chu trinh tam giac cling mau. C§,n chung tokhllng dinh dung voi n = k + 1.

Xet d6 thi d§,y du tiiy y Gk+l voi ak+l + 1 dinh va cac canh duoc t6 bang k + 1 mau. Gia S11 P la mot dinh tuy y cua Ck+l. Khi d6 P duoc n6i voi ak+! = (k + l)ak + 1 dinh boi cac canh duoc t6 bang khong qua k + 1 mau, nen xuat phat tir P phai c6 it nhat ak + 1 canh duoc t6 bang cling mot mau. Gia sa mau nay la rnau do va cac canh PAl, PA2, ••. , PAadl duoc t6 mau do. C6 hai kha nang xay ra:

1 0) N~u mot trong cac canh n6i giira cac dinh Ai, Aj (1 ~ i, j ~ ak + 1) duoc t6 mau do, chiing han canh (AI, A2) mau do. Khi do chu trlnh tam giac AIPA2 mau do, nen d6 thi Ck+l c6 chu trlnh tam giac mau do.

2°) Truong hop nguoc lai, khong c6 canh nao trong cac canh (Ai, Aj) (1 ~ i,j ~ ak+1) duoc t6 mau do. Khi d6 d6 thi con d§,y du Ck vdi t~p dinh {AI, A2, ... , Aak, Aak+d c6 cac canh duoc t6 bang kh6ng qua k mau, nen theo gia thiet quy nap Ck c6 chu trinh tam giac cling mau. Bdi v~y Ck+l c6chu trmh tam giac cling mau.

Khllng dinh diroc clnrng minh. Khllng dinh b) chimg minh tirong tv.

1.3 Gild to.an

1°) Xay dung d6 thi m6 ta quan he

Cac d6 thi tirong irng vdi ba bai toan da cho diroc xay dung nhu sau:

6

a) Dinh: L§.y 17 di~m tren m~t phang hoac trong khonggian tuong img vdi 17 nha bac hoc (17 nha ngoai giao, 17 d5i tuong da cho). Dung ngay ten cac nha bac hoc (nha ngoai giao, d5i tuong da cho) d~ ghi tren cac diem tucng irng.

b) Canh: Dung

- Canh do d~ n5i giira hai dinh tucng irng voi Hai nha bac hoc trao d6i v§.n d~ thir nh§.t (Hai nha ngoai giao trao d6i true ti~p duoc b~ng ti~ng Anh; Hai d5i tuong co quan h~ t1);

- Canh xanh d~ n5i gitra hai dinh tirong irng vdi Hai nha bac hoc trao d6i v§.n d~ thu hai (Hai nha ngoai giao trao d6i true ti~p duoc bang tieng Phap; Hai d5i tuong co quan he t2);

- Canh yang d~ n5i giira hai dinh tucng irng vdi Hai nha bac hoc trao d6i v§.n d~ thu ba (Hai nha ngoai giao trao d6i true ti~p diroc bang tieng Dire; Hai d5i tuong co quan h~ t3).

Db thi G, (1 :::; i :::; 3) mo ta toan bi) quan he dieu kien diroc cho trong bai toan i. 2°) Suy ra dap an.

Theo khKng dinh 1, trong cac db thi G, (1 :::; i :::; 3) d~u co tam giac cung mau, Neu tam giac nay

- Mau do, thl Ba nha bac hoc tucng irng trao d6i v~ v§.n d~ thu nh§.t (Ba nha ngoai giao tuong irng trao d6i true tiep duoc voi nhau bang tieng Anh; Ba d5i tuong tuong irng co quan h~ t1);

- Mau xanh, thi Ba nha bac hoc tuong irng trao d6i v~ v§.n d~ thu hai (Ba nha ngoai giao tuong irng trao d6i true ti~p diroc voi nhau bang ti~ng Phap; Ba d5i tirong tuong irng co quan h~ t2);

- Mall yang, thl Ba nha bac hoc tuong irng trao d6i v~ v§.n d~ thu ba (Ba nha ngoai giao tuong irng trao d6i true tiep diroc vdi nhau bang ti~ng Dire; Ba d5i tuong tuong irng co quan h~ t3).

2 Dang 2

2.1 Bai toan

Bai toan 1 M(Jt nhom g6m 5 thanh vien, trang do moi b(J ba d€u co 2 nguCli quen nhau va 2 nguCli kh6ng quen nhau. Chung minh ding co the xep ca nhom. ng6i xung quanh mot ban trim, de moi nguCli ng6i giila hai nguCli ma thixnh. uien. do quen.

Bai toan 2 Cha 5 86 nguyen du(}ng tuy y, ma cu 3 86 Mt ky d€ co 2 86 co udc chung va 2 86 nguyen t6 cung nhau. Chung minh rang co the ghi 5 86 tren len mot dUClng trim, de moi 86 d€u dung giila 2 86 ma no co uac chung.

Bai toan 3 Cha 5 d6i tU(Jng tuy y, ma cu ba d6i tU(Jng Mt ky, d€u co 2 d6i tU(Jng co quan h~ tl va 2 d6i iuonq co quan h~ t2. Chung minh rang co the xtp tat ca cac d6i tuatu;

7

dung tren mot duong vong, de moi d8i tut;fng d€u dung gifta 2 d8i tUr;Jng ma n6 c6 quan h~ ti (1 ~ i ~ 2).

2.2 Khang dinh

Vdi day (2) c6 khang dinh sau:

Khang dinh 2 D8 thi day d7l c6 bn+1 - 1 dinh (n ~ 2) vai n mini canh. {cac canh. duac to Mng n mau), sao cho khong tam giac cung mau nao, hum. luon c6 hhmh. 5 camh. uai cdc canh. cung miiu va cac duang cheo duoc to bang cac miiu kluu:

Chung minh. Bang quy nap theo n.

1. Co sd quy nap: Voi n = 2 dB thi tuong img G2 d§,y du c6 b3 - 1 = 5 dinh va 2 mau canh (xanh, do) khong c6 ee thi con K3 cling mau. Khi d6 G2 c6 th~ bi~u dien d dang hlnh 5 canh vdi canh cling mau do va duong cheo mau xanh.

That vay, do G2 d§,y du nen moi dinh xuat phat dung 4 canh diroc to bang 2 mau, Chinh xac han, tai tung dinh, moi mau diroc to tren dung 2 canh, Gia S11 ngiroc 11;Li, tai dinh A mau do diroc to tren 3 canh la AB, AC va AD. Khi d6 mot trong 3 canh BC, BD, CD mau do, dB thi c6 tam giac do. Ngiroc lai ca 3 canh d~u mau xanh, dB thi c6 tam giac xanh. Nhu vay mau thujin vdi gia thiet.

Gia S11 tai A c6 cac canh do la AB va AC (duong li~n net), con AD va AE la xanh, (dtrong net dirt}. Khi d6 canh BC phai xanh va ED Ia do (hlnh 1).

A

. ,

,

.

,

, ,

, ,

B ' , c

,.7'- - - - - - - - - -'\ ...

ciJ: .. ~

A

Hlnh 1

Hlnh 2

Hai canh BE va C E khong th~ cling xanh. Gia S11 BE do, thl C E xanh. Suy ra CD do va B D xanh. V~y ta dircc hlnh 5 canh voi canh mau do va dirong cheo xanh (hlnh 2).

2. Quy nap: Giii S11 khang dinh dung voi n = k. Xet dB thi Gk+1 d§,y du vdi b(k+I)+1 -1 dinh, k + 1 mau canh va khong c6 dB thi con K3 cling mau,

Moi dinh cua Gk+1 xu§,t phat (bk+1 - l).(k + 1) canh vdi k + 1 mau, nen phai c6 it nh§,t bk+l - 1 canh cling mau, Giii S11 tai dinh A c6 bk+1 - 1 canh cling diroc to bdi mau mI· Khi d6 trong cac dinh d6i cua A khong c6 cap dinh nao duoc n6i vdi nhau bdi canh

8

mau ml (trai lai thl c6 K3 cung mau ml). Xet dB thi con d§.y du Gk lap nen tu bk+1 - 1 dinh d6i cua A c6 canh chi to bdi k mau (trir mau ml) va khong c6 K3 cimg mau, nen theo gia thiet quy nap, trong Gk c6 hinh 5 canh vdi canh cung mot mau va dirong cheo la cac mau khac (t§,t ca d~u khong la mau mt). Vf),y trong Gk+1 c6 di~u c§.n khllng dinh,

3. Giai toan

1°) Xay d7,tng dB thi ma ta quan h~

Cac dB thi tucng irng vdi 3 bai toan dB. cho duoc xay dung nhu sau:

a) Dinh: L§,y 5 diem tren mat phiing, khong c6 3 di@m nao thllng hang tucng ling vdi 5 thanh vi en (5 s6 nguyen dirong, 5 d6i tirong dB. chon ra). Dung ngay ten cac thanh vien (cac t6, ten cac d6i tircng) d@ ghi tren cac di@m tirong img.

b) Canh: Dung

- Canh do d@ n6i giira hai dinh tucng ling voi hai ngiroi quen nhau (hai s6 co tree chung, hai d6i tucngco quan h~ tl)'

- Canh xanh d@ n6i giira hai dinh tuong irng voi hai ngiroi khong quen nhau (hai s6 nguyen t6 ciing nhau, hai d6i tirong c6 quan h~ t2)'

DB thi G, (1 ~ i ~ 3) mo ta toan bi) quan h~ di~u kien dtroc cho trong bai toan i, nen trong Gi khong c6 tam giac cung mau,

2°) Buy ra dap an. Theo khllng dinh 2 voi n - 2 dB thi G, la da giac 5 canh voi cac canh mau do va cac duong cheo mau xanh hoac nguoc lai. Khi d6 dua theo dirong g§,p khuc khep kin mau do rna ·s~px§p cac thanh vi en (cec s6; cac d6i tirong) tirong ling ngiroi xung quanh mot ban tron (len mot duong tron), thl moi thanh vien (moi s6, moi d6i tuong] se ngBi giira hai ngiroi rna thanh vi en c6 quen (dimg gitra hai s6 rna n6 c6 iroc chung; dirng giira hai d6i tuong rna n6 c6 quan h~ tl)'

3 Dang 3

3.1 Bai toan

Bai toan 1 Tren. miJ,t phiing za,y 6 diem tuy y, khang co 3 diim nao thiing hang va khoang c/ich. giila cdc ciJ,p diim kluic nhau titng d8i mot.

Chung minh rang tBn tq,i it nhat mot ciJ,p diim ma doq,nthiing n6i giila chung La conh. ngiin nhat cua mot tam quic nao do, dBng thai to, canh. dai nhat cua mi)t tam giac kluu: co dinh to, cac diim dii cho.

Bai toan 2 Chung minh rang trong n (n ~ 6) nguai tuy y luon chon duac n - 4 bi) ba, mi: trong m8i bi) ba nay hoiic titng d8i mot quen nhau hoiic titng dai mot kh8ng quen nhau.

Bai toan 3 Chung minh rang trong n (n ~ 6) 86 nguyen duotu; tily y lu8n luon choti dur,ic n - 4 bi) ba, mii trong m8i bi) ba nay titng ciJ,p 86 co UOc chung hoiJ,c nguyen t6 cilng nhau.

9

Bai toan 4 Chung minh rang trong n (n ~ 6) d6i tuonq tuy y lu6n chon ra d'l.t(fc n - 4 b(j ba, rna tronq moi b(j ba nay hoij,c tung ciip c6 quan h~ tl hoiic tung cij,p c6 quan h~ t2.

Bai toan 5 Vai n = 5 th'i cac khdng dinh. ph at bdu trong cacbai to an 2, 4 con dung nila kh6ng? ns« cac khdng dinh iren. khong dung hay cho phiin vi du.

3.2 Kh~ng dinh

Khang dinh 3 D6 thi day d'll g6m n dinh (n ~ 6) va duac to bang khong qua 2 mini canh, th'i lutm c6 it nhat n - 4 tam giac cung mau.

Chung minh. Truong hop 1:

DB thi d§;y du Gn co n dinh (n ~ 6) dU<;1C to bang mot mau canh. Khl1ng dinh de dang duoc chimg minh.

Truong hop 2: DB thi d§,y du Gn co n dinh (n ~ 6) vdi 2 mau canh (chl1ng han xanh, do). Ta phai khang dinh Gn co it nh§.t n - 4 tam giac cling mau, Di~u nay diroc chtrng minh bang quy nap theo s6 dlnh cua db thi.

1. Co sa quy nap: n = 6 thi db thi tuong irng G6 d§,y du voi 2 mau canh (xanh, do).

Ta phai chirng minh G6 co it nh§.t (6 - 4) = 2 tam giac cling mau.

Theo dinh 1y 2.19, G6 luon co it nh§.t mot tam giac cling mau, Do do ta phai chtrng minh trong G6 co them it nh§.t mot tam giac cling mau mra,

Khong m§.t tfnh t6ng quat, ta goi cac dinh cua G6 1a A, B, C, D, E, F va tam giac cling mau 1a tam giac ABC vdi cac canh mau do (net lien), (hlnh 3).

®

®

Hlnh 3

Hmh 4

Ta xet cac trirong hop sau co th~ xay ra:

1°) ea ba canh AD, AE, AF d~u diroc to mau do (hlnh 4).

Khi do, neu co it nhat mot trong ba canh DE, EF, DF do thl trong G6 co them it nh§.t mot tam giac cling mau mra (tam giac do).

Ngiroc lai, neu ca ba canh DE, EF, DF xanh (net dirt) thl trong G6 co tam giac cling mau thtr hai (tam giac xanh).

2°) Ca ba canh AD, AE, AF d~u diroc to mau xanh (hlnh 5). Chung minh tirong tv truong hop 1°).

10

,

,

,

, , ,

,

®

Hlnh 5

Hlnh6

Neu co it nh§,t mot trong ba canh DE,EF, DF xanh thl trong G6 co them it nh§,t mot tam giac nira cung mali (tam giac xanh).

Ngiroc 11;Li, ca ba canh DE, EF, DF do thl trong G6 co tamgiac cimg mali thu hai (tam giac do).

3°) Trong ba canh AD, AE, AF co hai canh do, chiing han AD, AE diroc to mali do (hlnh 6).

Khi do; n~li co it nh§,t mot trong ba canh CD, DE, CE do thl trong G6 co them it nh§,t mot tam giac mra cung mali (tam giac do).

Ngiroc 11;Li, ca ba canh CD, DE, CE xanh thi trong G6 co tam giac cung mali thii hai (tam giac xanh).

4°) Trong ba canh AD, AE, AF co dung motcanh do, chang han AD duoc to mau do (hlnh 7).

"

..........

'-'-"®

Hlnh 7

Hinh 8

a) Mot trong hai canh ED, CD do. Khiodo trong G6 co tam giac cung mau thii hai

(tam giac do).

b) Ca hai canh ED, CD xanh (hlnh 8).

b.) EF xanh: Khi do trong G6 co tam giac cung mau thir hai (tam giac AEF xanh). b2) EF do:

b2.1) EE do (hlnh 9)

b2.1.1) Hoac EF hoac CE do, ta co tam giac cung mau thir hai (tam giac do). b2.1.2) Ca hai canh EF va CE xanh (hlnh 10).

b2.1.2.1) Hoac DE hoac DF xanh, ta co tam giac cung rnau thu hai (tam giac xanh).

11

.1,

HInh 9

HInh 10

b2,1.2,2) Ca hai canh DE va DF d~u do, ta c6 tam giac cling mau thli hai (tam giac DEF do).

b2,2) BE xanh (hlnh 11).

HInh 11

Hinh 12

b2.2.1) DE xanh, ta c6 tam giac cling mau thir hai (tam giac BDE xanh). b2,2,2) DE do (hlnh 12).

b2.2,2,1) DF do, khi d6 ta c6 tam giac cling mall thir hai (tam giac DEF do). b2,2.2,2) DF xanh (hlnh 13).

b2,2,2.2,1) Hoac BF hoac CF xanh, ta c6 tam giac cling mau thti hai (tam giac xanh). b2,2.2,2.2) Ca BF va CF d~ll do, ta c6 tam giac cling mall thli hai (tam giac BCF

do).

V~y trong moi truong hop, G6 d~u c6 it nh§.t hai tam giac cling mau.

2. Quy nq,p: Gia su dinh 1:5' dung voi n = k. Xet dB thi Gk+l d§,y du vdi (k + 1) dinh, diroc to bang 2 mau xanh, do. Ta phai clnrng.minh Gk+1 c6 it nhat (k + 1- 4) = (k - 3) tam giac cungmau.

That vay, VI G k+1 d§,y du gom (k + 1) dinh (k + 1 ~ 7) vdi hai mau canh nen c6 it nh§.t mot tam giac cling mau,

Cia su cac dinh cua Gk+l la AI, A2, ••. ,Ak+1 va tam giac cling mau la tam giac AIA2A3 (chiing han mau do) (hmh 14).

LOl;Li Al va cac canh xuat phat tir Al ra khoi db thi Gk+1, ta c6 dB thi Gk vdi hai mall canh (xanh, do). Nen thea gia thi~t quy nap, trong Gk luon c6 it nh§.t (k - 4) tam giac cling mau,

12

@

,

. . .

"

Hlnh 13

Hlnh 14

Khoi phuc dinh Al cung cac canh thuoc Al ta dl1QC ae thi Gk+1 vdi hai mau canh (xanh, do) va Gk+I co It nhat (k - 4 + 1) hay (k - 3) canh cimg rnau. Kh~ng dinh dl1QC chtrng minh.

3.3 Giai toan

Gild bai toan 1. Xet t§,t ca cac tam giac co dinh 1a cac di~m da cho. VI khoang each gnra cac cap di~m da cho khac nhau tirng doi mot, nen moi tam giac co dinh la cac di~m da cho d§u co canh ngan nhat va canh dai nhat. D5i voi moi tam giac nay ta dung mall xanh d~ to canh ngan nhat. Sau khi t§,t ca cac doan thang n5i gnra cac cap diem da cho diroc phep to mau xanh da to xong, phan doan th~ng con lai to mau do.

D6 thi G nhan dircc la d6 thi d§.y du g6m 6 dlnh vdi cac canh dl1<;JC to bang 2 mall (xanh, do), nen thea khang dinh 3 trong G co ft nhat hai tam giac cung mau. VI tam giac nao ciing co canh ngan nh§,t dl1<;JC to mau xanh trudc, nen cac tam giac cung mau d§u la tam giac xanh. Khi do canh dai nh§,t trong moi tam giac nay la doan th~ng c§.n tim. Boi VI trong tam giac ta xet.no dong vai tro canh dai nh§,t, nhung VI doan th~ng nay co mau xanh, nen no da la canh ngiin nh§,t cua mot tarn giac nao do trong cac tam

giac co dinh la cac diem da cho. .

Gild bai toan 2-3.

1°) Xay d'l,tng d6 ihi rna til. quan h~.

Cac d6 thi tuong ling voi cac bai toan 2-4 dl1<;JC xay dung nhu sau:

a) Dinh: L§,y n diem (n ~ 6) tuong ling vdi n ngiroi (n s5 nguyen, n d5i tuong) da chon ra. Dung ngay ten ngirci (cac s5, ky hieu cac d5i tuong) d~ ghi tren cac di~m tuong ling.

b) Canh: Dung

- Canh do d~ n5i gitra hai diem tuong ling vdi hai ngiroi quen (hai s5 co uoc chung, hai d5i tuong co quan h~ tI).

- Canh xanh d~ n5i giira hai diem ttrong ling vdi hai ngiroi khong quen nhau (hai 85 nguyen t5 cung nhau, hai d5i tucng co quan h~ t2).

D6 thi Gi (2 ~ i ~ 4) nhan diroc mo ta toan bo quan h~ diroc cho trong bai toan i va tho a man di§u kien cua khang dinh 3.

2°) Buy ra dap an. Theo khang dinh 3 trong Gi (2 ~ i ~ 4) co It nhat n - 4 tam giac

13

cling mau,

- N@u tam giac mau do, thl ba nguoi tucng ling quen nhau tung doi mot (ba 86 tucng ling c6 iroc chung tung d6i mot, ba d6i tuong tircng ling tung doi mot c6 quan h~ t1);

- Neu tam giac mau xanh, thi ba ngiroi tucng irng khong quen nhau tung d6i mot (ba 86 tucng ling nguyen t6 cling nhau, ba d6i tucng tirong ling tung doi mot c6 quan he t2)'

Ghii bai toan 5.

V6i n = 5, thl cac khllng dinh phat bieu trong cac bai toan 2-4kh6ng con dung mra.

That vay, neu xuat phat tu da thi G d§.ydu gam 5 dinh (tirong ling vdi 5 d6i tucng diroc xet) voi cac canh duoc t6 bl1ng hai mau.

- Canhdo (net lien) bi~u hien quan he quen nhau (c6 iroc chung, c6 quan he tl)'

- Xanh xanh (net dut) bi~u hien quan he kh6ng quen nhau (nguyen t6 cling nhau,

c6 quan h~ t2)'

VI db thi G khong c6 tam giac cling mau (hlnh .15), nen khong c6:

Hlnh 15

- MOt bO ba nguoi nao tuong ling voi cac dinh rna hoac quen nhau tung doi mot hoac khong quen nhau tung doi mot,

- MOt bO ba 86 nao tuong ling voi cac dlnh rna hoac c6 iroc chung tung d6i mot hoac nguyen t6 cling nhau.

- MOt bo ba d6i tirong tuong ling vdi cac dinh, rna tung doi mot c6 quan h~ tl hoac tung doi mot c6 quan he t2•

4 Dang 4

4.1 Bai toan

Bai toan 1 Chung minh rang trong 9 nguai tuy y, rna ba nguai Mt ky dtu c6 2 nguai quen nhau luon tim dUr;!c 4 nguai dtu quen nhau (tung c(ip quen nhau).

Bai toan 2 Chung minh rang trong 9 88 nguyen dU(fng tuy y, rna ba 86 Mt ky dtu c6 2

14

s8 nguyen t6 cung nhau Zu6n Zu6n tim duoc 4 s6 nguyen t6 cimq nhau (tung ciJ,p nguyen t6 cung nhau).

Bai toan 3 Chung minh rang trong 9 d8i tuoru; tuyy, rna m8i ciJ,p d8i tUc;fng hoij,c chi co quan h~ i. hoiic chi co quan h~ ts, sao cho ba d8i tUc;fng btit ky d~u chua mot cij,p co quan h~ tl, lu6n lu6n tim duac 4 d8ituc;fng, ma tung dip co quan h~ u,

Bai toan 4 De mung nguiJi con doa; gidi trong kif thi ToanQu8c te zan thu 48, mat. gia d~nh du dinh. miJi basi den d'l,t ti~c. Trong s8 khach moi:

a) Nguai ua mu8n co it nhtit 3 nguai tung d6i mot quen nhau.

b) Nguai ch6ng Zq,i mu8n co it nhtit 4 nguai tung d6i mot chua quen nhau.

Hoi ho phdi min it nhtit bao nhieu ben, de it nhtit mong mu8n cua ch6ng hotic cua uo duac iluia man.

4.2 Khang dinh

D~ giai cac bai toan tren ta dua van kh§.ng dinh sau:

Khang dinh 4 1. D6 thi day du g6m 9 dinh vai hai mau canh: xanh, do, Zu6n hoij,c co tam quu: xanh (a) hoiJ,c co tit quic ma cae cordi va cac duang cheo d~u tniiu. do (b).

2. Neu d6 thi day du chi g6m 8 dinh, th~ khdng dinh iren. kh6ng dung.

Chung minh.

1) Xet dB thi d§,y dii G gBm 9 dinh: XI,X2,X3,X4,X5,X6,X7,X8,X9 voi cac canh d11<;lC

t6 b~ng hai mau: xanh, do.

Hai kha nang c6 th~ xay ra:

1°) C6 mot dinh thuoc it nh§,t 4 xanh mau xanh (net dill).

Gia S11 tai Xl c6 4 canh mau xanh la XIX2, XIX3, XIX4, XIX5. Khi d6 trong cac canh n6i doi mot giira 4 dlnh X2, X3, X4, X5 tBn tai it nh§,t mot canh mau xanh, ch§,ng han X2X3 mau xanh. Khi d6 tam giac XIX2X3 mau xanh (hinh 16). Kh§.ng dinh a) thea man. Ng11<;JC lai, n§u cac doan th§.ng n6i tirng cap trong cac di~m X2, X3, X4, X5 d~u mau do (net lien). Khi d6 tU giac X2X3X4X5 c6 cac canh va cac d11C1ng cheo d~u mau do (hrnh 17). Khang dinh b) d11<;JC thoa man.

2°) Khong c6 mot dinh nao thuoc qua 3 canh mau xanh. Khi d6 phai c6 it nh§,t mot dinh, chang han Xl 1a d§,u mut ella kh6ng qua 2 canh xanh (vI n~u moi dinh d~u xu§,t phat dung 3 canh xanh, thi s6 canh cua dB thi se la 3~9 khong nguyen). Boi vay, tai Xl phai xuat phat it nhat 6 canh do. Gia Stl' 6 canbdo xu§,t phat tir Xl la XIX4, XIX5, XIX6, XIX7, XIX8, XIX9 (hinh 18).

Xet dB thi con d§,y du GI gom 6 dinh X4, X5, X6, X7, X8, Xg voi hai mau canh xanh, do.

Theo khang dinh 1 voi n = 2 trong GI c6 tam giac cling mau.

Neu tam giac d6 mau xanh, thl kh§.ng dinh a) thea man.

15

, ,

".

. '.

/"'_/~"""""-'

~:::------------------.--:~

' .....

'\\ .........

, '.

, ,

,

®

Hlnh 16

Hlnh 18

N@u tam giac do mE1U do, thl ba dinh cua no hop vdi Xl thanh tli giac co cac canh va cac duong cheo d~u mau do, nen khang dinh b) thea man.

4.3 Giai toan

Giro bai toan 1-3.

1°) Xay d'(tng dB thi rna to. quan hf Cac d5 thi tuong ling voi cac bai toan 1-3 dircc xay dung nhu sau:

a) Dinh: L§,y 9 di~m tuong ling voi 9 ngirci tiiy y da chon ra (986 nguyen duong tuy y da chon ra, 9 d6i tuong tuy y da chon ra). Dung ngay ten cua ho (cac 86 da chon ra, cac d6i tirong duoc xet) d~ ghi tren cac di~m tirong ling.

b) Canh: Dung

- Canh do d~ n6i gifra hai dinh tuong ling voi hai ngiroi quen nhau (hai 86 nguyen t6 cung nhau, hai d6i tirong co quan h~ tl)'

- Canh xanh d~ n6i giira hai dinh ttrong ling voi hai nguoi khong quen nhau (hai 86 co uoc chung, hai d6i tirong co quan he t2)'

D5 thi G, (1 ~ i ~ 3) nhan duoc rna ta toan bQ quan h~ diroc cho trong bai toan i va thea man dieu kien cua khang dinh 4.

2°) Buy ra dap an. Theo khling dinh 4 trong G, (1 ~ i ~ 3) hoac co tam giac xanh

16

hoac co tu giac voi cac canh va duong cheo d@u do. Nhirng thea dieu kien d~t ra trong cac bai toan 1-3 va each to mau canh tam giac b§.t ky trong cac d5 thi Gi (1 :::;; i ~ 3) d@u co it nh§.t mot canh do. Boi v~y, trong cac d5 thi Gi (1 ~ i :::;; 3) d@u co tu giac voi cac canh va dirong cheo mau do, nen 8Uy ra khang dinh duoc phat bi@u trong cac bai . toan 1-3.

Gild bai toan 4. D5 thi d§.y du g5m 9 dinh voi hai mau canh: xanh, do luon luon co tam giac xanh hoac tu giac vdi canh va dirong cheo d@u mau do. Nhung khi 86 hrong dinh chi con la 8, thl t5n tai d5 thi d§.y du voi hai mau canh: xanh (net dut bi@u hien quen nhau), do (net lien bieu hien khong quen nhau), rna tren do khong co mot tam giac xanh nao cling nhu khong co mot tu giac nao voi cac.canh va dirong cheo d~u mau do (hmh 19).

Hlnh 19

Boi vay, d@ hoac thea man yeu c§.u cua vo: trong 86 khach moi co ba ngiroi quen nhau tung doi mot (tirong irng vdi ba dinh cua mot tam giac xanh) hoac thea man yeu c§.u c§.u cua chong: trong 86 khach moi co 4 ngiroi rna tirng cap khong quen nhau (tuong irng voi dinh cua mot tu giac rna cac canh va dirong cheo d@u mau do), thl 86 khach moi t6i thi@u phai tucng irng vdi dlnh ciia mot d5 thi d§.y du gom 9 dlnh, ttrc la d~ thoa man yeu c§.u hoac cua chong hoac cua VQ thi phai moi it nh§.t 9 khach.

17

Logic hrnh t.htrc va ap d ung

Nguyen Van Mau

1 Mil d§.u

Ly thuyet va cac bai toan t6 hop la mot b¢ phan quan trong, h§,p dan va ly thu cua Toan hoc roi rae, C6 nhieu v§,n d~ v~ co sa lap luan va ly thuyet dU<;1C trlnh bay duoi dang r§,t don gian nhung pham vi img dung thl r§,t phong phu va da dang: Nhi~u bai toan cua thuc tien doi s6ng c§.n diroc giai quyet th§,u dao, c§.n d~n mot co sa lap luan c6 dO tin cay cao va c6 kha nang ap dung mot each thong dung. Gtin chat vdi cac bai toan t6 hop va cac bai toan roi rae la cac bai toan v~ logic, v~ quy luat ciia nhtrng phep toan va cac tinh ch§,t noi tai cua t?,p hop,

Cac bai toan so c§,p lien quan d~n t6 hop va logic thuong xu§,t hi en trong r§,t nhieu bai toan ly thu va thucng r§,t kh6. Trong bai nay trlnh bay lai k~t qua cua Sylvester v~ logic hlnh thirc (xem [1]). Phan logic hlnh thirc nay khong nam trong ly thuyet logic menh d~ quen biet, Tuy nhien, n~u nhln theo g6c dO co sa l?,p luan cling nhir each thirc dan diit v§,n d~, thl logic hlnh thirc hoan toan tuong d6ng vdi each liet ke cac phan tu (hrc hrong) cua cac phep tinh t?,p hop nhir phep hop, phep giao, cong thirc bao ham va loai trir, ...

2 T~p hQP, cac phep t inh cd ban

Trude het, ta nhac lai mot s6 khai niem co ban lien quan d~n t?,p hop. Khi cho mot t?,p hop ttic la ta c6 th~ chi ra dU<;1C each rna ta t§,t ca cac phan tli cua n6. Cac phan tu cua mot tap hop c6 th~ mo ta bang each liet ke chung (thuong la cho t?,p hiru han phan tli) hoac la chl ra cac tinh ch§,t d~c trirng cua chung.

86 pban tli trong t?,p A dU<;1C goi la lire hrong cua t?,p A va duoc ky hieu bdi IAI. Moi tap B rna moi phan tli cua n6 d~u thuoc t?,p A d~u diroc goi la t?,p con cua t?,p A va viet B ~ A. Tap hop khong chua mot phan tli nao duoc goi la t?,p hop rong (hay t?,p hop tr6ng) va thuong diroc ky hieu bang 0. Ta coi t?,p hop rong la t?,p con cua moi t?,p A cho tnroc.

Truong hop B la t?,p con cua t?,p A va B =1= A, thl B duoc goi 180 t?,p con khong tam thuong (hay t?,p con thirc s11) cua t?,p A va vi~t B c A.

Gia sli t?,p B 180 t?,p con cua tap A. np gom t§,t ca cac phan tli thuoc A, nhung khong thuoc B duoc goi 180 t?,p phan bu (hay phan bu) cua tap B (d6i voi t?,p A) va ky hieu hoac bang CA(B).

18

tinh chat niio trong s8 ctic tinh chat a,/3, "(,... va A da li?t ke la

N - NOt. - N{3 - N-y - ... - NK, - N).. + NOt.{3 + NOt.-y + ... + NK,)..

- NOt.{3-y -"'-

Chimg minh.

Cia sir s6 hrong cac tinh ch§.t da liet ke a,/3, "(, ... , /'i, va A la n. Neu mot d61 tirong nao do trong s6 da cho co k tfnh ch§.t (1 ~ k ~ n), thl tang da neu trong dinh ly cho ta

don vi, Ngiroc lai, n~u mot d6i tuong nao do khong co b§.t ctr mot tinh ch§.t nao trong s6 cac tfnh ch§.t da liet ke a,[3, "(, ... , /'i, va A thl no cho ta 1 don vi va VI v~y bang s6 hang N. 0

H~ qua 1 Cia thiet da cho truce n d8i tU(jng tuy y duac slip xep theo m9t tr9.t t7,t va cae d8i tU(jng d6 c6 cac tinh chat a,/3, "(, ... va A nao d6. Cia s'li tat co, ctic d6i tU(jng d€u c6 tinh chat a tri: d6i tU(jng dau tien, tat co, ccc d6i tU(jng d€u c6 tinh chat [3 tru d6i tU(jng th71 hai, ... , va tat co, cdc d6i tU(jng d€u co tinh chat A tr'll d6i tU(jng xep cu8i cung. Khi d6 cong tluic trong Dinh. ly 1 c6 dr;mg

n - (~)(n -1) + (;)(n- 2) - ... + (-lr(:)(n - n).

(1)

Chimg minh.Th~t vay, trong truong hop dang xet, ta co N = n, NOt. = Nf3 = N; = ... = N).. = n -1,

Net{3 = Net-y = Net).. = ... = NK,).. = n - 2,

V~y nen

n - (~) (n - 1) + (;) (n - 2) - ... + (-1r(:}n - n) = O. (2)

do moi d6i tirong d§u co it nh§.t mot tinh ch§.t da liet ke. 0

Ti@p theo, ta phat bieu mot mo rong cua dinh ly 1.

Cac kien thtrc da trlnh bay lien quan mat thiet d~n tfnh toan lire hrong cua t~p hop rna chung ta da r§.t quen biet trong chuang trlnh pha thong.

20

chuy€n va bong r6; 3 nguoi tham gia bong chuy€n va b6ng r6; 3 nguoi tham gia cau long, bong ban va b6ng r6; 2 nquai tham gia cau long, b6ng chuy€n va bong r6; 2 nguoi tham gia dang thai cau long, bOng ban va b6ng chuY€n; mot nguoi tham gia dang thoi Mn man cau long, bong ban, bong chuy€n, bOng r6. Hoi c6 bao nhieu v{j,n dong vien tham gia thi . dau chinh thuc man bai?

Tuong tu, ta co loi giai eho bai toan t6ng quat sau day.

Bai toan 3 Cho bo s8 nguyen dUdng aI, a2, ... , am (m > 1) doi mot nguyen t8 cung nhau. Hoi trong cac s8 ihuoc {I, 2, ... , n} (n > 1) co bao nhieu s8 khong chia htt cho Mt cu s8 nao thuoc {aI, a2,· .. , am}.

Gicii. Nhan xet rling s6 cac s6 nguyen dirongkhong virot qua n chia h~t cho s6 a (1 ~ a ~ n) bling [~]. Ti~p theo, s6 cac s6 nguyen dirong khong virot qua n d6ngthoi

ehia h~t cho s6 a va b (1 ~ a, b ~ n va a, b nguyen t6 cung nhau, bang [:b] , ...

V~y nen s6 cac s6 thuoc {I, 2, ... , n} (n > 1) khong chi a h~t eho b§.t cu s6 nao thuoc {at, a2, ... " am} tinh bang cong thirc

n- ([~] + [2] + ... + [~])

+ ([a:J + [a:J + ... + Ln:aJ)

Nhan xet rling, khi cac tfnh eh§.t diroc liet ke nhu trong Dinh 19' Silvester khong e6 tinh doc l~p thl cong thirc tfnh toan phai thay d6i. Ta xet VI du sau day.

Bai toan 4 Hoi trong t{j,p A = {I, 2, ... , 280} co bao nhieu. s6 khong chi a htt cho 2, 3, 4,5,6,7.

Gilli. Nhan xet rling, s6 cac s6 trong A khong ehia h~t eho mot trong cac s6 2, 3, 4,5,· 6,7 ciing bang mot s6 cac s6 trong A khong chi a h~t cho mot trong cac s6 2, 3, 5, 7. Ta xet xem trong t~p A co bao nhieu s6 chi a h~t cho mot trong cac s6 2, 3, 5, 7. Kf hieu Alia s6 cac s6 thuoc A ehia h~t eho 2, A2 la s6 cac s6 thuoc A chia h~t cho 3, A3 1a s6 cac s6 thuoc A ehia h~t eho 5 va A4 la s6 cac s6 thuoc A ehia h~t eho 7. Khi do Al U A2 U A3 U A4 la tap cac s6 chia h~t cho mot trong cac s6 2, 3, 5, 7. Ta co

280 [280] 280 280

IAII = 2 = 140; IA21 = 3 = 93; IA31 = 5 = 56; IA41 = 7 = 40, va

[280] [280]

IAI n A21 = (3 = 46; IAI n A31 = 10 = 28;

22

[280] [280]

IAI n A41 = 14 = 20; IA2 n A31 = 15 = 18;

[280] [280]

IA2 n A41 = 21 = 13; IA3 n A41 = 35 = 8,

va

lA, n A, n Aal = [23~] = 9; lA, nA, n .1,,1 = [~20] = 6,

lA, n Aa n A.I = [~OO] = 4; lA, n Aa n A.I = [~~~] = 2, IA1nA2nA3nA41 = [~~~] = 1.

Sa dung cong thirc cua Dinh ly Silvester, ta thu diroc trong t?,p A c6 64 s6 khong chia h~t eho 2, 3, 5, 7, tire trong A c6 64 s6 kh6ng chia h~t cho 2, 3, 4,5,6, 7.

Ti~p theo, ta phat bi~u Dinh ly Sylvester cho truong hop c6 trong s6.

Dinh If 3 Cia thi€t dii cho truce N d6i tuanq tuy y. Cia 8U No. la 86 cdc d6i tU(Jng trong 86 dii cho co tinh chat o, /3", ... , A va m8i tinh chat dii cho giin vrJi mot gia tri 86 (tr9ng 86). C9i Vo. la t6ng c~c ironq 86 cua ctic d6i tuanq trong 86 dii cho c6 tinh chat ll', Vp la t6ng cac tronq 86 cua cac d6i tU(Jng trong 86 dii cho c6 tinh chat /3, v'")' la t6ng cac tr9ng 86 cua cdc d6i tU(Jng trong 86 dii cho c6 tinh chat " ... va VA la t6ng cdc troru; 86 cua cdc d6i tU(Jng trong 86 dii cho c6 tinh chat A. Tuatu; tu, gia 8U Vo.,B, Vo.'")', ... , Vo.,B'Y"'" Vo.,B'Y ... A la t6ng cac tronq 86 cua cac d6i tuanq trong 86 dii cho c6 d8ng thai cac tinh chat ll' va /3, tUdng ung, ll' va " ... , tuaru; ung ll',/3 va " tuanq ung ll',/3, " ... va A. Cia 8U V t6ng cac tronq 86 cua tat co, cac d6i tU(Jng dang xet. Khi d6 t6ng cac troru; 86 cua cac d6i tU(Jng khong co tinh chat nao trong 86 cac tinh chat ll',/3, " ... va A dii li~t ke la

V-Vo.-V,B-v,-···-VI\:-VA + Vo.,B + Vo.'Y + ... + VI\:A

- Vo.,B'Y - "'-

Chung minh. Phuong phap chirng minh hoan toan nhtr d6i voi Dinh ly Sylvester khi ta giin moi d6i tirong cling mot gia tri bang 1.

23

'" ... .

Cong t.htrc tinh so phan ttl' cua mot

hop cac t~p hop

vn Dinh Hoa

Trong CU9C s6ng nhieu khi xu§.t hien nhirng bai toan phai tfnh s6 hrong phan tu cua mot t~p hop thong qua nhirng t~p hop con cua chung. Chang han:

Vi du 1 Trong mat bai kiim tra iocn. co hai biu toano Trong cd lap co 30 em lam dU(1c bai tlni nhtU va 20 em lam dU(1c bai thithai. Chi coLt) em lam dniac cd hai bdi to an kiim tra. Hay tinh. s6 hoc sinh trong lap.

GQi A la t~p hop hoc sinh giai diroc bai toan thu nhat, va B la t~p hop hCQ sinh giai diroc bai toan thtr hai, thl An B la t~p hop hoc sinh giai diroc ca hai bai toano Bai toan diroc d~t ra la phai tfnh s6 phan tu cua A U B.

N§u A va B la hai t~p hop roi nhau, thl th§.y de dang r[ng:

IAnBI = IAI + IBI·

Trong tnrong hop A va B c6 giao khac rang thl d~ng thirc tren khong con dung mra, rna ta se c6 cong thirc

IAUBI = IAI + IBI-IAnBI,

bai VI trong tiSng IAI + IBI thl cac phan tirchung trong IA n BI (cua ca A va B) duoc tinh lap dung hai l§.n.

Su dung cong thirc nay, ta th§.y s6 hoc sinh cua lop trong vi d\l tren cua ta 18. 30 + 20 - 10 = 40 em.

Nhieu khi, bai toan ta g~p tra nen phirc tap han khi phai tinh s6 phan tu cua mot tap hop c6 nhieu han hai t~p hop.

Vi du 2 Lap 12A phdi lam m¢t bdi kiim tra Totui g6m co ba bdi toano Bitt ding m8i em trong lap d€u lam dU(fc it nhtU m¢t biii, trong lap co 20 em lam duac bdi toan thit nhat, 14 em gidi dU(fc bai iotui thit hai, 10 em gidi dU(1c biii totin th'l1 ba, 6 em gidi dU(1C cd hai bdi ioam. th'l1 nhat va th'l1 ba, 5 em gidi dU(fc cd hai bdi th'l1 hai va th'l1 ba, 2 em gidi dsiac cd hai bai thit nhat va th'l1 hai, va co mot em duoc 10 diim vi da gidi duoc cd ba bdi toano Hoi ding lap h9C co bao nhieu em tat cd?

GQi A la t~p hop cac em hoc sinh giai diroc bai toan thu nhat, B la t~p hop cac em hoc sinh giai diroc bai toan thtr hai va C la t~p hop cac em hoc sinh giai duoc bai toan

24

thir ba, ta phai tmh s6 phan tu cua t?,p hop Au B u C. Trong nhieu truong hQP khac, chung ta phai tfnh s6 phan tu cua mot hQP g6m nhieu t?,p hQP con, va phan nhieu cac bai toan nay la cac bai toan kh6 voi hoc sinh khong h@ duoc hoc c6ng thirc tfnh t6 hQP.

1 X~p co l~p vdi tftn 86 cho trtioc

MOt phan tu a, cua mot day k phan tu cho tnrdc al a2 ... ak duoc n6i la co t§,n s6 lap ki' n~u nhu n6 xu§.t hien trong day dung k; 1§,n. Trong muc nay chung ta xet bai toan sau:

Bai toano Cho truac m9t tq,p hop A = {ali a2; ... ; an} va k La mat s8 dUdng nguyen cung vai cec s8 tit nhien. kl' k2, ... , k« thoa man ~:'lki - k. Hay tinh s8 P(k1' k2, ... , kn) cae day k phan tit cua tq,p hop A sao cho phan tit a, ciLa A c6 tan s8 Lij,p Za k; cho moi i ~ n. De dati gio'n h6a trinh. My, ta sf. n6i qot: Za day k ph an tit c6 Zij,p (kl' k2, ... , !s;n) neu k;

Za tan s8 Zij,p ciLa phan tit ai trong day. .

Vi du 3 Cho t(lp h(Jp A = {a; b; c; d}. Tiit co, cac day c6 Zij,p (3,2,0,0) ciLa A Za

aaabb, aabba, abbaa, bbaaa, aabab, ababa,babaa,abaab,baaba,baaab.

Dinh If 1 Cho truce mot t(lp h(Jp A c6 ti phan tit aI, a2, ... ,an' 88 cae day c6 a9 deli k vai k = ~i=l k; phan tit sao cho ph an tit a; xuat hi~n dung ki Zan trong day La

Chung minh Khong m§.t tinh t6ng quat c6 th@ gia su ki ~ 1 cho moi gia tri i ~ n. Ta xet t?,p hQP

B - {a1·a2. ·akl.a1·a2. ·ak2. ·a1·a2. 'akn}

- 1l 1"", I' 2' 2"", 2,···, n' n"", n .

Nhu v?,y moi day c6 lap (k1' k2, ... , kn) 1a mot hoan vi cua cac phan tu cua B (t§.t ca c6 (k1 + k2 + ... + kn)! = k! hoan vi), trong d6 moi day bi tinh 1l;ip kl!k2!'" kn! 1§,n, do khi d6i cho cac phan tu at (1 ~ j ~ ki) vdi nhau ta van chi thu dircc chinh day d6 rna thoi.

V?,y s6 cac day c6 lap (kl' k2, ... ,kn) la

Vi du 4 Tinh s8 cac s8 tit nhien. c6 My chit s8, trong d6 c6 ba chit s8 1, hai chit s8 2 va hai chit s8 3.

Gio'i. S6 cac s6 tv nhien c§,n tim 1a

7!

P(3, 2, 2) = 3!2!2! = 210.

25

2 Khai tri~n lily t.hira cua nhi thue

Chung ta da bi~t nhimg cong thirc khai tri§n nhi tlnrc sau

(x + y) 2 = x2 + 2xy + y2,

(x + y)3 _ x3 + 3x2y + 3xy2 + y3.

Neu nhan ca hai v~ cua dl1ng thirc cu6i vdi x + y, ta nhan dU<;1C cong thuc sau:

Chung ta co cong thiic t6ng quat tinh h~ s6 cua (x + y)n (con goi Ia cong thirc nhi thirc Newton) nhu sau:

D!nh If 2 Cha m8i 88 t'l/ nhien n ;;::: 1 ta co

n

(x+yt = LC~xn-kyk.

k=O

Chung minh. D§ khai tri§n (x + y)n, ta thirc hien phep khai tri§n Illy thira mot each hlnh thirc rna khong rut gon chung, Chl1ng han

(x + y)2 = xx + xy + yx + yy.

Nhu v~y ta co th~ bi§u diE~n hlnh tlnrc

n

CI d6 c6 t§.t ca 2n s6 hang CIC2 ... en vdi c, E {x; y}. Trong khi thirc hien phep rut gon, ta phai dem nh6m t§.t ca cac s6 hang c6 cling s6 mii cua x va y lai vdi nhau. Vdi moi o ~ k ~ n, ta tfnh s6 cac day n phan tu trong d6 x lap k 1iln con y lap n - k 1iln. Theo

, ,. " n! 'k

dinh 1y 1 da biet, thl so cac day c6 lap theo tan so nay la k!(n _ k)!' chinh bang Cn. Do

d6 ta c6:

n

(x + yt = L C~xn-kyk.

k=O

Vi du 5

(x + y)5 = cgx5 + Cgx4y + C~x3y2 + C~x2y3 + Ctxy4 + Cgy5 = x5 + 5x4y + lOx3y2 + lOx2y3 + 5xy4 + y5.

(2x + 3)5 = 32x5 + 240x4 + 720x3 + l080x2 + 810x + 243.

26

Luu y. Tam giac Pascal, trong d6 s5 b[t d§.u va k~t thuc cua moi dong trong tam giac Pascal la s5 1, va moi s6 khac ciia moi dong bang t6ng cua hai s5 cua dong tren n6, cho each tinh nhanh ch6ng he s5 cua nhi thirc Newton.

1

1 1 2

1

1

1

3

3

1

1

4

6

4

1

3 Tfnh 86 phan tit cua mot t~p hop cac t~p hop

Quay lai vdi vi du 2 da cho ban d§.u: GQi A la t~p hop cac em hoc sinh giai dtroc bai toan thtr nhat, B la t~p hop cac em hoc sinh giai diroc bai toan thu hai va C la t~p hop cac em hoc sinh giai diroc bai toan thu ba, ta phai 'tinh s5 phan tu cua t~p hop AUBuC.

Khong kh6 khan, cluing ta c6 th§ th§.y cong thirc sau la dung:

IAUBU CI = IAI + lEI + ICI-IAnBI-IE nCI-lcnAI + IAnBncl.

Theo cong thirc cua ta, s5 hoc sinh trong lop se la:

20 + 14 + 10 - 6 - 5 - 2 + 1 = 32.

Nhung chung ta c6 th§ b[t g~p nhirng bai toan c6 su tham gia cua nhieu han ba tap hop con, chang han:

Vi du 6 Tinh. s6 cadi treo 5 d6i tat tren mot day phoi sao cho kh6ng co hai chitc tat nao cung d6i mot duac phai canh. nhau.

D§ giai bai toan nay, cling nhir nhieu bai toan tuong tu, ta chirng minh dinh ly sau:

Dinh If 3 Cho truoc ctic t{),p hop AI, A2, ... , An. Khi do ta co

n

IAI U A2 U··· U Ani = L IAil- L IAi n Ajl + ...

i=1 I »<i n

ChUng minh. Dinh ly nay c6 th§ dUc;1C chirng minh hai each:

Cach 1: Chung minh bang quy nap theo n. Voi n = 1 hi§n nhien d&ng thirc (*) dung.

Voi n = 2, ta cling de ki§m tra d§ th§.y r[ng d&ng thirc (*) dung.

27

Ta gia 8U (*) dung cho n ~ 2 t?,p hop tuy y.

n

IAI U A2 U ... U Ani - L IAil - L IAi n Ajl + ...

i=l I i<j n

Bay gia xet n + 1 t?,p hop tiiy Y AI, A2, ... , An, An+l' Luu y r~ng

n

(AI U A2 U··· U An) n An+l = U(Ai n An+l).

i=l

(**)

Cho nenta c61(A1 U A2U .. · UAn) nAn+l1 = U~=l (Ai nAn+l)I. 8u dung (*) cho v~ phai cua (**), ta thu diroc

n

I(AI U··· U An) n An+ll = L IAi n An+ll- L IAi n Aj n An+ll+

i=l

1 i<j n

1 i<j<k n

- ... + (-.It-1IA1 n A2 n··· nAn n An+ll.

DM A = A1UA2u·· ·UAn va B = An+l va ap dung dllng thtrc IAUBI = IAI+IBI-IAnBI (truang hop n = 2), ta thu diroc di~u c§,n chiing minh

n+I

IAI U .. · U An U An+ll = L IAil- L IAi n Ajl + ...

i=l 1 i<j n

- ... + (-1)n-IIAl n A2 n··· nAn n An+ll.

V?,y dinh 1y diroc chirng minh cho trirong hop n + 1. Dinh 1y duoc chtrng minh.

each 2: Ta xet mot phan tu a E Al U A2 U ... U An b§.t kyo Gia 8U r~ng a thuoc VaG r (1 :::;; r :::;; n) t?,p hop trong 86 cac t?,p hop nay. 86 l§,n xu§.t hien cua a trong cong thirc (*) theo dinh 1y 2 cho x = 1 va y = -1 1a

C;: - C; + C; - ... + (_Iy'~IC; = 1 - (1 - It = 1.

V?,y (*) 1a mot dllng thirc dung.

Liru y. Trong trirong hop dac biet khi

lSI = m, IS - (AI U A2 U .. · U An)1 = mo, IAil = mI, i = l,n,

IAi n Ajl = m2,

IAi n Aj nAkl = m3,

... ,

28

thl ta c6

IAI U A2 U··· U Ani = C~ml - C~m2 + C~m3 - ... + (-It-IC;:mn, ma = m - C~ml + C~m2 _ ... + (-ltC;:mn.

Bai toan trong vf du 6 giai de dang bang each su dung cong tlnrccua dinh 15' 1 va dinh 15' 2.

Boo t~p 1 Trang mM ky thi h9C sinh gioi Totui, LV, H oa co mot s6 em tham gia. Bi~t rd.ng co 20 em tham gia thi Totin, 14 em tham. gia thi LV, 10 em tham gia thi Hoa, 6 em vita thi Totiti vita thi LV, 5 em vita thi LV vita thi H oa, 2 em thi Totui va thi H oa va co mot em tham gia tat ca ba ky thi Totui, Lv va H oa. Hoi rang co baa nhien em tham gia ky thi hoc sinh gioi nay?

Boo t~p 2 Tinh s6 ctic hotui vi cua day chit "TOANHOC" sao cho khong co hai chit cai niu: gi6ng nhau dung canh. nhau.

Boo t~p 3 Tinh s6 cac hocn vi cua day chit "XAXAM" sao cho khong co hai chit cai nao gi6ng nhau dung canli nhau.

Boo t~p 4 Bay da quic deu codien tich la 1 nam trang mot h'inh vuong co d(j dai canli 10, 2. Chung minh rang it nhat co hai da giac cift nhau vai dien ticli phan chung khong nho luni ~.

5 Tai lieu tham khao

[1] Vii Dlnh Hoa, M(jt s6 ki~n thuc ca sa ve h'inh hoc t6 hap, Nha xu§.t ban Khoa hoc va Ghill due, Ha NQi, 1999.

[2] Vii Dlnh Hoa, Lv thuy~t t6 hop va bai ttj,p ung d1,mg, Nha xuat ban Ciao due, Da N[ng, 2002.

29

?

Mang lu'oi 0 vuong tren mat phang

Vl1 Dlnh Rca

1 Da giac co canh khong tv ciit

Trong hlnh hoc phang chung ta da lam quen vdi nhieu hlnh 16i, chiing han cac hlnh tam giac, cac hinh vuong ... Trong sach giao khoa, cac da giac 16i duoc dinh nghia nhu sau : m(jt da giac dUf/e qoi la da quu: 18i khi no nil,m hoan. toiui v€ m(jt phia cua d'l1ong thang di qua m(jt cordi Mt k'i eua da giae.

N~u tren m~t phang cho tnroc mot t~phc;:Jp n ~ 3 di~m thi khi n6i cac di~m vdi nhau boi cac doan thiing co dinh la di~m trong n di~m da cho thl ta thu diroc mot da giac 16i hoac mot doan thiing chua cac di~m con lai 0 ben trong, va ta goi no (da giac 16i hoac doan thiingnay) la bao l8i cua t~p hop n di~m nay.

R§.t nhieu bai toan cua 11 thuyet t6 hop co th~ giai bang each van dung bao 16i cua cac hlnh hoac cua tap hop di~m cho trudc,

Vi du 1 Treti miit phang cho truac mot s6 diim khong ciuu; nil,m tren. mot d'l1ong thang. Chung minh rang t8n ici ba diim sao eho duong troti di qua ba diim nay khong chua ditfm nao eho truac iJ ben trong.

Ta xet bao 16i cua t~p hop cac di~m cho trirdc nay. Do co ba trong s6 cac di~m nay khong thiing hang, cho nen bao 16i cua chung la mot da giac 16i. Xet mot canh AB cua da giac 16i nay. Trong nhfmg dinh con lai, gia Slr C la dinh nhin canh AB voi mot goc IOn nhat. Khi do dircng tron ngoai tiep tam giac ABC khong chua di~m nao da cho ben trong.

Ngoai tam giac la da giac co 3 dinh luon la hlnh 16i, mot n-giac b§.t kl vdi n ~ 4 co th~ khong phai la da giac 16i. Mot da giac duoc goi la da giae co canh. khong t'l,t edt n~u nhir cac canh cua no doi mot khong cEtt nhau tnt nhirng canh lien ti~p co th~ c6 d§.u rmit chung. MQi da giac 16i cung la da giac co canh khong tu cEtt. Cho truce mot da giac co canh khong tu cat, chung ta luon co th~ chia no thanh cac hlnh 16i, cu th~ 18. chia no thanh cac tam giac, bdi cac duong cheo khong cEtt nhau trong da giac, D~ chtrng minh di~u nay, truce h~t ta chirng minh k~t qua sau:

Dinh If 1 Trong mnt da quic co camh. khong t7,t edt luon. co m(jt duong cheo nil,m hoar: toiui trong no.

That vay, neu da giac da cho la da giac 16i thl khiing dinh cua bai toan la hi~n nhien, Nguoc lai, neu da giac da cho khong phai la da giac 16i thi no co mot dinh, goi la A,

30

e6 g6e Ion hon 1800• Tir dinh A nay ta ke mot tia Ax nao d6, e~t bien cua da giac tai mot diem M. Do A> 1800, eho nen e6 mot trong hai canh ben cua A, changhan AB, sao eho tia AB (di tir A tdi B) khong d,t canh cua da giac e6 chua di~m M. Khi ta cho . tia Ax quay quanh A theo chieu tir vi tri ban d§.u Wi vi tri AB, thl di~m M chay tren bien cua da giac va I UC nao d6 n6 phai di chuyen tir mot canh nay sang canh khac cua da giac da cho. T9-i thai di~m d6, tia Ax phai di qua mot dinh C nao d6 khac A cua da giac, Luc d6 dirong cheo AC chinh la duong cheo nam hoan toan trong da giac,

Dinh If sau day hi~n nhien la h~ qua cua dinh Ii tren:

Dinh If 2 M9t n-giac co cordi kh6ng t'l,t cat lu6n co thi chia thank n - 2 tam giac Mi cac duang cheo kh6ng cat nhau n6.m hoom toim trong da giac.

Ta chirng minh khang dinh bang quy nap theo s6 dlnh cua da giac,

Vdi n = 3 khang dinh cua dinh 11 hien nhien dung. Gia su k~t luan cua dinh If dung cho moi k-giac (k :::; n) c6 canh khong tv c~t.

Ta xet mot (n + l)-giac c6 canh khong tv c~t b§.t kl. Theo dinh If da chirng minh tren thl ta c6 th~ chi a n6 thanh hai da giac vdi k va f. dinh c6 canh khong tv c([t boi mot dirong cheo nam hoan toan trong da giac, Ta c6 k, f. ~ 3 va k + f. = n + 3. Theo gia thi~t quy nap thl cac k-giac va f.-giac nay e6 th@ chia thanh k - 2 va f. - 2 tam giac bdi cac dirong cheo khong c~t nhau va hoan toan nam trong chung. B[ng each do, ta da chi a (n + 1 )-giac da cho thanh k + f. - 4 = n - 1 tam giac boi cac dirong cheo khong c([t nhau va hoan toan nam trong (n + 1 )-giac da cho. Dinh If diroc chirng minh cho tnrong hop (n + l)-giac.

Ta goi mot tam giac co cac dinh la dinh hroi cua mang 1110i 0 vuong la tam giac dot: neu nhir n6 khong chua mQt dlnh hroi nao khac tren canh hoac ben trong tam giac.

Dinh If 3 Ctic dinh cua mot k-giac co comli kh6ng t'l,t cift F (kh6ng nh[U thiet pluii l8i) nlim rJ cac diim nguyen. Ben trong no co n diim nguyen, con tren bien m diim nguyen. Ta n8i cac diim nguyen loi vai nhau b6.ng cdc dotu: thilng kh6ng ciit nhau sao cho tIlt co' cae tam quic thu duac la tam quic nguyen, khi do s8 tam quic dati thu duac b6.ng nhau va bting dung 2n + m -2.

GQi s la s6 cac tam giac don thu diroc. T6ng cac goc cua starn giac don nay la 1f x S.

T6ng nay bang dung t6ng cac goc quanh cac di~m d ben trong F va cac di@m tren chu vi cua F (g6m cac dinh cua F va cac di@m nguyen d tren bien' cua no). T6ng cac goc quanh cac dinh cua F bang 1f x (k - 2) (do F c6 th~ chia thanh k - 2 tam giac boi cac dirong cheo khong tv c~t d hoan toan ben trong n6 theo dinh li tren). T6ng cac g6c quanh moi di~m nguyen d tren canh cua F la 1f, va t6ng cac g6e quanh moi dinh trong F la 21f. Do d6 ta c6 dl1ng thirc:

1f X (k - 2) + 1f X (m - k) + 21f x n = tt X S.

Va ta thu diroc dl1ng thirc 2n + m - 2 = S.

Bai t~p

31

1. Chung minh rang mot da giac la da giac 16i khi va chi khi b6n dinh b§,t kl cua chung t9-0 thanh mot tu giac 16i.

2. Tren mat phii.ng cho truce nam di~m sao cho khong c6 3 di~m nao trong chung : thii.ng hang. Chung minh ding c6 th~ chon ra trong chung 4 di~m la dlnh cua mot tu giac 16i.

3. Tren mat phang cho triroc n 2:: 5 di~m trong d6 khong c6 3 di~m nao thii.ng hang.

Chung minh r~ng t6n tai it nh§,t C~-3 tu giac 16i rna cac dinh cua clning 1a 4 di~m trong s6 n di~m da cho. (Ki thi Toan quoc t§ l§.n thir 11, nam 1969)

4. Tren mat phang cho mot s6 hiru han di~m. Chung minh r~ng luon tim d11<;1C mot di~m sao cho g§.n n6 nh§,t c6 khong qua 3 diem da cho.

5. Tren mat ban d?,t n hinh vuong bang cat-tong va n hlnh vuong bang nhua sao cho cac hinh vuong cling 109-i bang cat-tong (cilng nhir cac hlnh vuong cling bang nhua) doi mot khong c6 diem chung. Sau khi xep chung len m?,t ban, ngiroi ta nhan xet th§,yr~g t~p dinh hinh vuong bang nhua hoan toan trung vdi t~p dlnh hinh vuong bang cat-tong.

Hay chirng minh r~ng cac hlnh vuong bang cat-tong d11<;1C d?,t trung hoan toan voi cac hlnh vuong bang nhua,

2 Mang hrdi dinh 0 vuong

M9t h~ thong vo han 0 vuong tao tren mat phang giong nhu ta lat gach mot cai san boi nhimg vi en gach Iat 0 vuong d11<;1C goi la mang hroi dinh 0 vuong. Cac 0 vuong nay dU<;1C

. ( .

goi la cac 0 vuong co sa. Cac dinh 0 vuong chinh la cac diem nguyen diem c6 ca. tung

d9 Ian hoanh d9 la cac s6 nguyen) ciia mot h~ true tea d9 song song voi cac canh cua cac hlnh vuong co sa va c6 don vi g6c 1a d9 dai canh hlnh vuong co sa. M9t da giac c6 dinh la cac dlnh 1110i ciia mang 0 vuong d11<;1C goi la da giac nguyen.

Vdi mang 1110i dinh 0 vuong c6 nhieu bai toan kha thii vi. Sau day la mot tfnh ch§,t co ban cua mang l110i dinh 0 vuong.

Dinh If 4 Da giac deu duy nhat c6 dinh toi au: diim luai 0 vuong 10. hznh vuong.

De thay r~ng ta c6 th~ dung dU<;1C nhirng hlnh vuong c6 dinh tai cac di~m hroi .0 vuong, chang han hlnh vuong co sa cua mang l110i 0 vuong la mot hlnh vuong nhu vay, Bay gio ta clnrng minh r~ng ngoai hlnh vuong, ta khong th~ dung mot da giac d~u nao khac c6 dinh tai cac di~m 1110i cua mang 1110i 0 vuong nao ca.

Ta chirng minh bang phirong phap phan chiing.

Truce h§t ta chtrng minh ding khong th~ dung d11<;1C tam giac d~u c6 dlnh tai cac di~m hroi 0 vuong. That vay, gia Slt dung d11<;1C tam giac d~u canh a c6 dinh tai cac di~m hroi

cua mang hroi 0 vuong. Khi d6 dien tich cua tam giac d~u nay la a2j3 la mot s6 vo ti, do a2 la mot s6 nguyen. MM khac, dien tich cua tam giac c6 ba dinh la di~m l110i ciia mang hrdi 0 vuong 1a mot s6 hiiu ti, vo If. V~y, khong th~ dung d11<;1C tam giac d~u c6

32

dinh tai cac di§m hroi cua mang hroi 0 vuong.

Bay gio ta chtrng minh r~ng khong th§ dung dtroc ngii giac d@u co dinh tai cac diem 1uoi cua hinh vuong. That vay, gia S11 khiing dinh nay khong dung, va t6n tai ngii giac . d@u co dinh t<;Li di§m luoi cua mang 0 vuong, d§ gon ta goi do la ngii giac d@u t6t.

Xet t~p hop:

A = {a2 : a 1a canh cua ngu giac d@u t6t}.

Do A 1a mot t~p con khac rong cua t~p hop cac s6 nguyen, cho nen t6n tai mot phan t11 nho nh§.t a cua A. Xet nam dinh cua ngii giac d@u canh a nay.

Ta dem n6i cac dirong cheo cua ngii giac 1<;Li. De th§.y r~ng moi duong cheo cua mot ngu giac d@u song song vdi canh d6i dien cua no. Do do hai duong cheo tuy y cua ngii giac cimg vdi hai canh d6i dien cua chiing l~p thanh mot hlnh blnh hanh, cho nen giao di~m cua hai dirong cheo cfing la mot di§m cua mang hrdi 6 vuong. D~ chtrng minh dieu nay ta lap mot h~ toa d9 co true song song voi canh cua 0 vuong co sa va co d9 dai bang d9 dai ciia canh hlnh vuong co sa. Khi do cac dinh cua mang hroi 0 vuong co sa chinh lacac di~m co toa d9 nguyen.

Ban dQC de dang chirng minh r~ng giao di~m nay co tea d9 bang t6ng hai toa d9 cua hai dinh cu6i cua hai canh d6i dien trir di tea d9 cua dinh chung cua hai canh nay. Nhir v~y nam giao di~m cua cac dirong cheo l~p nen mot hlnh ngii giac d@u co nam dlnh 1a nam di§m nguyen.

Hlnh 1: N gii giac d@u

De th§.y r~ng ngii giac d@u nay co canh nho hon a, mau thuan vdi gia thiet r~ng trong cac ngii giac d@u co nam dinh 1a cac di§m nguyen, ngu giac d@u co canh nho nhat la a. Mau thuan nay chirng to r~ng khong t6n tai ngii giac d@u nao co ca nam dinh 1a nam di~m nguyen.

Mat khac, ta co th§ thay r~ng khong th§ dung diroc luc giac d@u co dinh tai mang ludi dinh mang ludi 0 vuong, VI n~u co mot luc giac d@u nhu v~y, thl cimg co mot tam giac d~u (diroc tao bdi ba dlnh doi mot khong k@ nhau cua luc giac d@u nay) co dinh 1a dinh hroi cua mang 0 vuong da cho.

Bay gio ta chtrng minh r~ng voi n 2:: 7 thi khong th~ dung diroc n-giac d@u co dinh tai dinh hrdi mang 0 vuong. That vay, gia S11 dung dtroc mot n-giac d@u AIA2 ... An co dinh tai cac dinh cua mang luoi 0 vuong. Khi do ban kinh R cua duong tron ngoai ti~p n-giac Ion hon canh a cua n-giac d@u A1A2··· An. Ta chon mot dinh 0 cua mang hroi 0 vuong

-> ---+

va chon cac dlnh Bl, B2·· . Bn sao cho OBi=AiAi+l (An+l = Ad. VI cac dlnh Ai va 0

la dinh hrdi 0 vuong, cho nen cac di§m Bi ciing la dinh hroi 0 vuong. Ngoai ra,ta th§.y

33

de dang la E1E2••· En la mot n-girio deu, d5ng dangvdi n-giac d~u A1A2••• An theohe s6 ~ < 1. Cir tien hanh nhu vay, sau mot s6 k budc hiru han du ldn, ta thu diroc mot n-giac d~u co canh nho hon canh cua 0 vuong co sa la di~u vo If. Do d6 khong th~ dung . duoc n-giac d~u (voi n 2: 7) c6 dlnh tai dinh hroi cua mang 0 vuong.

Bai t~p

1. TIm ban kinh IOn nh§,t ciia duong tron chi di qua cac dinh cua mang hroi 0 vuong rna khong clit canh 0 vuong nao a di~m trong cua no.

2. Cho trirdc mot s6 dinh ella mang hrdi 0 vuong, clnrng minh r[ng ta c6 th~ to mau cac dinh nay boi hai mau xanhva do sao cho tren moi duong nam ngang va tren moi duong th[ng dirng s6 di~m diroc to do va s6 di@mdu<;Jc to xanh x§,p xi nhau. (D~ thi TOM quoc t~ nam 1986)

3. Chung minh r~ng neu di doc theo cac canh cua 0 vuong co sa tu mot dinh b§,t kl r5i ta tra v~ dinh ban d§.u sau hiru han btroc (c6 do dai bang canh hlnh 0 vuong co sa), thl s6 biroc di cua ta se 1a mot s6 chan.

4. Dien vao moi 0 vuong co sa mot s6 nguyen sao cho moi s6 nay bang dung trung blnh cong cua bon s6 a bon 0 vuong co sa co canh k~ voi n6. Hay chtrng minh r[ng:

a) Tat ca cac s6 diroc dien bang nhau, neu nhu n6 bi chan.

b) C6 mot each dien sao cho cac s6 diroc di~n khong nh§,t thi~t phai bang nhau t§,t ca.

5. Tren ban co vo han ta thuc hien mot tro choi nhu sau:

D§.u tien xep n2 quan vao mot hlnh vuong g5m n x n 0 vuong canh li~n canh sao cho trong moi 0 vuong cua hinh vuong chua mot quan co. Cach di trong tro choi la quan co chi duoc nhay theo mot chieu ngang hoac chieu doc qua mot 0 co chua quan co a ngay sat ben canh sang mot ben mot 0 trong ti~p ngay sau d6. Khi d6 quan co a 0 bi nhay qua se bi 109-i bo.

TIm cac gia tri cua n d~ c6 th~ k~t thuc tro choi sao cho tren ban co chi con 19-i dung mot quan co.

(Ky thi Toan quoc t~ nam 1993)

3 Dinh Ii Picard

.

Mot irng dung kha thiet thuc cua mang hrdi 0 vuong la ling dung vao viec tfnh dien tich cua cac hlnh phang. Ta co th~ thtrc hien viec tfnh dien tich cua mot hlnh ph[ng bang each dem phu n6 bdi mot hrdi 0 vuong du nho r5i tfnh dien tfch hinh da cho. Co sa cho viec tfnh dien tfch da giac nguyen (da giac c6 dlnh 1a cac di~m tea do nguyen) 1a viec xac dinh dien tich cua cac hinh tam giac co dlnh la cac dinh hroi (con goi 18. dinh nguyen) rna khong chua dinh nguyen nao khac ben trong hoac tren canh cua no (tam giac don).

34

Dinh Ii 5 Dier: tich ciia tam quic dan tren. mt;Lng luai 0 vuong dan vi dung bang k.

Truce tien ta th§.y dien tich cua mot tam giac nguyen khong nho hon k VI ta c6 th~ , noi ti~p n6 trong mot hinh chit nhat bdi cac canh di qua dinh n6 va song song voi cac dirong th§.ng cua mang hrdi 0 vuong. Khi d6 dien tich cua tam giac nguyen bang t6ng hoac hieu cua dien tich cua cac hlnh chtr nhat con va cac tam giac vuong c6 C90nh 180 s5 nguyen diroc t900 thanh, nen c6 dang ~,voi k 180 mot s5 nguyen duong, tuc 180 it nh§.t cling phai 180 ~.

M~t khac giii sit hlnh chfi nhat phu tam giac don ABC cho truce theo each tren c6 dQ dai hai C90nh 180 m va n se c6 dien tich bang m x n va chua (m -1) (n -1) diem nguyen ben trong va 2(n + m) di~m nguyen tren bien. Ta chia hlnh chtr nhat nay ra thanh cac tam giac don boi cac doan thang n5i cac di~m nguyen, biit d§.u 180 cac C90nh cua tam giac ABC, va thu diroc nhu dinh If 3 cua muc truce cho ta 180 2(m -l)(n -1) + 2(m+n) - 2 = 2mn tam giac don, Do dien tfch cua moi tam giac don (cling 180 tam giac nguyen) khong nho hen k, cho nen dien tich cua hlnh chtr nhat phu n6 khong nho hen mn. Do a day xiiy ra d§.ng tlnrc, nen dien tfch cua moi tam giac don thanh phan, dac biet 180 dien tfch cua tam giac den ABC, cling bang dung ~.

Dinh If Picard sau day cho ta each tfnh dien tich cua cac da giac nguyen co C90nh khong tv ciit:

Dinh Ii 6 Ctic dinh cua mot da quu: F co canh. khong t'l! edt (khong nh(U thi€t phdi 16i) nli.m a aic di€m nguyen. Ben trong no co n dilm nguyen, con iren. bien m dilm nguyen. Khi do dien. tieh eua no bang

m

SF = n+ 2-1.

Theo dinh If 3 dB. chirng minh a muc truce, ta co th~ chia F thanh 2n + m - 2 tam giac den boi cac doan thang n5i cac di~m nguyen nam trong hoac tren bien ciia n6. Theo dinh If dB. chirng minh a tren, dien tfch cua moi tam giac don chinh b[ng ~, nen dien tich cua F dung bang

m

SF = n+ 2-1.

1. Cac dinh cua mot tam giac nguyen ABC khong chua mot diem nguyen nao tren C90nh ngoai cac dinh cua n6 va chua dung mot di~m nguyen a trong tam giac, Chung minh r1ing di~m nguyen a trong tam giac nay 180 trong tam cua tam giac,

2. Tren mat phang 1§.y nam di~m nguyen phan biet. Chung minh r[ng t6n tai trong chiing hai di~m nguyen sao cho dean th§.ng n5i hai di~m nguyen nay di qua mot di~m nguyen nao d6.

3. Chung minh r[ng trong mot hinh ngii giac 16i nguyen luon c6 It nh§.t mot diem nguyen 180 di~m trong.

35

4. Hay tfnh dien tich nho nhat rna mot ngii giac 16i c6 dinh la cac diem nguyen c6 th~ c6.

5. Tren mot to gi§.y c6 ke mang luoi 0 vuong c6 n 0 bi to den. Chung rninh r~ng c6 . th~ clit tu to gi§.y nay ra mot 86 hiru han cac hlnhvuong thea man d6ng thoi hai dieu kien:

a) Tat ca cac 0 den d~u nam trong cac hlnh vuong nay,

b) TI 1~ dien tich bi to den va dien tich toan phan trong rnoi hlnh vuong nay (j . _ 1 ,4

gnra - va-

5 5'

36

Nguyen ly Dirichlet va mot bai toan ap d ung

'" -

so

Nguyen Duy Thai Son

Nguyen ly Dirichlet (thuat ngir ti~ng Anh: the pigeonhole principle, cling c6 noi goi la the drawer principle) - d dang dan gian nh§,t - diroc phat bi§u d§,u tien bdi G.Lejeune Dirichlet (1805-1859), mot nha toan hoc Dire g6c Phap, nhir sau:

"N~u nhnt n + 1 con tho vao n cai chuong (n E N*) thl luon c6 (it nh§,t la) hai con tho bi nh6t trong cimg mot chuong".

Mot each t6ng quat, ta c6 nguyen ly Dirichlet IDd rong:

"N~u nh6t m con tho vao n cai chuong (m, n E N*) thl luon t6n tai mot chuong chua

, [m - 1] ?

it nhat la 1 + n con tho".

o day, ky hieu [a] diroc dung d§ chi phan nguyen cua s6 thirc a tuc la s6 nguyen IOn nhat khong vuot qua a.

Dung phung phap phan chirng, ta c6 th§ dua ramot each clurng IDinh kha ngan gon cho nguyen ly Dirichlet (ngay ca dirdi dang IDd rong); hoc sinh THPT cling c6 th~ lam dU<;1C viec nay; va di~u d6 khong h~ lam giarn di gia tri cua ban than nguyen IY. Nguyen ly Dirichlet c6 r§,t nhieu iing dung (hieu qua d~n b§,t ngo): su dung n6, ta c6 th§ clnrng minh diroc nhieu k~t qua sau s~c cua toan hoc. Chinh VI v~y) tai cac cuoc thi hoc sinh gioi toan (quoc gia va quoc t~), nguyen ly Dirichlet thirong xuyen diroc khai thac. D~ minh hoa, dirdi day, ta xet mot s6 bai toan cu th~.

'.

Bai 1 (Putnam 1993). Cho mot day 86 gam 1986 nguyen duaru; kh6ng ouat qua 93 va

mot day 86 gam 93 86 nguyen dUdng kh6ng iniat qua 19. Chung minh rllng tit hai day 88 d6 ta c6 thi zan luat tricli ra hai day con c6 t6ng ctu: 88 hq,ng 10, Mng nhau.

Giro. Ta xet bai toan t6ng quat:

Cho hai day (hitu han) cac s6 nguyen dung:

37

Theo nguyen ly Dirichlet, t6n tai hai t~p con khac nhau c6 t6ng nghich dao cac phan tv thuoc vao cling mot mra khoang. Loaibo khoi hai t~p con d6 cac phan tv chung (hai t~p con 7-ph§.n tv khac nhau thl c6 t6i da sau phan tv chung), ta se thu diroc hai t~p . con k-ph§.n ti! (voi k nguyen duong, k :S 7), thoa yeu c§.u cua bai toan: hieu cua hai t6ng nghich dao cac phan h! trong hai t~p con nay se sai khac nhau it han 1/1000.

Bai 3 (Iberaamerica 1998). Otic dt;Li difn cua n qu6c gia (n E N*\ {I}) ng6i quanh m9t ban tron thea each sao cho hai nguai ng6i sat ben pluii hai dai difn Mt ky vai ciniq qu6c ticli thi phrli co qu6c iicli kluic nhau. Hay xiic dinh. (thea n) s6 Zan nhtit co the co cae dt;Li difn ng6i quanh ban trim do.

GUti. Ta se dung X1,X2, ... ,Xn d~ ky hieu n quoc gia c6 dai dien ng6i tai ban tron; cac dai dien c6 quoc tich Xi (N* :3 i:S n) se dU<;1C danh s6 la x;, x~, ...

1/ Tnroc tien, n@u c6 nhieu han n2 dai dien ng6i tai ban tron, thl theo nguyen ly Dirichlet t6n tai i (N* :3 i :::; n) d§ qu6c gia Xi c6 it nhat la n + 1 dai dien, rna ta ky hieu la x;, x~, ... , X~+l; nhung chi c6 n quoc tich, nen van theenguyen ly Dirichlet, trong s6 n + 1 ngiroi ngoi sat ben phai cac dai dien x;, x~, ... , X~+l, phai c6 hai ngiroi c6 cling quoc tich; mau thuan vdi yeu c§.u cua bai toano Mau thuan -d6 chimg to r~ng c6 khong qua n2 dai dien ngoi tai ban tron,

2/ Tiep theo, gia sv mai quoc gia c6 dung n dai dien: cac dai dien cua Xi (N* :3 i :S n) la x;, x~, ... ,xi; ta se chi ra bang quy nap theo n r~ng c6 mot each slip xep toan h9 n2 dai dien d6 vao ban tron sao cho moi yeu c§.u cua bai toan d~u duoc tho man.

Mai each slip x@p toan b9 n2 dai dien vao ban tron thuc ch§,t la mot hoan vi

(1)

cua t~p hop {x{ Ii, j E N*; i, j :S n} vdi quy uoc r~ng trong each slip xep nay:

1. ak+l la dai dien ng6i sat ben phai ak (vdi moi k nguyen dung rna k < n2)

(ta d6ng nhat cac hoan vi c6 th§ thu duoc til nhau qua mot phep hoan vi vong quanh), Khi n = 2 de th§,y each slip x@p (xL x~, x~, x~) thea man moi yeu c§.u cua bai toano Gia sv vdi n E N*\{l} nao d6 da c6 mot each siip xep, rna ta van gift ky hieu (1), thoa man moi yeu c§.u cua bai toano Khi d6, c6 th§ chirng minh r~ng:

Vdi mai i E N n [1, n], t6n tai duy nh§,t j E N n [1; n] sao cho ngiroi ngoi sat ben phai x{ cung c6 quoc tich Xi.

That v~y, neu n nguoi ng6i sat ben phai cac dai dien x;, x;, ... ,xi d~u c6 quoc tich khac Xi (chi c6 n -1 quoc tich nhir the), thl theo nguyen ly Dirichlet, it nh§,t hai ngiroi trong s6 ho phai c6 cling quoc tich; mau thujin vdi yeu cau cua bai toano Mau thuan nay chimg to sir t6n tai cua j = kHan mra, neu N n [1; n] :3 k =I- ji, thi - theo yeu c§.u cua bai toan - nguoi ngoi sat ben phai xf se khong cling quoc tich Xi voi ngirci ngoi sat ben phai X{i. Di~u d6 cho th§,y tinh duy nh§,t cua i. va (2) da diroc chirng minh.

39

lam ella moi hoc sinh c6 th@ diroc d~t tuong irng vdi mot bo x = (Xl; X2; X3; X4; X5) E ]5; trong d6, Xi la s5 tlnr tu cua phuong an rna hoc sinh da chon d@ tra loi cho cau hoi thir

i (i E Z, 1 :::; i :::; 5). Ngoai ra, n~u X E ]5 ta cling se chap nhan each vi~t X = (Xl; X') vdi X' := (X2; X3; X4; X5) E ]4. Bang each nhu vay, de th§.y ]5 dircc phan hoach thanh .- 44 = 256 tap con (rei nhau); moi t?,p con g6m dung. 4 bo chi khac nhau 0 thanh ph§.n tlnr nh§.t, tUc la t?,p con c6 dang

Axl := {(I; x'); (2; x'); (3; x'); (4; x')} c ]5

(1)

voi x' E ]4. VI 2000 > 7 x 256 nen theo nguyen 1:9' Dirichlet c6 tam hoc sinh (khac nhau) AI, A2, ... ,A8, rna bai lam cua ho thl trng vdi cac bi? thuoc cimgmct t?,p con A nao d6 (trong s5 256 t?,p con n6i tren). Nhung 2000 - 8 = 1992 > 7 x 256, nen lai theo nguyen 1:9' Dirichlet - t6n tai tam hoc sinh (khac nhau, trong s5 1992 hoc sinh con lai), la Bi ; B2, ••. ,B8, c6 bai lam Ung vdi cac be) thuoc cling mot t?,p con B nao d6 (trong s5 256 t?,p con n6i tren). Cuoi cling, van c6 1992 - 8 = 1984 > 7 x 256 nen dung nguyen ly Dirichlet mot l§.n nfra, tir 1984 hoc sinh con lai, ta ti~p tuc tim ra tam hoc sinh (khac nhau) , la Gl, G2, ... , C8, rna bai lam cua ho irng vdi cac be) trong cling mot t?,p con C nao d6. 'I'ir b5n hoc sinh b§.t ky trong s5 24 hoc sinh

ta luon chon ra duoc hai hoc sinh c6 bai lam ling vdi cac be) thuoc cling mot t?,p con (mot trong ba t?,p con A, B hoac C); gia S11, chl1ng hl;l.ll,d6 la cac hoc sinh A va Aj (i,j E Z; 1 :::; i < j :::; 8) rna bai lam img vdi hai be) trong A. Theo each xay dung cac t?,p con (1), bai lam cua hai hoc sinh nay chi c6 th@ khac nhau d t5i da la mot cau hoi (chinh la cau hoi tlnr nhat, n~u hai bai lam khong hoan toan giong nhau); yeu c§.u cua bai toan da khong diroc thoa man!

2/ Bay gio, ta chi ra mot tinh huong rna n = 25 thoa yeu c§.u cua bai toano Muon v?,y, xet

5

S:= {x E J512:::Xi = 0 (mod 4)}

i=l

(2)

R6 rang S g6m dung 4 x 4 x 4 x 4 x 1 = 256 be); hon nfra, (2) cho thay: khi x, yES thl

(3)

L§.y V la mot t?,p con 250 phan t11 cua S va xet tlnh huong rna: vdi rrioi X E V t6n tai dung 8 hoc sinh c6 bai lam cling img voi be) X (tmh huong nay hoan toan c6 th@ xay ra VI 2000 = 8 x 250. Khi §.y (do 25 > 8 x 3 theo nguyen 1:9' Dirichlet, cli 25 hoc sinh thl tim diroc b5n hoc sinh (trong s5 25 hoc sinh nay) c6 bai lam irng vdi b5n be) x, y, Z, t EVe S doi mot khac nhau; va theo (3), hai hoc sinh nao trong s5 b5n hoc sinh d6 cling c6 bai lam khac nhau d it nh§.t la hai cau hoi, dung nhu yeu c§.u cua bai toano

Tir 1/ va 2/, ta thay: s5 tu nhien be nh§.t phai tim la n = 25.

41

Nguyen ly Dirichlet va mot bai toan ap d ung

'" -

so

Nguyen Duy Thai Scm

Nguyenly Dirichlet (thu~t ngfr ti~ng Anh: the pigeonhole principle, ciing co noi goi la the drawer principle) - d dang don gian nhat - duoc phat bi~u d§.u tien bdi G.Lejeune Dirichlet (1805-1859), mot nha toan hoc Dire g6c Phap, nhir sau:

"N~u nhot n + 1 con tho vao n cai chuang (n E N*) thi luon co (it nh§.t la) hai con tho bi nhot trong cung mot chuang".

Mot each t6ng quat, ta co nguyen 15' Dirichlet mo rong:

"N~u nh6t m con tho vao n cai chuang (m, n E N*) thi luon tan tai mot chuang chua

it nh§.t la 1 + [m: 1] con tho".

a day, ky hieu [a] diroc dung d~ chi phan nguyen cua s6 thuc a tuc la s6 nguyen Ion nhat khong vuot qua a.

Dung phung phap phan chirng, ta co th~ dua ramot each chirng rninh kha ngdn gon cho nguyen ly Dirichlet (ngay ca duoi dang mo rong); hoc sinh THPT ciing co th~ lam duoc viec nay; va di~u do kh6ng h~ lam giam di gia tri cua ban than nguyen 15'. Nguyen 15' Dirichlet co r§.t nhieu trng dung (hieu qua d~n b§.t ngo): su dung no, ta co th~ chtrng rninh diroc nhieu k~t qua sau s[c cua tOM hoc. Chinh VI v~y, tai cac CU9C thi hoc sinh gioi toan (quoc gia va quoc t~), nguyen ly Dirichlet thircng xuyen duoc khai thac. D~ rninh hoa, dirdi day, ta xet mot s6 bai toan cu th~.

- -

Bai 1 (Putnam 1993). Cho mot day s8 gam 19 s8 nguyen duaru; kh6ng uuot qua 93 va

mot day 88 gam 93 s6 nguyen d'l1dng kh8ng V'l1(Jt qua 19. Chung minh rang tit hai day s6 d6 ta c6 the zan lstat tricli ra hai day con c6 t6ng ctic 88 hr;mg la bang nhau.

GUu. Ta xet bai toan t6ng, quat:

Cho hai day (htru han) cac s6 nguyen dung:

37

"\'h L...j=jl Xj.

Giai bai toan t6ng quat:

D~t ap := 2:f=l Xi (p E Z, 1 ~ p < m) va bq := L:J=l Yj (q E Z, 1 ~ q ~ n). Thay . d6i vai tro cua cac "ky tv" X, a, m, i,p tuong trng voi cac ky tu y, b, n, i, q n~u can, ta c6 th~ xern r~ng am ~ bn. Khi d6, voi moi p E Z n [1, m], t6n tai J(p) := q E Z n [1, njla chi s6 be nh§,t rna ap ~ bq. Xet m hieu:

(1)

Trude h~t, ta c6 nhan xet r~ng moi hieu d~u be han m. Th~t vay, n~u c6 mot chi s6 p E Z n [1, mj nao d6 sao cho m ~ bf(p) - ap, thi (m < bf(p) nen J(p) > 1 va)

m ~ bf(p)-l + Yf(p) - ap =? 0 ~ m - Yf(P) ~ bf(p)-l - ap =? ap ~ bf(p)-b

mau thudn vdi dinh nghia cua J(p).Mau thuan d6 chtmg to r~ng:qua thirc, moi hieu d (1) la be han m.

Bay gic, n~u mot trong cac hieu d6 triet tieu: bf(p) - ap = 0, thl ra rang ta c6 ngay dpcm voi each chon i1 := 1 =: j1, i2 := p, j2 := J(p). Trong truong hop con lai, theo nhan xet tren, tOM be) m hieu d (1) deu thuoc Z n [1, m - 1], mot t~p hop chi c6 m - 1 phan tu, nen thee) nguyen ly Dirichlet, c6 hai hieu bang nhau; tire la, t6n tai r, S E Z n [1, m], r > s, d~

bf(r) - ar = bf(s) - as =? bf(r) - bf(s) = ar - as;

va ta cling c6 dpcm, voi i1 := s + 1, i2 := r, i, := J(s) + 1, j2 := J(r).

Bai 2 (Vo dicli COng hoa Czech 1998). Cho X la mOt tq,p hop gam 14 88 nguyen dUrJng plutn biet. Chung minh ding c6 mot 88 nguyen duatu; k ~ 7 va c6 hai U),p can k-phan tu:

rai nhau cua X sao cho

Giai. Xet Cl4 = 3432 tap con 7-ph§.n tit cua X. T6ng (cac) nghich do ciia cac phan tu

- 1 1 1

trong moi t~p con nay ra rang la khong VU<;1t qua l' + 2" + ... + '7 < 2, 6 nen phai thuoc

vao mot trong s6 2600 mra khoang:

( 0 1] (1 2] (2599 2600]

1000; 1000' 1000; 1000 , ... , 1000; 1000 .

38

Theo nguyen ly Dirichlet, t6n taihai t~p con khac nhau c6 t6ng nghich dao cac phan ttl thuoc vao cling mot mra khoang. Loai bo khoi hai t~p con d6 cac phan ttl chung (hai t~p con 7-ph§.n ttl khac nhau thl c6 t6i da sau phan ttl chung), ta se thu diroc hai t~p . con k-ph§.n ttl (vci k nguyen dirong, k ::; 7), thoa yeu c§.u cua bai toan: hieu cua hai t6ng nghich dao cac phan ttl trong hai t~p con nay se sai khac nhau It han 1/1000.

Bai 3 (Iberoamerica 1998). Cac dQ,i di~n cua n qu8c gia (n E N*\{I}) ng6i quanh mot ban troii theo each sao cho hai nguai ng6i sat ben pluii hai aQ,i di~n Mt ky vai cung qu6c ticli thl, phai co qu8c ticli kluu: nhau. Hay xac dinh (theo n) s6 Zan nhat co the co cdc dQ,i di~n ng6i quanh ban trim do.

Gilli. Ta se dung X1,X2, ... ,Xn d~ ky hieu n quoc gia c6 dai dien ngoi tai ban tron; cac dai dien c6 quoc tich Xi (N* :7 i::; n) se dU<;1C danh s6 la xI, x;, ...

1/ Tnrdc tien, n@u c6 nhieu han n2 dai dien ng6i tai ban tron, thl theo nguyen ly Dirichlet t6n tai i (N* :7 i ::; n) d~ quoc gia Xi c6 It nhat la n+ 1 dai dien, rna ta ky hieu la xI, x;, ... , X~+l; nhung chi c6 n quoc tich, nen van theo nguyen ly Dirichlet, trong s6 n + 1 ngiroi ng6i sat ben phai cac dai dien xi, x;, ... , X~+l, phai c6 hai ngiroi c6 cling quoc tich; mau thuan vdi yeu c§.u cua bai toan.Mau thuan ·d6 clnrng to r~ng c6 khong qua n2 dai dien ngoi tai ban tron.

2/ Tiep theo, gia Stl moi quoc gia c6 dung n dai dien: cac dai dien cua Xi (N* :7 i ::; n) la xI, x;, ... ,xi; ta se' chl ra bang quynap theo n r~ng c6 mot each slip xep toan bi? n2 dai dien d6 vao ban tron sao cho moi yeu c§.u cua bai toan d~u diroc tho man.

Moi each slip xep toan bi? n2 dai dien vao ban tron thuc chat la mot hoan vi

(1)

ciia t~p hop {xii i,j E N*; i,j ::; n} voi quy UdC r~ng trong each slip x@p nay:

1. ak+l la dai dien ngoi sat ben phai ak (voi moi k nguyen dung rna k < n2)

(ta d6ng nh§.t cac hoan vi c6 th~ thu diroc tir nhau qua mot phep hoan vi vong quanh).

Khi n = 2 de th§.y each slip xep (xL x~, x~, x~) thoa man moi yeu c§.u cua bai toano Cia Stl vci n E N*\{l} nao d6 da.co mot each slip xep, rna ta van gift ky hieu (1), thoe, man moi yeu c§.u cua bai toano Khi d6, c6 th@ chirng minh r~ng:

Vdi moi i E N n [1, n], t6n tai duy nh§.t j E N n [1; n] sao cho ngiroi ng6i sat ben phai x{ cling c6 quoc tich Xi.

That vay, n@u n nguoi ng6i sat ben phai cac dai dien xI, x; , ... ,xi d~u c6 quoc tich khac Xi (chi c6 n -1 quoc tich nhu the), thl theo nguyen ly Dirichlet, it nh§.t hai ngiroi trong s6 ho phai c6 cling quoc tich; mau thufin vdi yeu c§.u cua bai toano Mau thudn nay chirng to Sl,t t6n tai cua j = kHan mra, neu N n [1; n] :7 k =1= ji, thl - theo yeu c§.u cua bai toan - nguoi ng6i sat ben phai x7 se khong cling quoc tich Xi vdi ngiroi ngoi sat ben phai X{i. Dieu d6 cho th§.y tfnh duy nhat cua i. va (2) da diroc chirng minh.

39

Do (2), bang each hoan vi yang quanh neu can, trong (1) ta c6 th@ xem al va a2 c6 cung quoc tich; tu d6, al khac quoc tich voi an2: vay, van theo (2), voi moi i E Nn [1; n], tbn tai duy nh§.t k; E N n [1; n2 - 1] sao cho aki va aki+l c6 cung quoc tich Xi.

Bay gio, gia sir moi quoc gia Xi (N* :3 i ::; n) c6 them dai dien thir n + 1 la X?+l ngoai ra, c6 them quoc gia Xn+l voi n + 1 dai dien la x~+l' X~+l' ... , x~t~. Khi d6, tir bi? (1), b[ng each:

) th x ( ). b" be ~ ( I n+l n+l )

+ ay cap ak1, ak1+1 oi Q nam . ak1, Xn+l' Xn+ll Xl ,ak1+1

+) thay c~p (aki' aki+l) bdi bo ban (aki' X~+l' X~+l i aki+l) vdi moi i E N n [2; n] ta se thu diroc mot each sEip x~p thoa man moi yeu c§,u cua bai toan cho truong hop n + 1 qu6c gia. V~y 21 da diroc chimg minh thea nguyen ly quy nap.

Tu II va 2/, ta thay: sa 16n nh§.t c6 th@ c6 cac dai dien ngbi quanh ban tron da cho la n2.

BM 4 (Trich dt chon d{}i tuyin Vi~t Nam d'l,t thi IMO, 1999). Cho A = {al; a2; a3; ... } c N* thod man ditu ki~n 1 ::; ap+l - ap ::; 1999 v6i moi p E N*. Chung minh rang t6n iai ciJ,p chi s6 p, q v6i p < q sao cho apl al.

Giai.

D~t A(I; j) := al + j -1 voi moi j nguyen dirong rna j ::; 1999 va bang quy nap, ta dinh nghia:

1999

Bi := II A(i - 1; k), A(i; j) := B, + A(i - 1; j)

k=l

voi moi i, j nguyen ducng rna i 2: 2, j ::; 1999. Tu each xay dung tren, de dang chirng minh A(m; j) < A(n; j), A(m; j)IA(n; j) voi moi bo ba sa nguyen dirong m, n, j rna j ~ 1999 va m < n. Tir each xay dung tren, cling de th§.y (voi moi i E N* :

A(i; 1), A(i; 2), ... , A(i; 1999)) la 1999 sa nguyen duong lien tiep (kh6ng be hon ad do do, theo gia thiet ella bai toan v~ tap hop A, thl tbn tai ji E Zn [1; 1999] d@ A(i;ji) EA.

B§.y gio, VI jl, j2, ... ,j2000 E Z n [1 : 1999], nen theo nguyen ly Dirichlet, c6 hai s6 nguyen dircng m < n ::; 2000 rna jm = i; =: j; voi cluing, ta tim diroc c~p chi sa p < q sao cho

ap = A(m;jm) = A(m;j)IA(n;j) = A(n;jn) = aq (dpcm).

Bai 5 (Vo didi Trung QU6c 2000). C6 2000 hoc sinh tham gia m{}t cuoc thi triic nghi~ g6m 5 cau hoi; moi cau hoi c6 4 pliuatu; an trd lai, va moi hoc sinh chi dUr;Jc phep chon 1 trong s6 4 phuatu; an tung ung di tra loi cho cau hoi. Tim s6 tu nhien n be nhat sao cho aic hoc sinh c6 thi lam bai thi theo cadi nao d6 mii cu n hoc sinh th'i tun dU(fc b6n h9C sinh (trong s6 n hoc sinh nay) di hai hoc sinh nao trong s6 b6n hoc sinh d6 cung c6 bai lam kluic nhau a it nhat la hai ciiu hoi.

Giai.

II Tr110C tien, ta chtrng minh r[ng neu n ::; 24 thi trong moi tnrong hQP (cua bai lam cac hoc sinh) yeu e§,u cua bai toan d~u khong diroc thoa man. Xet I := {I; 2; 3; 4}. Bai

40

lam cua moi hoc sinh c6 th@ diroc di;i.t tuong ling vdi mot bo x = (Xl; X2; X3; X4; X5) E [5; trong d6, Xi la s6 thir tv cua phuong an rna hoc sinh da chon d@ tra loi cho cau hoi thir i (i E Z, 1 ~ i ~ 5). Ngoai ra, neuz; E [5 ta cling se chap nhan each vi~t X = (Xl; X') . voi X' := (X2; X3; X4; X5) E [4. Bang each nhu vay, de th§.y [5 diroc phan hoach thanh 44 = 256 t?,p con (roi nhau); moi t?,p con g6m dung.4 bo chi khac nhau a thanh phan thir nhat, tuc la t?,p con c6 dang

Ax' := {(I; x'); (2; x'); (3; x'); (4; x')} c 15

(1)

vdi x' E [4. VI 2000 > 7 x 256 nen theo nguyen 1:9' Dirichlet c6 tam hoc sinh (khac nhau) AI, A2"'" As, rna bai lam cua ho thl ling vdi cac b<) thuoc cimgrnot t?,p con A nao d6 (trong s6 256 t?,p con n6i tren). Nhimg 2000 - 8 = 1992 > 7 x 256, nen lai theo nguyen 1:9' Dirichlet - t6n tai tam hoc sinh (khac nhau, trong s6 1992 hoc sinh con lai), la El, B2, ... , Es, c6 bai lam (rug vdi cac bo thuoc cling mot t?,p con l3 nao d6 (trong s6 256 t?,p con n6i tren). Cu6i cling, van c6 1992 - 8 = 1984 > 7 x 256 nen dung nguyen 1:9' Dirichlet mot l§.n mra, til 1984 hoc sinh con lai, ta ti~p tuc tlrn ra tam hoc sinh (khac nhau) , la Gl, G2, ... , Gs, rna bai lam cua ho ling vdi cac bo trong cling mot t?,p con C nao d6. Til b6n hoc sinh b§.t ky trong s6 24 hoc sinh

ta luon chon ra diroc hai hoc sinh c6 bai lam ling voi cac bo thuoc cling mot t~p con (mot trong ba t?,p con A, B hoac C); gia su, ch~ng han, d6 la cac hoc sinh Ai va Aj (i,j E Z; 1 ~ i < j ~ 8) rna bai lam ling vdi hai bo trong A. Theo each xay dung cac tap con (1), bai lam cua hai hoc sinh nay chi c6 th~ khac nhau a t6i da la mot cau hoi (chinh la cau hoi thu nhat, neu hai bai lam khong hoan toan giong nhau); yeu c§.u cua bai toan da khong diroc thoa man!

2/ Bay gio, ta chi ra mot tlnh huang rna n = 25 thea yeu c§.u cua bai toano Muon vay, xet

5

S:= {x E [512:: Xi == 0 (mod4)}

i=l

(2)

Ro rang S g6m dung 4 x 4 x 4 x 4 x 1 = 256 bo; han mra, (2) cho thay: khi x, yES thl

(3)

..

Lay V la mot t~p con 250 phan tu cua S va xet tlnh huong rna: vdi rrioi X E V t6n tai dung 8 hoc sinh c6 bai lam cling ling voi bo X (tlnh huong nay hoan toan c6 th~ xay ra VI 2000 = 8 x 250. Khi §.y (do 25 > 8 x 3 theo nguyen 15' Dirichlet, cli 25 hoc sinh thl tim diroc b6n hoc sinh (trong s6 25 hoc sinh nay) c6 bai lam ling vdi b6n b<) X, y, z, t EVe S doi mot khac nhau; va theo (3), hai hoc sinh nao trong s6 b6n hoc sinh d6 cling c6 bai lam khac nhau a it nh§.t la hai cau hoi, dung nhir yeu c§.u cua bai toano

Til 1/ va 2/, ta thay: s6 tv nhien be nh§.t phai tim la n = 25.

41

BAi 6 (De nghi, Todm. 11, ky thi Olympic 30/4 tuim 2006 chung toi do' dua theo y cua m9t de thi Va dich. Romania 1997). G9i A la tfj,p hop tat ca cdc b9 ba x - (Xl, X2, Xg) mi:

XI, X2, Xg E [0,7] nz. B9 X = (Xl; X2; Xg) E A dur;c 99i la tr9i hem b9 Y = (YI; Y2; yg) E A neu X ¥- Y va Xi ~ Yi vai moi i E {I; 2; 3}; khi 0,6, ta viet X >- y. 12m s6 tu nhien n be . nhat sao cho moi tfj,p con n-pluui tit cua A dtu chua it nhat la hai b9 x, y ma X >- y.

Giiii.

1/ Tnroc het, xet t~p hop B := {x E Alxl + X2 + Xg = 11}. C6 th~ kiem tra true ti~p r~ng B la mot t~p con 48-ph§,n tu ella A (B gam dung: 4 phan tu c6 dang X = (0; X2; 11- X2) vdi 4 ~ X2 ~ 7; 5 ph§,n tu c6 dang X = (1; X2; 10 - X2) voi 3 ~ X2 ~ 7; 6 phan tu c6 dang x = (2; X2; 9 - X2) vci 2 ~ X2 ~ 7; 7 ph§,n tu c6 dang x = (3, X2, 8 - X2) voi 1 ~ X2 ~ 7; 8 phan tu c6 dang x - (4, X2, 7 - X2) vdi 0 ~ X2 ~ 7; 7 phan tu c6 dang x = (5; X2; 6- X2) vdi 0 ~ X2 ~ 6; 6 phan tu c6 dang x = (6; X2; 4 - x;) vdi 0 ~ X2 ~ 5; va 5 phan tu c6 dang x - (7; X2; 4 - X2) voi 0 ~ X2 ~ 4.

R6 rang B khong chua hai bo x, y nao rna x >- y.

2/ Ti~p theo, cho N :1 n ~ 49, va B Ia mot t~p con rz-phan tu b§,t ky cua A. Ta se chirng minh r~ng B chua it nh§,t hai bo x, y rna x >- y.

M uon v~y, xet cac t~p con sau day ella A:

CI:= {x E Alxl = ov X2 = 7},C2:= {x E AI(xl = 1 VX2 ~ 6) V (Xl ~ I/\X2 = 6)}, Cg := {x E AI(xl = 2 V X2 ~ 5) V (Xl .~ 2/\ X2 = 5)},

4 C4:= {x E AI(xl = 3/\ X2 ~ 4) V (Xl ~ 3/\ X2 = 4)},C:= nCi'

i=l

'D := A/C = {x E Alxl ~ 4 V X2 ~ 3},

c., := {x E Cilxg = j} (i,j E Z; 1 ~ i ~ 4; 0 ~ j ~ 7),

'Dp,q:= {x E VI Xl =P,X2 = q} (p,q E Z;4 ~p ~ 7; 0 ~ q ~ 3);

va chi c§,n khao sat hai trirong hop:

(i) N~u B n C la mot t~p hop c6 nhieu hen 32 phan tu, thl do chi c6 32 t~p con c., (i,j E Z; 1 ~ i ~ 4; 0 ~ j ~ 7), tao thanh mot "phan hoach" cua C, nen theo nguyen 1:9' Dirichlet, B phai chua it nh§,t la hai phan tu x va y (x i= y) ella cling mot t~p con Ci,j nao d6 (i,j E Z; 1 ~ i ~ 4; 0 ~ j ~ 7); tu c§,u true cua Ci,; ta th§,y x va y co. th~ so sanh diroc voi nhau theo quan h~ "troi", rna ta c6 th@ gia su la x >- y.

(ii) N~u B n C c6 khong qua 32 phan tu, thl B n 'D chua it nhat n - 32 ~ 17 phan tu; nhung chi c6 16 t~p con 'Dp,q (p, q E Z; 4 ~ p ~ 7; 0 ~ q ~ 3) tao thanh mot "phan hoach" ella 'D; nen van theo nguyen 1:9' Dirichlet, B phai chua it nhat la hai phan tu x va y (x i= y) ella cling mot t~p con 'Dp,q nao d6 (p, q E Z; 4 ~ p ~ 7; 0 ~ q ~ 3); tu c§,u true cua Vp,q ta th§,y x va y c6 th~ so sanh diroc voi nhau theo quan he "troi", rna ta cling c6 th~ gia sl'r la x >- y (dpcm).

'I'ir 1/ va 2/, ta thay: s6 tv nhien be nhat c§,n tim la n = 49.

42



86 phuong phap giai cac bai

?

toan to hop nang cao

Mot

D i;ing H ling Thllng

Trong khoa hoc cling nhu trong dai s6ng chung ta thuong g~p bai toan xac dinh s6 hrong cac d6i tuong co mot tinh ch§.t nao d6. Ta goi d6 Ia bai tOM d~rn.T6 hop 1a mot nganh toan hoc nghien crru cac bai toan rnang c§.u true rai rae trong do c6 bai toan d~rn . Ky nang va ki~n thirc cua toan t6 hQP 1a r§.t c§.n thiet cho nhieu khoa hoc tit kinh t~ Wi sinh vat, tin hoc , h6a hoc va quan tri kinh doanh. Nho cac phirong phap rna t6 hop cung c§.p, chung ta co th~ xac dinh diroc s6 hrong cac phan tu cua mot t~p hQP mot each nhanh ch6ng va chinh xac rna khong c§.n ( va nhieu khi cling khong th~) li~t ke diroc VI s6 d6 r§.t IOn.

Chinrcng trmh ph6 thong (lOp 11) da trang bi cho hoc sinh hai quy t~c d~m co ban(quy t~c cong va quy t~c nhan) , cac khai niem hoan vi , chinh hop, t6 hop, Nho d6 hoc sinh c6 th~ giai diroc cac bai toan t6 hop co ban, tuong d6i don gian, Tuy nhien d6i voi nhung bai toan t6 hop phirc tap, c§.n c6 nhirng cac phuong phap "cao c§.p" hon,s~c ben han. Bai giang nay nham cung c§.p mot s6 phuong phap d~ c6 th~ cong pha duoc cac dang tOM kh6 trong t6 hQP.

" ?

1 Quy tac cong tong quat

Ban chat toan hoc cua quy t~c cQng( phat bi~u cho cong viec vdi nhieu phuong an ) la cong thirc tinh s6 phan tu ella hQP n tap hQP hiru han doi mot khong giao nhau. C1,1 th~ ta c6

Cho n tf):p hop AI, ... , An d6i mot kh6ng giaa nhau. Khi do

n

IAI U A2 U ... Ani = L IAil

i=I

Trong nhieu bai toan t6 hQP, chung ta phai tinh s6 ph§.n tu ella hop n t~p hQP b§.t ky ( khong nhat thiet roi nhau). Khi d6 ta c6 quy t~c cong cho s6 phan tu cua hop cua n t~p hop b§.t ky , thirong diroc goi la cong thirc bao ham va loai trir,

Dinh ly (Cong thirc bao ham va loai trlt)

43

Cho n ~ 2 t~p hop hiru han AI, ... , An . Khi d6 ta c6

n

= L IAil- L IAi n Akl + L IAi n Aj n Akl

i=l 1 i<k n

1 i<j<k n

+ ... + L

Dinh ly nay c6 th~ chimg minh tuong d6i de bang phirong phap quy nap , xin danh cho ban doc.

Vi du 1 Trong t~p S = {I, 2, ... , 280} c6 bao nhieu s6 khong chi a h~t cho 2, 3, 5, 7 ? Gidi

Ta d~m xem trong t~p S c6 bao nhieu s6 chia h~t cho ft nhat mot trong cac s6 2, 3, 5, 7

Kf hieu Al = {k E S: k chi a h~t cho 2}, A2 = {k E S: k chia h~t cho 3} A3 = {k E S : k chi a h~t cho 5}, A4 = {k E S : k chi a h~t cho 7}. Khi d6 Al U A2 U A3 U A4 la t~p cac s6 chia h~t cho it nh§.t mot trong cac s6 2, 3, 5, 7

Ta c6

280 280 280 280

IAll= 2 = 140; IA21 = L 3 J -93; IA31 = 5 = 56; IA41 = 7 = 40

280 280 280

IAI nA21 = L6J = 46;IAlnA31 =LlOJ = 28; IAI nA41 = L14J = 20;

280 280 280

IA2 n A31 = L 15 J = 18; IA2 n A41 = L 21 J = 13; IA3 n A41 = L 35 J = 8

280 280

IAI n A2 n Agi = L 30 J = 9; IAI n A2 n A41 = L 42 J = 6

280 280

IAlnAgnA41= L70J =4;IA2nAgnA41= L105J =2

280

IAI n A2 n Ag n A41 = L210J = 1

8u dung cong thirc bao ham va loai tnl ta tim diroc IAI U A2 U A3 U A41 = 216. Thanh. thu, trong t~p S c6 280 - 216 = 64 s6 khong chia h~t cho 2, 3, 5, 7.

Vi du 2 (Cong thuc ham Euler)

Vdi moi s6 nguyen ducng n > 1, ky hieu ¢(n) la s6 cac s6 nguyen dirong be han n va nguyen t6 vdi n. Ham ¢(n) dircc goi 1a ham Euler. N6 dong mot vai tro quan trong trong nhieu bai toan s6 hoc.

Chung minh r~ng

¢(n) = n IT (1 - ~)

i=l Pt

trong d6 PI, P2, ... , Pm 1a t§.t ca cac irdc nguyen t6 phan biet cua n.

44

Giili

Ky hieu S = {I, 2, ... , n}. Ta d~m xem trong t~p S c6 bao nhieu s5 chia h~t cho it

nh§,t mot trong cac s5 PI, ... ,pm. G9i Ai = {k E S: k chi a h~t cho Pi} (i = 1,2, ,m).

Khi d6 Al U A2 ... U Am la tap cac s5 chia h~t cho it nh§,t mot trong cac s5 Pl, ,Pm.

Ta c6

Do d6 thea dinh ly 1

+ ... + (_l)m+l n

PIP2···Pm

VI ¢( n) chinh bang s5 cac s5 khong chia h~t cho t§,t ca cac S5 PI, ... , Pm nen

+ ... +(_l)m 1 )

PIP2"'Pm

= n fI(l- ±)

i=1 Pt

2 Thi~t k~ cac cong doan t.hich hop

D~ ap dung duoc quy tlic nhan di~u c5t y~u la phai thiet k~ mot mo hlnh g6m viec thirc hien lien ti~p cac cong d01;Ln. Quy tlic nhan phat bi~u: V6i moi each thuc hien d cong d01;Ln trudc thl cong d01;Ln thir k c6 th~ lam theo nk each. Nhu vfJ.,y: s5 each thuc hien d moi cong d01;Ln phai khong phu thuoc vao each nao da diroc thirc hien d cong d01;Ln tnroc do.Thanh thu, muon su dung diroc quy tlic nhan , trong me hlnh cua ta gom viec thuc hien lien ti~p cac cong d01;Ln, s5 each thirc hien d moi cong d01;Ln la phai nhu nhau vdi moi each da diroc thuc hien d cong d01;Ln truce d6.

Vi du 3 C6 4 nguoi A,B, C, D cari chon vao chirc giam d5c, k~ toan tnrong va chu tich HDQT.Gia sl'r viec chon nhan su phai thea man yeu e§.u : ong A khong th~ diroc chon lam giarn doc, chirc chu tich HDQT phai la ong C hoac ong D. Hoi c6 bao nhieu each chon.

45

Giai:

C6 mot lei giai nhu sau: Viec chon ba vi trf giam doc, k@ toan trtrong va chu tich HDQT ti@n hanh theo 3 cong doan:

Cong dean 1: Chon giarn d6c: C6 3 each chon chirc giam d6c ( chon B,C,D).

Cong dean 2: Chon K@ Toan trirong : C6 ba each chon k@ toan truong tu ba ngiroi

con lai .

Cong doan 3: Chon Chii tich HDQT C6 hai each chon ( ong C hoac ong D). Theo quy tilc nhan thl s6 each la 3.3.2=18.

Cach nay khong dung: VI s6 each tlnrc hien cong dean 3 phu thuoc vao k@t qua cua cac cong doan 1,2 tnrdc d6 : N@u d cac cong doan trirdc , ong C va ong D khong diroc chon thl cong doan 3 mdi c6 hai each, Can n@u C hoac D da diroc chon thl d cong doan 3 chi c6 mot each hoac tham chi khong co.

Tuy nhien n~u ta thiet ke· viec chon ba vi trf giam d6c, k@ toan trtrong va chu tich HDQT ti~n hanh theo 3 cong doan khac thl van c6 th~ ap dung quy tilc nhan. Cu th~

Cong doan 1: Chon Chu tich HDQT: Luon c6 hai each chon: C hoac D.

Cong doan 2: Chon Ciam d6c: Ta luon c6 c6 hai eachchon du d k@t qua cua cong doan 1 th@ nao .Sau cong doan 1 con ba ngiroi trong d6 c6 ong A. Bo ong A ra ta con hai ngiroi c6 th~ chon vao chirc Ciam d6c.

Cong doan 3: Chon K@ Toan trirong : luon c6 hai each ( tir hai ngiroi con lai ). V?,y k@t qua la c6 2.2.2=8 each chon. Day la dap s6 dung.

Vi du 4 a) Gia su c6 8 van dong vien b6ng ban tham du mot giai d§,u. Trong vong d§.u cua giai, ban t6 chirc c§.n phan ra 4 cap d§,u. Hoi c6 bao nhieu each ghep thanh 4 cap d§,u?

b) Gia su c6 2n van dong vien b6ng ban tham dtr mot giai d§,u. Trong yang d§.u cua giai, ban t6 chirc c§.n phan ra n cap d§,u. Hoi c6 bao nhieu each ghep thanh n cap d§,u?

c) Tu b) chirng to r~ng vdi moi n E N* ta c6 (n + l)(n + 2)(2n -1)(2n) 'chia het cho 2n .

Gidi

a) Ta thi@t k@ viec thuc hien chon theo cac cong doan sau:

Cong doan 1: Chon 2 ngiroi trong 8 ngiroi lam thanh cap d§,u thu nhat. C6 Gi each chon. Cong dean 2: Chon 2 ngiroi trong 6 nguoi con lai d~ lam thanh cap d§,u tlnr hal. C6 Ct each chon. Cong doan 3: Chon 2 nguoi trong 4 nguoi con lai d~ lam thanh cap d§,u thu ba. C6 Cl each chon. Cong dean 4: C6 Ci = 1 each chon: Hai nguoi con lai se lam thanh cap d§,u thu tmr,

Theo quy tilc nhan c6 GrGt.C1Gi each chon. VI thu tu 4 cap d§,u khong diroc xet d~n nen 86 each ghep thanh 4 c?,p d§,u la

GrGt~~l.G? = 105.

46

b) Ly luan tung tv nhu tren, s6 each ghep thanh n cap d§.u la T = C?n·C?n_2··· Cl·C?

n!

c) De bien d5i

T = (2n)! = (n + l)(n + 2}(2n - 1)(2n)

n!2n 2n

VI T la mot s6 nguyen dung nen cong thtrc nay chung to (n + l)(n + 2)(2n -1)(2n) chia h~t cho 2n .

3 Sa dung phep song anh

C6 n nguoi d~n du mot bu5i n6i chuyen trong mot hOi truong gom 200 gh~. Gia su mai ngiroi chi chiem mot gMng6i va maigh~ chi c6 nhieu nh§.t mot ngirci ng6i. N~u ta duoc thong bao r~ng moi ngiroi d@u c6 cha ng6i thl ta k~t luan diroc n ~ 200 N~u ta bi~t them r~ng khong c6 ghe nao trong thl ta bi~t ngay 1a n = 200. N~u c6 mot, s6 ngiroi phai dirng VI khong c6 gh~ thi ta suy ra n > 200. Nhinr v~y c6 th~ xac dinh hay uac 111Qng s6 phari tu cua mot t~p hQP A nao d6 thong qua mot t~p hQP B rna ta da bi~t s6 phan tu cua B nho mot phep tung irng (anh xa) giiia A voi B .. Day la mot phuong phap duoc su dung r§.t hieu qua d~ giai nhieu bai toan d~m nang cao.

Cho anh xa f : A ~ B ..

• Anh xa f duoc goi 1a mot don anh neu voi hai phan tu b§.t ky aI, a2 E A , n~u al =I- a2 thl f(al) =I- f(a2). Tuc 1af(al) = f(a2) {::} al = a2 .

• Anh xa f duoc goi la mot toan anh n~u vdi moi b E B deu t6n tai a E A d~ f(a) = b.

• Anh xa f duoc goi 1a mdt song anh neu moi b E B, t6n tai va duy nh§.t a E A d~ f (a) = b. N 6i each khac f la song anh khi va chi khi n6 d6ng thai la dan anh va toan anh .

Dinh ly Cho A va B la hai t~p hop htru han,

• N~u c6 mot dan anh f: A ~ B thiiAI ~ IBI

• Neu c6 mot toan anh f : A ~ B thl IAI2 IBI

• N~u c6 mot song anh f: A ~ B thl IAI = IBI

Vi du 5 Cho t~p A = {I, 2, ... , 2n}. MOt t~p con B cua A goi la mot t~p can n~u trong t~p d6 s6 cac s6 chan va s6 cac s6 Ie bang nhau. ( Tap rang la mot t~p can VI s6 cac s6 chan va s6 cac s6 Ie trong t~p rang d@u bang 0). Hoi c6 A c6 chua bao nhieu t~p can? Chiing han vdi n = 2, A = {I, 2, 3, 4} . A c6 6 tap con can la cac t~p sau 0,{1,2},{1,4},{2,3},{3,4},{1,2,3,4}.

47

Gidi: Ky hieu X = {2, 4, ... , 2n} la t~p hop t§.t ca cac s6 chan cua A va Y = {I, 3, ... , 2n - I} la t~p hop t§.t ca cac s6 Ie cua A. GQi C 1a ho t§.t ca cac t~p can cua A va V 1a ho cac t~p con cua A c6 dung n phan tu.

Ta lap mot anh X<;L I tir C vao V nhir sau: Gia su B 1a mot t~p can. Ky hieu Bi, B2 tucng irng 1a t~p cac s6 chan va t~p cac s6 Ie cua B. Khi d6 d~t

I(B) = s, n (Y \ B2)'

DoBlat~pcannenlBll = IB21·Thanhthull(B)1 = IBll+IY\B21 = IBll+IYI-IB21 = IYI = n. V~y I(B) E V,

Tiep theo ta chtrng minh I 1a mot song anh,

+ I 1a dan anh: Gia su I(B) = I(C). Suy ra Bl U (Y \ B2) = C1 n (Y \ C2).

VI s; c, la t~p cac s6 chan ; (Y \ B2), (Y \ C2) la t~p cac s6 Ie nen tu d6 suy ra B1 = C1; (Y \ B2) = (Y \ C2) . Do d6 B1 = Cl; B2 = C2 -+ B - C ..

+ I la mot toan anh: Gia su MEV la mot t~p con cua A c6 n phan tU .. Ky hieu Ml, M2 tuong irng la t~p cac s6 chan va t~p cac s6le cua M. D~t B, = Ml; B2 = Y\ M2 va B = B, U B2 . Ta c6

IBll = IMII; IB21 = IYI-IM21 = n -IM21 = IMI-IM21 = IMll

V~y IBII = IB21 do d6 B la mot t~p can. R6 rang I(B) = Bln(Y\B2) = MIUM2 = M.

VI c6 mot song anh giira ho cac t~p can va ho cac t~p con c6 n phan tu cua A nen theo dinh 1y tren s6 cac t~p can cua A bang s6 cac t~p con c6 n phan tu cua A . V~y A c6 t§.t ca la C2n t~p can.

Vf du B. Cho triroc s6 nguyen duong n va s6 nguyen dirong r thoa.man r < n-r+ 1.

Gia su X = {I, 2, ... , n}. Hoi c6 bao nhieu t~p con A cua X c6 r phan tu rna khong chua hai s6 nguyen lien ti~p. Ch~ng han voi n = 7, r = 3, X = {I, 2" 7} . Cac t~p con A cua X c6 ba phan tU rna khong chua hai s6 nguyen lien ti~p ca thay g6m 10 t~p sau day

{1,3,5},{l,3,6},{I,3, 7},{I,4,6},{l,4,7},{I,5, 7},{2,4,6}{2,4, 7},{2;5,7}{3,5, 7} Gidi: GQi A 1a ho t§.t ca cac t~p con cua X c6 tfnh chat. da neu goi B 1a ho t§.t ca cac t~p con c6 r phan tu cua t~p hop Y = {I, 2" n - (r - I)}. Ta thiet l~p mot anh X<;L I : A -+ B nhmr sau: Gia su A = {aI, a2, ... , ar} E A .Ta c6 th@ gia thiet al < a2 < ... < a; . D~t

b, = ai - (i - 1) = ai - i + 1 i = 1, 2, ... , r

Ta c6 bi+ I - b, = aH I - ai - 1 2 1 ( do ai+1 - ai 2 2). Do d6 bi < b2 < . . . < b; :::;; n _ r + 1 Ta dinh nghia I(A) = B Ta c6 BE B , do vay I la mot anh X<;L tir A vao B. Ta se chimg minh I la song anh.

+ I la dan anh: Tir cong thirc b, = ai - (i - 1) = a; - i + 1 suy ra a, = bi + i-I.

Do d6 neu I(A) = I(A') thl A = A'.

+ f la toan anh: Gia su B = {bl < b2 < '" < b; :::;; n - r + I} E B. Xet t~p A = {aI, ... , ar} a d6 a; = b, + i-I . Ta c6 al < a2 < ... < a; :::;; n- r + 1 + r _ 1 = n , va ai+1 - a, = bi+! - bi + 1 2 2 do d6 A E A, I(A) = B ..

48

Suy ra IAI = IBI = C~-r+1 V~y c6 t§,t caC~_r+1 cac t~p con cua X c6 tinh ch§,t da neu.

Vi du 7. MQt cua hang kern c6 ban ba IOl;Li kern: kern xoai, kern so co la va kern , sua. MQt nh6rn c6 6 .ngiroi VaG an kern va goi 6 c6c kern. Hoi

a) HQ c6 t§,t ca bao nhieu su Iva chon?

b) HQ c6 t§,t ca bao nhieu su Iva chon trong d6 ca ba IOl;Li kern d~u c6 rn~t? GMi :a)

Ta thu liet ke mot vai su Iva chon

+) 2 kern xoai , 1 kern so co la, 3 kern sua. +) 1 kern xoai , 4 kern so co la, 1 kern sua +) 2 kern so cola, 4 kern sua

+) 3 kern xoai , 3 kern sfta

MQt sir Iva chon : "a kern xoai, b kern so co la va c kern sua" dU<;1C ky hieu bdi mot be) ba (a, b, c) trong d6 a, b, c la cac s6 nguyen khong am thoa man dieu kien a + b + c = 6.

Chling han b6n su IVa chon 0 tren diroc ky hieu la cac be) (2,1 ,3) ; (1,4,1); (0,2,4) va (3,0,3). Vdi rnoi be) ba (a, b, c) nhuu vay ta d~t tuong irng voi mot day nhi phan ( day gam cac ch'u s6 ° va 1) theo quy t~c sau: vi~t tir trai sang phai a s6 1 lien ti~p , s6 0, b s6 1 lien tiep, s6 0, r6i c s6 1 lien ti~p.

~o~o~

abc

Nhu v~y rnoi be) ba (a, b, c) dU<;1C tung irng voi mot day nhi phan de) dai 8 ( ttrc la gam 8 ky tv) trong d6 c6 6 ky tv 1 va 2 ky tv 0. Chling han

(2,1,3) -4 11010111

(1,4,1) -4 10111101

(0,2,4) -4 01101111

(3,0,3) -4 11100111

Ro rang phep tirong irng d6 la mot don anh: Ngiroc lai, vdi rnoi day 8 ky tv vci 6 ky tu 1 va 2 ky tv ° khi ta dem tit trai sang phai rna c6 :a s6 1 lien ti~p ,s6 ° , b s6 1 lien· ti~p , s6 ° va c chit s6 1 lien ti~p thl day d6 se irng voi be) (a, b, c) thoa man a + b + c = 6.

Chang han day 10110111 se irng vdi be) (1,2,3) tiic laimg voi su Iva chon : 1 kem xoai.Z kern so cola va 3 kern sua, Day 01011111 ling vdi be) (0,1,5) tuc la ling voi su Iva chon 1 kern so cola va 5 kern sua.

Nhu v~y ta da thi~t lap mot song anh gifra t~p hop cac su IVa chon voi t~p hQP cac day nhi phan de) dai 8 trong d6 c6 6 ky tu 1 va 2 ky tv 0. Do d6 s6 cac su Iva chon bang s6 cac day nhi phan de) dai 8 trong d6 c6 6 ky tv 1 va 2 ky tv 0. M~t khac mot day nhi phan de) dai 8 vdi 6 ky tu 1 va 2 ky tv ° tuong irng vdi each chon 2 vi trf trong 8 vi trf d@ ghi s6 ° ( 6 vi trf con lai ghi s6 1). Thanh thu c6 C~ = 28 day nhi phan dQ dai 8 vdi 6 ky tu 1 va 2 ky tv 0. Do d6 s6 cac sir Iva chon la 28.

b) Mot sir hra chon : " a kern xoai, b kern so co la va c kern sua II dl1<;1C ky hieu boi mot be) ba (a, b, c) trong d6 a, b, cIa cac s6 nguyen duong thoa man di@u kien a + b+ c = 6.

49

Vdi moi bQ (a, b, c) thoa man di@u kien tren ta cho tirong trng voi bQ (x, y, z) vdi x = a-I; y = b - 1; z = c - 1. Khi d6 (x, y, z) 1a cac s6 nguyen khong am thoa man di~u kien x + y + z = a + b + c - 3 = 3. D@ kiem tra r~ng day 1a mQt phep song anh gifra t~p cac bQ ba (a, b, c) trong d6 a, b, c la cac s6 nguyen dirong thoa man di@u kien ' a + b + c = 6. vdi t~p cac bQ ba (x, y, z) 1a cac s6 nguyen khong am thoa man di~u kien x + y + z = a + b + c - 3 = 3. Bang suy Iuan tirong tv nhu cau a) ta tim dU<;1C s6 cac su IVa chon 1a Cl = 10.

Clui y: Ta c6 bai toan t6ng quat sau:

MQt cira hang kern c6 ban m loai kern. MQt nh6m c6 n ngiroi vao an kern va goi n

c6c kern. Hoi

a) HQ c6 tAt cel bao nhieusl,l Iva chon?

b) H9 c6 t§.t eel bao nhieu su Iva chon trong d6 ca m loai kern d@u c6 mat? Dap s6:

)Cm-1

a n+m-l

b)Cm-1

n-l

4 Dung cong cu 86 phirc

Vi du 8 Cho p la mot s6 nguyen t6 va m E N*. Giel su

p-l

E = {(kl' ... , kp-d, 0::::; k; ::::;m - 1 : L ik; == ° (mod p)}

i=l

Xac dinh s6 ph§,n tu cua t~p lEI Gidi

Vai moi r = 0, 1, ... , p - 1 ta dinh nghla t~p E;

p-l

E; = {(kr, ... , kp-1), 0::::; k; ::::; m - 1 : L ik; = r (mod p)}

i=l

m-l

F(x) = IIf':}(L Xik) =

k=l

p-1 'k X i=1 ~ i

o ki m-l

p-l

An = {(kl' ... , kp-1)1 L ik, = n} an = IAnl

i=l

50

Khi do

(1)

n=r (mod p)

n=r (mod p)

GQi w 13. mot nghiem phirc tuy y cua g(x) = xp-1 + ... + X + 1 = o. Ta c6 wi =1= 1 VI ~ j ~ p - 1. That vay neu wj = 1 thl goi h 13. 86 nguyen dirong be nh~t d~ wh = 1 ta c6 hlp, hI) -7 h = P -7 plj. Mau thuan. Khi d6

jm 1

F( ) = rr~-lf( j) = rr~-l w -

w J=l W J=l wj _ 1

Truong h(Jp 1. (m,p) = 1. De th§,y n@u jm = r (mod p) -7 wjm = wr. VI ho {jm},j = 1,2, .. ,p - 1 13. h~ thang du thu gon (modulo p) nen{wjm, i = 1,2, .. ,p - I} = {wi,j = 1,2, .. ,p - I}. Suy fa F(w) = 1. D~t b; = IErl . VI n = r (mod p) -7 wn = wr nen tU (1 ) suy fa

h p-l

1 = F(w) = L:anWn = Lbrwr

(2)

n=O r=O

D~t Q(x) = 2:~:~ b.a:". + bo - 1. Theo (2) vdi moi nghiem w of g(x) ta c6 Q(w) = a . Do do t6n tai hang 86 a Q(x) = ag(x) -7 bp-1 - ... = b1 = bo - 1. Tir do

bo _ 1 = E~:~ br + bo - 1 p

_ L~:~ br - 1 _ 2::=0 an - 1

p P

F(I) - 1 mP-1 - 1

p p

Hence

mP-1 - 1 mP-1 + p - 1

IEol=bo= +1=----

p P

Truong h(Jp 2 plm. Khi do wjm = 1 -7 f(wi) = W~;_-;:l = a -7 F(w) = a -7 0 = F(w) =

",N n ",p-1 b r b b b D -,. x . a 1 2 1 t <.

L..-n=o anw = L..-r=o rW -7 p-1 = ... = 1 =. O. 0 v~y VOl Ifl Ol T = , , , ... , p - a

c6

T6m lai

neu (m,p) = 1 n@uplm

51

Vi du 9 Cho p > 2 la s6 nguyen t6 Ie va s6 nguyen duong n > p. Xac dinh s6 cac tap eon A ella S = {I, 2, ... , n} thea man hai di~u kien sau

i) IAI = p

ii) S(A) == 0 (mod p) trong d6 S(A) la t6ng cac s6 nam trong A.

Gidi Voi moi r= 0, 1,2, ... ,p - 1 goi n; la s6 cac t~p eon A ella S thoa man

i) IAI = p

ii) S(A) = r (mod p)

DM g(x) = 2:f':~ Xi. C6 dinh mot nghiem plnrc b§,t ky w ella f(x). Ta e6 {wi,i = 1,2, ... ,p - I} la p - 1 nghiem phan bi~t ella f(x). (Th~t v~y nhtr trong vf du 8 da ehi ra wi =1= 1, (wi)P = (WP)i = 1. N~u Wi = wj -+ wi-j = 1 -+ pli - j mau thujln )

Do d6

p

xP -1 = (x - l)g(x) = I1(x - Wi)

(3)

i=l

L Wil wi2 ... wip = (-I)P an-p 1 <il<i2<···<ip n

Suy ra

AcS,IAI=p

N~u S(A) = r (mod p) -+ wS(A) = wr. Do d6

p-l

_ '"" r

an-p - - L._; nrW

r=O

Ta e§.n xac dinh he s6 an-p' D~t k = l~J ~ n = kp + r, 0 ~ r ~ p - 1. .VI r[ng voi i = 1,2, .. p ta c6 Wkp+i = Wi, i == 1,2, ... , rand W1p+i = Wi, l = 0,1, ... , k-l, i = 1,2, ... ,p nen ta e6

Q(x) = II~=l(X - Wi) = IIf=l(X - Wi)kII~=l(X - Wi)

= [xkp ~ kX(k-l)p + C~x(k-2)p + ... + (_I)k] [z" + b1xr-1 + ... + brl

VI r[ng lp + i = hp + j, 0 ~ i, j ~ r ~ i = i, l - h nen v~ phai ella diing tlnrc tren e6 (k + 1)(r + 1) don thirc va h~ s6 ella xn-p = x(k-l)p+r is -k . V~y

(4)

Xet da thirc R(x) = 2:~:i nrxr + no - k. From (4) ta e6 R(w) = 0 viroi moi nghiern w ella g(x) V~y t6n tai hang s6 a sao cho R(x) = ag(x) -+ np-l = ... = nl = no - k. Tir

52

d6

k . L:~:~ n; + no _ k

no _ = =0...:::.: _

P

= L:~:~ nr _ k = C~ _ k

p p

CP-k ( . 2 1) k CP-k

Hence n; = ~, r = 1, , ... , p _ ,no = + ~

Thanh thl'r dap s6 cua bai toan la

CP_ k

no=k+ n ,

p

Chu 11 Bai toan s6 6 ciia IMO 1995 la trirong hop rieng cua VI du tren vdi n = 2p.

H~ qua N~u pIa s6 nguyen t6 va n > p thi

c~= k (mod p), d d6 k = l~J

5 Thi~t l~p quan h~ truy h6i

Vi du 9 (Bai toan thap Ha noi) Ttrjrong truyen ding tai mot ngoi thap d Ha noi c6 mot t§.m d~ bang d6ng tren d6 c6 d~t ba chiec CQC bang kim cung.Luc khai thien l~p dia, tren CQC s6 1, PhM t6 Nhir Lai da xep 64 chiec dia bflng vang c6 dirong kinh khac nhau sao cho cac dia c6 ducng klnh Ian hon xep d dirdi, cac dia d phia tren cang d tren cao cang nho d§.n. Cac nha str dU<;1C yeu c§.u chuyen t§.t ca cac chiec dia d CQC s6 1 sang CQC s6 2 voi quy tlic sau:

- Moi l§.n chi diroc chuyen di mot chiecdia,

- Trong qua trinh di chuyen khong diroc d~t dia Ian len tren dia nho ( do d6 c§.n

thiet phi c6 them chiec CQC trung gian thii ba). Gia sl'r moi l§.n chuyen mot chiec dia m§.t mot giay, Hoi cac nha sir c§.n It nhat la bao nhieu nam d@ chuyen t§.t ca cac chiec dia a

CQC s6 1 sang CQC s6 27 .

Girli

Gia sl'r hie d§.u tren CQC s6 1 c6 n chiec dia. GQi Un la s6 l§.n It nhat d@ chuyen t§.t ca'

cac chiec dia d CQC s6 1 sang CQC s6 2. Ta thl'r tinh mot vai gia tri cua Un.

Voi n = 2. Ta c§.n thuc hien ba phep chuyen sau -Chuyen dia be sang CQC s6 3.

- .Chuyen dia IOn sang CQC s6 2.

- Chuyen dia be v@ CQC s6 2

V~y U2 = 3

VOi n = 3. Ta can thuc hien theo ba giai dean sau - Chuyen hai dia d phfa tren sang CQC s6 3. Nhir da thay d trirong hop n = 2 , ta c§.n 3 phep chuyen.

53

- Chuyen dia 16n nhat sang CQC s6 2

- Chuyen hai dia d CQC s6 3 v~ CQC s6 2. Nhir da th§.y d tnrong hop n = 2 ta c§.n 3

phep chuyen.

Vf),y ta c§.n 3+ 1 +3=7 phep chuyen. Do d6 U3 = 7;

Truong hop n = 3 goi Y cho ta thiet If),p h~ thirc truy h6i rna day (un) phai thoa man.

D~ chuyen diroc n chiec dia theo quy tiic tren ta phai thuc hien theo ba cong doan sau:

- Cong doan 1: Chuyen (n - 1) dia d phi a tren chiec dia Ian nh§.t sang CQC s6 3 thea quy t[c tren, Ta c§.n Un-l phep chuyen. Chiec dia IOn nh§.t van gill' nguyen d CQC s6 1 khi di chuyen (n - 1) chiec dia d tren n6.

- Cong doan 2:Chuy~n dia IOn nh§.t sang CQC s6 2.

- Cong dean 3: Chuyen (n - 1) dia d CQC s6 3 v~ CQC s6 2 va d~t len tren chiec dia

IOn nh§.t Ta c§.n Un-l phep chuyen. Do vf),yd~ chuyen n chiec dia tu CQC s6 1 sang CQC s6 2 ta c§.n Un-l + 1 + Un-l = 2Un-l + 1. Vf),y ta c6 h~ thirc truy h6i sau

Un = 2Un-l + 1.

Tir he thirc truy h6i nay ta c6 th~ If),p diroc cong thirc cua s6 hang tiSng quat cua day. Bang quy nap de clnrng minh diroc

Vdi n = 64 thi U64 = 18.446.744.073.709.531615

D6 la s6 l§.n chuyen dia rna cac nha sir phai thirc hien d~ hoan thanh cong viec. N~u moi l§.n chuyen mot dia m§.t 1 giay thl c§.n d~n hem 500 ty nam cac nha sir moi chuyen diroc t§.t ca 64 chiec dia sang CQC s6 2.

VI du 10 (IMO 1979, bai s6 6)

Gia S11 A va E la hai dlnh d6i dien cua mot bat giac d~u. Mot con ~ch bkit d§.u nhay tu dinh A. Tai moi dinh cua bat giac (tru dinh E) , moi cu nh~y con ~ch chl c6 th~ nhay tdi hai dinh k~ voi dinh d6. Khi con ~ch nhay vao dinh E n6 se bi ket vinh vi en d d6. Cho trircc s6 nguyen duong n . Hoi vdi n cu nh~y, c6 bao nhieu each d~ con ~ch nhay vao dinh E.

Gidi . GQi an la s6 each d~ can ~ch nhay vao dinh E. De th§.y al = a2 = a3 = 0; a4 = 2.

Gia S11 tu A theo chieu kim d6ng h6 cac dinh l§.n hrct la A ~ B ~ C ~ D ~ E ~. F ~ G ~ H ~ A. Tir A con ~ch d~n B phai qua mot so Ie btroc.Tir B con ~ch d~n C phai qua mot so Ie biroc.Tir C con ~chd~n D phai qua mot sole birdc.Tir D con ~ch d~n E phai qua mot so Ie birdc. Vf),y s6 budc d~n E dut khoat phai la mot s6 chan. N6i each khac n~u n Ie thl khong c6 each nao nhay vao E. Vf),y a2k-l = O. Ta c§.n tfnh a2k, k 2:: 1

Xu§.t phat tu A, vdi hai biroc nh~y d§.u tien con ~ch c6 th~ c6 cac each sau :

1) A ~ B -v+ A

2) A ~ H ~ A

2) A ~ B ~ C

3) A ~ H ---+ G

54

N~u thea each 1) thi s6 cachtdi E 1a a2k-2.N~u thea each 2) thl s6 each t6i E la a2k-2. G9i en, gn l§.n hrot la s6 each d~ con ~ch, xu§,t phat tmrong irng tir C, G, nhay vao dlnh E Val n eli nhay, Vt ly do d6i xirng ta co en = gn' V~y n~u theo each 3) thl s6 each Wi E la C2k-2; neu thea ~aeh 4) thi soeach toi E la g2k-2 . Theo quy t[e cong ta co

a2k = a2k-2 + a2k-2 + C2k-2 + g2k-2 = 2a2k-2 + 2C2k-2

Xuat phat tu C,v6i hai buoc nh§.y d§.u tien con ~eh co th~ co cac each sau : 1e) C --+ B --+ A

2c) C --+ B --+ C

3e) C --+ D --+ C

4e) C --+ D --+ E

N~u thea each 1c) thi s6 each tdi E la a2k-2.N~u thea each 2c) thl s6 each t6i E la C2k-2.N~u theo each 3c) thi s6 each tdi E la C2k-2.N~u theo each 4e) thl s6 each t6i E la O. Theo quy t~e cong ta co

C2k = a2k-2 + 2C2k-2 (6)

Tir (5) va (6) rut ra C2k = a2k - a2k-2 --+ C2k-2 = a2k-2 - a2k-4. Thay vao (5) ta duoc

(5)

a2k = 4a2k-2 - 2a2k-4

D~t Uk = a2k ta co Uk '= 4Uk-l - 2Uk-2, Ul = a2 = 0; U2 = a4 = 2. Bang each giai phirong trlnh d~e tnrng ta di d~n cong tlnrc sau

. (2 + yi2)k-l - (2 - V2)k.,-1

a2k = Uk = . y'2 ,k = 1,2, ....

Vi du 11 Cho s6 nguyen duong n va S = {1, 2, ... , n}. TIm s6 cac tap con (k~ ea t?,p rang) cua S rna khong chua hai s6 nguyen duong lien ti~p.

Girli G9i an la s6 phai tim. De th§,y al = 2, a2 = 3, a3 = 5. Ch~ng han voi n = 3 e6 5 t?,p con thea la 0; {1}; {2}; {3}; {1; 3}. G9i An la ho cac t?,p con co tfnh ch§,t da neu. Mai t~p A E An+2 gom hai 10l;Li : LOl;Li 1 gom cac t~p chua n + 2.Lol;Li 2 gom cac t?,p khong chua n + 2. N~u A la tap 10l;Li 1 thl A khong chua n + 1. Do do n~u bo di khoi A phan tv n +2 ta duoc mot tap concua An. Nguoc lai voi rnoi t~p con B cua An thl t?,p A = B U {n + 2} 1a tap 10l;Li 1 cua An+2. Thanh thv s6 t~p 10l;Li 1 la an. Moi t~p 10l;Li 2 ro rang la mot tap con cua An+l va nguoc lai. Thanh thir s6 t~p 10l;Li 2 1a an+1' Do d6 ta co quan he sau

an+2 = an+l + an

Mat khac voi day Fibonaey ta co Fn+2 = Fn+1 + Fn. VI al = F3 = 2; a2 = F4 = 3; a3 = F5 = 5 ta suy ra an = Fn+2. V~y

d do a - 1+V5 a = I-v'5

- 2' 2

Vi du 12 Cho s6 nguyen duong n va S = {1, 2, ... , n}. G9i en la s6 cac t?,p con cua S rna chua dung hai s6 nguyen ducng lien tiep . Chung minh r~ng

2nFn+1 - (n + 1)Fn

en= --------~--~-

5

55

Gidi G9i Cn la ho cac t~p con co tfnh ch§.t da neu, Moi t~p C E Cn+2 g6m ba loai :

Loai 1 g6m cac t~p chua n + 2, n + 1. Loai 2 gorn cac tap khong chua n + 2 . Loai 3 1a cac t~p chua n + 2 nhirng khong chua n + 1.

+) Neu C la t~p loai 1 thl C cling khong chua n ( vi neu C chua n thi C chua 2 cap 86 nguyen lien ti~p 1a n, n+ 1 va n+ 1, n+2) . B6 di khoi C phdri tll n, n+ 1, n+2 ta diroc mot t~p con cua {I, 2, ... , n - I} khong chua hai 86 nguyen duongIisn ti~p, do do no la phan tll cua An-1.NguQc lai vdi moi t~p con A cua An-1 thl t~p C = Au {n + 2, n + I} 1a t~p loai 1 cua Cn+2• Phep ttrong tmg nay 1a song anh, Thanh thll s6 t~p loai 1 1a an-l = Fn+1.

+) Moi t~p loai 2 ro rang la mot t~p con cua Cn+! va ngiroc lai. Thanh thll s6 t~p loai 2 1a Cn+l.

+) N~u C Ia t~p loai 3 thl C khong chua n + 1. Do do neu bo.di khoi C phan tll n + 2 ta diroc mot t~p con cua Cn. N giroc lai voi moi t~p con B cua Cn thl t~p C = B U { n + 2} 1a t~p loai 3 cua Cn+2. Thanh thir ta co h~ thtrc sau

Tli do bang quy nap, su dung he thirc truy h6i cua day Fibonaci ta co cong thirc neu tren.

56

Xay d trng song anh giai mot - bai toan t6 hop

"" -

so

Huynh T§,n Chau (THPT Chuyen Luang Van Cluuih, n« Yen)

Trd ngai 16n nh§.t khi giai mot bai toan t6 hQP 1a xac dinh huang di. R6 rang d~ c6 kha nang dinh huang t6t thl viec ren luyen cac phuong phap ti~p can 1a r§.t c§.n thi~t. Bai vi~t naygioi thieu vdi cac ban doc mot phuong phap hieu qua trong nhieu bai toan t6 hop rna ta tarn goi la phuong piuip song tinh.

Phuong phap nay dua tren ket qua hi~n nhien sau: "Neu c6 m(jt song anh di tit mot t{j,p hilu har; X t6i mot t{j,p hiiu hom. Y th2 hsc lU(Jng [tnic s6 ph an tit) eua X va Y bang nhau",

1. MQt each tu nhien, k~t qua tren huang chung ta d~n y tucng S11 dung song anh d~ so sanh luc luong hai t~p hQP.

Bai toan 1 (Vo dich. Lien Xo} C6 mot nh6m nguai rna trong d6, moi e(i,p khong quen nhau c6 dung hai nguai quen chung, con moi e(i,p khong quen nhau thZ khong c6 nguai quen chung. Chung minh ding s6 nguai quen ciui moi nguai la nhu nhau.

Loi giai. Gia S11 a quen b va t~p cac nguoi quen cua a va b (khong k@ a, b) la A va B. Moi ngiroi a' thuoc A se quen vdi duy nhat mot nguoi thuoc B (do a' va b khong quen nhau, hon mra ho da co mot ngiroi quen chung la a). Tuong tu, rnoi ngiroi thuoc B cling quen voi duy nhat mot ngiroi thuoc A. V~y t6n tai mot song anh di til A tai B, Wc a va b c6 s6 ngiroi quen bang nhau.

N~u a khong quen b thl t6n tai e quen ca a va b, do d6 s6 ngiroi quen cua a va b bang . nhau do cling bang s6 ngirci quen ciia c.

Bai toan 2 (Trung Qu6e - 1997) Trong cae xiiu nhi phiin c6 d(j dai n, gQi an la s8 cdc xiiu khong chua qua 3 s6 lien tiep 0,1, ° va bn la s6 cdc xiiu khong chua 4 s6 lien tiep 0,0,1,1 hoiic 1,1,0,0. Chung minh rang: bn+1 - 2an.

Loi giai, Ta goi mot xau thuoc 101;Li A neu n6 khong chua 3 s6 lien ti~p 0,1, ° va goi mot xau thuoc 101;Li B neu n6 khong chua 4 s6 hang lien ti~p 0,0,1, 1 hoac I, 1,0, O.

Voi moi xau X = (Xl, X2, ... , Xn), ta xay dung f(X) = (Yl, Y2, ... ,Yn+l) nhu sau:

YI = 0, Yk = Xl + X2 + ... + Xk-l (mod 2). R6 rang X chua 3 s6 lien ti~p 0,1, ° khi va chi khi f(X) chua 4 s6 hang lien ti~p 0,0, I, 1 hoac 1, 1,0,0, tuc la X thuoc 10!;1.i A khi va chi khi f(X) thuoc B.

57

V~y f la mot song anh di tu t~p cac xau 10l;1.i A dO dai n d@n t~p cac xau 10l;1.i B dO dai n + 1 rna b1it d§,u bang O. Nhung tir moi xau X thuoc B ta nhan diroc mot xau X cling thuoc B bang each d5i cac phan tu cua X theo quy t1ic 1 ~ 0, 0 ~ 1 nen 86 cac xau 10l;1.i B c6 dO dai n + 1 g§.p doi 86 cac xau 10l;1.i B dO dai n + 1 rna b1it d§,u bang 86 o. Tir d6 ta c6 dpcm.

2. T'ir viec 80 sanh luc luong cac t~p hop, phuong phap song anh c6 th§ giiip chung tit d@m 86 phan tu cua mot t~p thong qua sir 80 sanhhrc hrong cua t~p d6 vdi mot t~p khac rna ta da biet 86 phan tu cua n6.

Bai toan 3 (V6 dich. Ucraina - 1996) Goi M to. ou: 86 nguyen ·duang vitt trong h~ thq,p ph an c6 2n chit 86, trong d6 c6 n chit 86 1 va n chit 86 2. Goi N la 86 tat ca ccc 86 vitt trong h~ thq,p phtm c6 n chi] 86, trong d6 chi c6 chit 86 1,2,3,4 va 86 chit 86 1 bang 86 chi] 86 2. ChUng minh rang M = N.

LCti gild 1. V6i moi 86 c6 n chit 86 g6m cac chit 86 1,2,3,4 va 86 chit 86 1 bang 86 chit 86 2, ta "nhan doi" thanh 86 c6 2n chit 86 theo quy t1ic sau: d§,u tien, hai phien ban cua 86 nay duoc vi~t k~ nhau thanh 86 c6 hai chit 86, sau d6 cac chit 86 3 (j n chit 86 d§,u va cac chit 86 4 (j n chit 86 sau diroc d5i thanh chit 86 1, cac chit 86 3 (j n chit 86 sau va cac chit 86 4 (j n chit 80 d§,u diroc d5i thanh chit 86 2. Vi du: 1234142 ~ 12341421234142 ~ 12121221221112.

Nhir the, ta thu diroc mot 86 c6 dung n chit 86 1 va n chit 86 2. R6 rang day 1a mot don anh. D§ cluing minh day la mot song anh, ta xay dung anh Xl;1. ngtroc nhu sau: voi moi 86 c6 n chit 86 1 va n chit 86 2, ta c1it n chit 86 d§,u va n chit 86 cuoi r6i cong chung theo cot vdi quy t1ic: 1 + 1 = 1, 2 + 2 = 2, 1 + 2 = 3, 2 + 1 = 4, va ta thu duoc mot 86 c6 n chit 86 g6m cac chit 86 1,2,3,4 vdi 86 chit 86 1 bang 86 cac 86 2. Vi du 12121221221112 ~ 1234142 nhu sau:

1212122 1221112 1234142

Nhu th~ song anh giUa hai t~p hop da duoc thi~t 1~p va ta c6 M = N. Cach xay dung song anh nhu tren kha dep song hoi c§,u kyo Chung toi da tim ra mot loi giai ngdn gon hen nhir sau:

LCti gild 2. Vk E S = {O, 1, ... , n}, ta thirc hien 2 1§,n (doc 1~p voi nhau) viec danh d§.u k trong n vi tri tren mot hang. Sau d6, (j moi vi trf ta ghi: chit 86 3 n~u diroc danh d§.u 3 1§,n, chit 86 4 n~u khong duoc danh d§.u 1§,n nao, chit 86 1 neu chi duoc danh d§.u 1§,n

d§,u va chit 86 2 neu chi duoc danh d§.u 1§,n sau, .

R6 rang khi cho k chay tren S thl 86 t§.t ca cac 86 thu duoc chinh la N. ROn nita, Vk E S thl c6 th§ xem 86 each danh d§.u 1§,n d§,u la 86 each chon k trong 86 n vi tri, con 86 each danh dau 1§,n sau la 86 each chon n - k vi trf trong n vi tri, Do d6 t5ng 86 each danh d§.u khi k chay tren S dung bang 86 each chon n ph§,n tu tir 2n phan tu (tuc 1a C2n), va 86 nay chinh la M. V~y ta c6 dieu phai chirng minh.

58

Bai toan 4 Cho cac sB nguyen dUring n, k uai n ~ k. Xet phep tocn f dBi vai b(J slip thu tu X = (Xl,"" xn) niiu sau: moi lan chon. k sB lien tilp tuy y trong X va d6i dau cua chung. Tim sB cac b(J thu t'l,t X = (Xl, ... ,Xn) thoa man cac dieu ki~n:

i) M9i phan tit cua X dJu thu(Jc tfj,p {a, I}.

ii) C6 thi thuc hi~n huuhQ,n zan phep iotui f di tit X nhfj,n duac b(J (1,2, ... ,1).

Lai gild. Xet bo thu tv X = (Xl,"" xn) tuy y. Ta c6 2 nhan xet sau:

1) C6 dung n - k + 1 nh6m k 86 lien ti~p.

2) Sau mot s6 chan l§,n thuc hien phep toan f cho mot nh6m k s6 lien ti@p trong X thl gia tri k s6 d6 khong d6i.

NhH v~y, moi phirong an thirc hien hiru han l§,n phep toan f cho X tuong ling voi mot b9 nhi phan A = (aI, a2, ... ,an), trong do a; tinh theo modun 2 cua s6 l§,n thirc hien f cho nh6m k 86 lien ti~p (Xl, Xi+l, ... ,Xi+k-l), va X tro thanh

((-1)a1XI, ... , (-1)a1+'+akXk, (_1)a2+···+ak+1Xk+l, ... , (_1)an-k+lXn).

3. Cling til viec so sanh luc hrcng: cac t~p hop, phucng phap song anh 1a mot cong cu diic lire d~ thi@t l~p va chung minh cac cong thirc t6 hop. Thong thirong, ngiroi ta xay dung mot song anhdi til mot t~p vao chinh n6, va nguyen tiic d day c6 th~ phat bi~u nhu sau: "Khi d@m s6 phan tu mot t~p hop bang nhieu each thl cac k@t qua thu diroc bang nhau", Chang han til bai toan 3, n@u ta tinh N theo each "truyen thong": c6 C~C~_l each chon vi trf cho i chir 86 1 va i chfi s6 2,con lai 2n-2i each chon vi tri cho

, [nJ ?

cac chir so 3 va 4, voi i chay til 0 Wi "2 thl ta da chirng minh duoc mot dang thirc kha

thu vi:

i=O

Bai toan 5 Chung minh rang uoi n E N th2

n (0)2 ( 1)2 ( k)2

C2n = Cn + Cn + ... + Cn .

Lai giai, Ta dem 86 each chon n phan tu til mot t~p gam 2n phan tu theo hai each.' Cach thu nh§,t: moi l§,n chon ra n phan tu, khi d6 86 each hien nhien Ia C2:n. Cach thir hai: tnroc h@t ta ·chia t~p 2n phan tU thanh hai t~p con, moi tap gam n phan tUj sau d6 chon til t~p con tlnr nhat k phan tu (co C~ each chon) va chon til tap con thu hai n - k phan tU (co C;:-k = C~ each chon) , ta 8e co (C~) 2 each chon; cu6i cling cho k chay til o tdi n ta dtroc t6ng 85 each chon c§,n tim 1a (c~) 2 + (c~) 2 + ... + (C~) 2. Til d6 ta c6 dpcm.

Bai toan 6 Chung minh rang uai m, n E Z+, m > k th'i:

c: c: CO Ck-l Cl CO c:

rn+rr+I = m n + m-l n+l + ... + m-k n+k'

59

· "_"'~"-"~·'~"'~·'·'~'"",,--_'._"',"'-;O\c.""'·a""-'~,· ... "':."::;;;;r;;:~.;.':~~~~t:}~~~::t';J" ... :;;.~~~·;:;r:-;:''::!:';'.i$';(::::f:-m~:'~!.:~~1jO<",,~~;m~;;wi!P*§'MS.~

'};:

)~

Loi giai. Ta d@m 86 cac bo 86 nguyen T = (aI, a2, ... , am+n+l-k) VGi 1 ::;;; al < a2 < ... < am+n+l-k ::;;; m + n + 1 bang hai each, Cach thir nhat: ta xem d6 la 86 each chon m + ti + 1 - k 86 tir m + n + 1 86 nen 8e c6 C~+n+l each chon, Cach thu hai: . ta th§,y 86 cac bo T c6 am+l-k = m + 1 - i la C~-=-ii.C~+i do c6 ex: each chon cac

b<) T, = (al, , am-k) thoa 1 ::;;; al < ... < am-k ::;;; m - iva C::+l each chon cac bo

T2 = (am+2-k, , am+n+l-k) thoe, man m+2-i ::;;; am+2-k < ... < am+n+l-k ::;;; m+n+1,

tu d6 cho i chay tu 0 Wi k (do m + 1- k ::;;; am+l-k ::;;; m + 1) ta diroc t6ng 86 each chon 18. C~C~ + C~-=-\ C~+l + ... + C~-kC~+k. K@t qua d6 cho ta dpcm.

Bai toan 7 (Va dicli Trung QU6c - 1994) Chung minh ding

n

~ 2kCkC[(n-k)/2j = en 'tin E Z+

~ n n-k 2n+l, .

k=O

Loi giai. Ta chon ra n 86 tu 2n + 1 86 nhu sau. TrUGc h@t, tu 2n + 1 s6, ta chia ra n cap va s6 x. Sau d6 bUGC 1: ta chon ra k c~p roi tir moi cap chon ra mot s6, bUGC 2: chon [en - k)/2] cap trong n - k cap con lai, ngoai ra, s6 x se diroc chon n@u n - k Ie va khong diroc chon neu ri - k chan. R6 rang bUGC 1 c6 2kC~ each chon va biroc 2 c6 Cl~;;k)/2j each chon va ta chon diroc t6ng cong n s6, trong d6 k chay tir 0 tdi n. Lap 1u~n d6 cho ta dpcm.

Bai toan 8 Cho truac cdc 86 nguyen dU(Jng m, n. Tinh

Loi giai, Ta chirng minh t6ng c§.n tinh bang 1, tilc 1a

m n

~ c: 2m-k + ~ c: 2n-k = 2m+n+l

~ n+k ~ m+k .

k=O k=O

Cac luy thira cua 2 goi ta lien tuong d@n s6 d@n s6 t~p con mot t~p hop.

Trong cac t~p con cua t~p S = {I, 2, ... , m + n + I}, de th§,y c6 C~+k2m-k t~p dang {aI, a2,· .. , an+l}, (1 < i ::;;; m + 1) trong d6 al < a2 < ... < an+l va an+l = n + k + 1

VGi 0 ~ k ::;;; m (do c6 C::+k each chon n phan tu tu t~p {I, 2, , n + k}, ngoai ra c6 th@

"b6 sung" them mot trong s6 2m-k t~p con cua t~p {n + k + 1, , n + m + I}. Nhtr vay

2:;;=0 C~+k2m-k 1a s6 t~p con cua S c6 nhieu hon n phan hr.

Tuong tv 2:~=0 C~+k2n-k la s6 t~p con cua S c6 nhieu hon m phan tu, cung tuc la s6 t~p con cua S c6 khong qua n ph§.n tu.

V~y 2:~0 C~+k 2m-k + 2:~=0 C~+k2n-k la s6 t§,t ca cac t~p con cua S, tuc 1a 2m+n+l .

D6 chinh 1a dieu phai chimg minh.

4. M<)t irng dung 1111'a cua phuong phap song anh 1a dung d@ tinh t6ng cac phan tu mot t~p hop nao d6. C6 th@ xem nhu y ttrong nay da duoc d~ xu§.t ngay trong ,bai toan

60

quen thuoc: "Tinh 1 + 2 + ... + 100", vdi each giai tuyet voi rna tircng truyen la cua Gauxo, d day chung ta c6 th§ dien dat 19>i each tfnh d6 nhu sau: Vi E S - {I, 2, ... , IOO}, goi f la anh X9> xac dinh nhu sau: f(i) = 101 - i. R6 rang f la mot song anh tu S tdi . S, do do:

2 L i = L(i + f(i)) = IAI·101 = 10100

iES iES

Suy ra t6ng c§,n tinh bang 5050.

Bai toan 9 (VMO - 2002) Cho t4p S g8m tilt c oic 88 nguyen trong doen [1,2002]. G9i T to, t4p hop tat co' ole t4p can kh6ng r8ng cua S. V6i m6i X thu9C T, ky hi~ m(X) la trung binh. c9n9 cac phan tiJ: cua X. Tinh

m=

2::m(X) ITI

"

(t6ng lay thea tat c cac t4p X thu9C T).

Loi giai. Xay dung song anh f : T -+ T nhu sau: f(X) = {2003 - x/x EX}, VX E T. R6 rang m(X) + m(J(X)) = 2003. Do do:

2 L m(X) = L(m(X) + m(J(X))) = ITI.2003 =? m =

2::m(X) 2003

ITI --2-'

Bai toan 10 Hay tinh trung binli C9n9 tat co, cac 88 N gdm ~002 chft 88 thao' man N ".99 va cac chii 88 cua N thu9c {I, 2, 3, 4, 5, 6, 7, 8}.

Loi giai. GQi M la t~p cac s6 N tho di~u ki~n d~ bai. Xay dung anh X9> f nhtr sau:

N@u N = ala2 ... a2002 thl f(N) = b1b2 •.. b2002, voi b; = 9 - ai. Do N + f(N) = ~:99 nen f la song anh M -+ M.

2002cs9

Til d6:

2 L N= L(N+f(N)) = IMI~.

NEM NEA 2002cs9

Suy ra trung binh cong cac s6 N la:

102002 - 1

~= 2

2002cs9

Cac ban thay d§.y, loi giai bang phuong phap song anh thuong r§.t §.n tuong ve su ng~n gon va dep de. Song d§ co dU<;1C nhirng loi giai d6, chung ta phai khong ngirng tim toi, suy nghi dang c6 mot tu duy nhay ben.

Cuoi cling la mot s6 bai t~p d§ ren luyen.

61

Bai t~p 1 Cho m, n E Z+. Chung minh r1ing s6 each. biiu dien n duai dq,ng t6ng ciu: nhitng 86 nguyen dUdng khong lan han m bang s6 oicli biiu dim n duai dq,ng khong it hoti m s6 nguyen dUdng.

Bai t~p 2 Nguoi ta xep n nam sinh va n nil sinh thanh. mot hang, sau d6 tim each. cilt hang thiinh. hai kluic sao cho moi khuc co s6 nam sinh. bang s6 nil sinh. G9i A la s6 truong hop khong the cilt hang theo yeu cau tren, B la s6 truong hap chi c6 the cilt hang thea yeu cau tren mot aicli duy nhat.

Chung minh r1ing: B = 2A.

Bai t~p 3 M9t ciiu lac b9 leo nui c6 n thimh. vien t6 chsic 4 CU9C leo tuii va El, E2, E3, E4 la oic d9i tham gia vao cac cuoc leo nui ay. Hoi c6 baa nhieu cacti chia cac d{)i sao cho e, n E2 i= 0, E2 n E3 i= 0, E3 n E4 i= 0. .

Bai t~p 4 (IMO - 1987) G9i Pn(k) la s6 hoom vi cua tiJ,p {I, 2, ... , n} c6 dung k diim c6 dinh, Chung minh rfing

n

L k·Pn(k) = n!.

k=O

Bai t~p 5 Tinh trung b2nh c{)ng cac s6 N g6m n (n > 1) chii s6 thoa man cdc di€u ki~n:

i} N g6m cae chil s6 {I, 2,4, 5} va hi~ 2 chit s6 lien ti& luon. lOn han 1. ii} N chi a het cho 11.

Bhl t~p 6 DQ,t

F(k) = t (-:~)i, kEN

1,. i=O

Tinh

n F(k)

~ (n - k)!'

t=O

62

Phu'dng phap thi~t lap h~ t.htrc 'truy h6i trong t6 hop

Huynh Un Chau

MQt trong nhtrng phuong phap co hieu qua d~ giai bai toan t6 hop la thi~t l~p h~ tlnrc truy h6i. NQi dung co ban cua phucng phap nay nhir sau: Thay VI ta d~m tT1!C ti~p J(n) theo yeu c§,u bai toan, ta se thi~t l~p h~ thirc quan h~ gnra J(n), J(n - 1), ... d~ tu do tinh dU<;1C J(n).

Bai toan 1, (Bungari - 1995) Cho s6 nguyen n ~ 2. Hay tim s6 ctic hotui vi (aI, a2, ... , an) cua 1,2, ... , n sao cho tan toi duy nhat mot chi s6 i E {I, 2, ... , n - I} thoa ai > ai+1'

Bai giai, GQi 5n la s6 cac hoan vi thea di~u kien bai toano D~ y r~ng s6 cac hoan vi rna an = n la 5n-l. Con s6 cac hoan vi (aI, a2, ... , an) vdi ai = n (1 ~ i ~ n - 1) la C~-=-~. Do vay:

n-l

s; = 5n-1 + L C~-=-~ = 5n-1 + 2n-1 - 1.

i=l

Chu y r~ng 52 = 1, til do ta co: 5n = 2n - n - 1.

o bai nay ta thiet lap h~ thirc truy h6i xu§,t phat til 5n di d~n 5n-l. Trong mot s6 trirong hop ta lai di theo huang ngU<;1C lai. Chang han:

Bai toan 2 Gid sit Fk La t¢,p hop tat cd CaCb9 (AI, A2, .•. ,Ak) trong do Ai (i = 1,2, ... ,k) La m9t t¢,p con cua {I, 2, ... ,n} [Oac t¢,p AI, A2, ••• , Ak co th€ trimq nhau}. Hay tinh

{Truong hap n = 1998 La bdi thi APMO 1998}

Bai giai. Do co 2n t~p con cua {I, 2, ... ,n} nen co 2nk bo (AI, A2, .•• ,Ak)' Vdi moi k-bQ (AI, A2"'" Ak) cua t~p {I, 2, ... , n - I} ta co th~ them hoac kh6ng them n vao t~p Ai d@ dU<;1C k-bQ (AI, A2, .•. ,Ak) cua {I, 2, ... ,n}. Vdi chu y r~ng s6 k-bQ (AI, A2, •.. ,Ak)

cua tap {I, 2, , n - I} Ia 2(n-l)k va co 2k - 1 each them ti vao k-bQ (AI, A2, •.. , Ak)

cua t~p {I, 2, ,n - I} thl ta co: 5n = 2k5n_l + (2k _ 1).2k(n-l).

Deth§,y: 51 = 2k-1. Tild§,yb~ngquinl;Lptachungminhdu<;1c: Sn = n.2k(n-l)(2k_l).

Cling gi6ng bai toan 1, ta co bai toan sau:

63

Bai toan 3 Tim s8 t{j,p con cua t{j,p {I, 2, ... ,n} sao cho trong moi t{j,p con chua it nhfit 2 ptuin. tit La hai s8 nguyen Lien tiep.

Bai gild. CQi Sn la t?,p hQP cac t?,p con khong rang cua t?,p {I, 2, ... , n} rna trong rnai t~p con khong co hai phan tl'r nao la hai s6 nguyen lien ti~p. Chia cac phan tu cua Sn thanh hai nhom:

Nh6rn khong chua {n}: s6 cac t?,p con nhu v?,y la ISn-il

Nhom chua phan tu n: {n} hoac {aI, a2,.·., an, n} (k ~ 1). R6 rang a, =J n - 1 (i = 1,2, ... ,k) nen s6 cac t?,p con nhu v?,y la: ISn-21 + 1.

Do vay:

ISnl - ISn-il + ISn-21 + 1 Vdi chu y IS21 = 2, IS31 = 4 ta co:

_ 1 [( 1 + V5)n+2 (1 - V5)n+2]

IS 1- - - -1

n J5 2. 2

Mat khac s5 t?,p con khong rang cua t?,p {I, 2, ... , n} la 2n - 1. V?,y s5 t?,p con rna trong rnai t?,p con khong co 2 phan tl'r nao la hai s6 nguyen lien ti~p la:

Bai toan 4 C6 n qua b6ng bI, b2, ... ,bn va2n h9P hI, h2' ... ba«. Bitt rang qua b6ng b, (i = 1,2, ... , n) chi bO uao dUr;lc oic h9P hI, h2, ... , h2i. Hoi c6 bao nhieu cacli bO k (1 ~ k ~ n) qua b6ng uiio ccc h9P bitt rang moi h9P chua nhitu nhfit ttiot qua b6ng. (H ai each bO b6ng duac qoi La kluic nhau khi it nhfit m9t qua b6ng dUr;lC bO vao hai h9P kluic nhau trong hai aicli do).

Bai gtai, DM Sn,k la s5 each bo k qua b6ng vao cac hop. Cia sl'r 2 ~ k ~ n. Neu mot trong k qua bong diroc chon 1a b.; thl (k - 1) qua b6ng con lai c6 thil bo vao cac hop b[ng Sn-l,k-I each, Dong thai, bn co 2n - (k - 1) = 2n - k + 1 each chon 1 hop trong cac . hop con lai dil boo Do do s6 each bo b6ng trong truong hQP nay la: (2n - k + l),Sn-l,k-l.

Truong hop qua bn khong duoc chon, Dil y r[ng k ~ n - 1. MQi qua b6ng trong cac qua bong bl, bz, ... , bn-I d~u c6 thil bo vao cac hop bang Sn-I,k each,

Suy ra:

Sn,k = Sn-l,k + (2n - k - l)Sn-l,k-l(n ~ 3, 2 ~ k ~ n)

De th§.y:

Sn,n = (n + l)Sn-l,n-l, Sn,l = n(n + 1), Sl,l = 2 Til do bang quy nap ta clnrng rninh diroc:

(n + 1).k!(C~)2

S k - ~-_:_____.:.,~:._ n, - n - k + 1

64

...

i,1I

}",

Bai toan 5 C6 n > 1 thi sinh ng8i tren mot bai iron: Hoi c6 bao nhieu each ph at at sao cho hai thi sinh ng8i canh. nhau hum. c6 dt khac nhau, bi€t ding trong ngan hiing dt c6 dung m (m > 1) at va hiln nhieti m6i dt c6 nhitu ban.

Bai giai.

Nhij,n zet: Do thi sinh ng6i thea vong tron, nen mot each tu nhien chung ta nghi toi viec tim each "cat" vong tron thanh hang tht1.ng.

each 1. Ky hieu Pn la s6 each phat d~ hop 1~ cho n hoc sinh aI, a2, ... ,an ng6i theo vong tron (mot each phat d~ diroc coi la hop l~ n~u moi thi sinh diroc nhan chi mot d~ va hai thf sinh b§,tky ng6i g§.n nhau thl nhan diroc hai loai d~ khac nhau).

Ta viet a; = aj (i =1= j) n~u a, va aj cling loai d~ va ai =1= aj trong trircng hQP ngiroc lai ta clnrng minh:

Pn+! = (m - 2)Pn + (m - l)Pn-1 Xet mot each phat d~ hop 1~ cho n + 1 thf sinh aI, a2, ... , an+!.

N ~u al =1= an thl be an+! di ta co mot each phat d~ hop l~ cho n thi sinh (aI, a2, ... , an) , va co (m - .2) each phat d~ cho an+l.

(1)

Neu al = an thl ta be an+l va an di ta co mot each phat d~ hop 1~ cho (n - 1) thi sinh (aI, a2, ... ,an-I)., Ta CO (m - 1) each phat d~ cho (an, an+l) d@ hQP l~ voi an = al. V~y ta co duoc (1).

Mat khac de thay: P2 = m(m - 1); Pg = m(m - l)(m - 2).

Bang quy nap ta cluing minh diroc: Pn = (m - l)n + (m - 1)( _1)n.

each 2. Bai toan ttrong duong vdi viecdem s6 cac day (ai) (i = 1,2, ... ,n) thea: a; E M = {I, 2, ... ,m} Vi = 1,2, ... ,n va al =1= a2, a2 =1= ag, ... ,an =1= al.

Trong t~p hop cac day (ai) thea ai E M, Vi = 1,2, ... , n va al =1= a2, a2 =1= ag, ... , an =1= aI, goi An, En l§.n hrot la t~p hQP cac day (ai) rna al =1= an va al = an.

Do vdi moi day thuoc En, n~u be di s6 an thl ta diroc mot day thuoc An-l nen IBnl = IAnl· Mat khac, de thay IAnl + IBnl = m(m - l)n-1 (do al co m each chon, ai+l co (m - 1) each chon khac ai vdi moi i = 1,2, ... ,n - 1).

V~y

IAnl = m(m - It-l -IAn-II

Voi chu y IA2 = m(m - 1) ta duoc: IAnl = (m - It + (m - 1)(-I)n. Do chinh la dap s6 c§.n flm.

Nhij,n xet: Thirc ch§,t ca hai lai giai la nhu nhau. Nhung voi lai giai 2, "dircng nhu" ta da hlnh thanh nen mot huang giai maio D@ th§,y ro hieu hrc cua huang moi nay, ta d~n voi bai toan sau:

Bai toan 6 (IMO - 1979) Cho A vii E lii hai dinh d6i tam cua mot hinh tam comh. dtu. C6 mot con €ch biit d6.u nhiiy tit A. TQ,i bilt cu dinh niio tri: E, €ch c6 the tfJi m{)t trong hai dinh k€. st« €ch nhiiy toi E thi n6 ditng lQ,i (; d6. G9i an lii s6 dttang di piuin.

65

bi~t cua dung n lnioc nhiiy di &h nhiiy tit A den E. Chung minh ding:

Bai giiii. Ki hieu cac dinh nhu hlnh ve.

A

G

c

E

a2n-l = 0 1a hien nhien, GQi bn la s6 dirong di tir C Wi E qua n buoc nhay, Qua 2 biroc d§.u tien, ~ch co th~ v~ A hoac d~n C hoac G.

Do do:

(1)

Tu C (hoac G), sau hai buoc nhay ~ch co th~ v~ lai C hoac tdi A, n~u n > 2, do do:

(2)

Tu (1) va (2) suy ra:

a2n = 4a2n-2 - 2a2n-4.

Cling vdi a2 = 0, a4 = 2 ta de dang co di~u phai chirng minh.

Bai toan 7 (Du tuy~n IMO - 1996) Cho bring 0 vuong n x n (n > 1). Hoi c6 bao nhieu each danh dflu cac 0 vuong trong bring sao cho trong m6i hinh. vuong 2 x 2 c6 dung 20 vuong duac dtinh. dflu. (Hai each domh. dflu dU(Jc coi to. kluu: nhau neu c6 m9t 0 vuong nao d6 ma trong each. nay thZ duac danh dflu contronq aicli kia thi khong).

Bai giiii. GQi Sn 1a s6 each danh d§.u trong bang n x n. Xet t~p T gom cac 0 vuong nfun trong cot cu6i cling (tinh tir phai sang) va hang cu6i cling (tfnh tu tren xuong), ta goi An 1a cac each danh d§.u rna co hai 0 vuong k~ nhau trong T cling duoc danh d§.u ho~c cling khong diroc danh d§.u va Bn 1a cac each danh d§.u rna cac 0 vuong trong T diroc danh d§.u xen ke.

De thay moi each danh d§,u thuoc Bn se irng vdi mot each danh d§,u thuoc Bn-1, con moi each danh d§,u thuoc An se irng vdi mot each danh d§,u thuoc An-1 va mot each danh dau thuoc Bn-l. (Di~u nay suy ra khi xet bang 0 vuong (n -1) x (n - 1) co diroc tu bang n x n sau khi bo T).

66

Tu d6 ta c6:

IBnl = IBn-d, IAnl = IAn-II + IBn-II (n> 2) Ma S« = IAnl + IBnl \In > 1, nen S« = 2Sn-1 - Sn-2 \In> 3.

De th§.y r~ng: S2 = 6, S3 = 14. Tir d6 b~ng quy nap ta c6: S« = 8n - 10, \In ~ 2.

Bai toan 8 TIm cac 86 nguyen duang n thoa man: i) n e6 1000 chi: 88.

ii) Tflt co, cae chil 88 eua ti lo' leo

iii) Hieu cua 2 88 lien titp Mt kif eua n lu6n bang 2.

Bai giai. Trong t~p hop Sk cac s6 nguyen dirong n c6 k chi! 86 thoa (ii) va (iii), goi Ak, Bk, Ck, Dk, Ek l§.n hrot la t~p hop cac s6 t~n cling boi 1,3,5,7,9.

Tir moi s6 thuoc Ak n~u ta bo di chi! 86 t~n cling thl nhan diroc mot s6 thuoc Bk-I, mat khac tU moi s6 thuoc Bk-I neu ta b5 sung them s6 1 lam chi! s6 t~n cling thlnhan diroc mot s6 thuoc Ak, do d61Aki = IBk-ll. Tir moi s6 thuoc Bi; neu ta bo di chi! 86 t~n cling thl nhan diroc mot s6 thuoc Ak-I hoac Ck-I, n~u ta b5 sung them 86 3 lam Ch11S6 t~n cling thl nhan duoc mot s6thu<)c Bi; do d6:IBkl = IAk-11 + ICk-ll.

Tuong tv: ICkl = jBk-11 + IDk-ll, IDkl = ICk-11 + IEk-11 va IEkl = IDk-11 (\lk> 1) Su dung 5 diing thirc tren, bang each th~ lien tuc, ta c6:

ISkl = IAkl + IBkl + ICkl + IDkl + IEkl

= IAk-11 + 21Bk-11 + 21Ck-11 + 21Dk-11 + IEk-11 = 21Ak-21 + 31Bk-21 + 41Ck-21 + 31Dk-21 + 21Ek-21 = 31Ak-31 + 61Bk-31 + 61Ck-31 + 61Dk-31 + 31Ek-31

Suy ra: ISkl = 31Sk-2 (\lk > 3). Do IS21 = 8 nen ISlOool = 8.3499.

Tuong tv nhu tren ta c6:

Bai toan 9 TIm 88 cae b9 88 nguyen (al, a2, ... ,an) (n > 1) thoa man:

"Ii = 1,2, ,n

\Ii = 1,2, , n - 1

Bai giai. Trong t~p Sn gom cac bo n s6 nguyen (ar, a2, ... , an) thea d~ bai, goi An, Bn, Cn la t~p hop cac bo c6 an bang -1,0,1 tuong img,

Ta c6 ngay: ISnl = IAnl + IBnl + ICnl.

Mat khac, de th§.y tu moi b<) thuoc An hoac Bn ta c6 th~ b5 sung an+1 = -1 d~ dtroc mot b<) thuoc An+1 nen IAn+11 = IAnl + IBnl.

Tuong tv ta c6: ICn+11 = lenl + IBnl va IBn+11 = IAnl + IBnl + ICnl = ISnl.

67

Til do ta co:

K~t hop vdi 1821 = 3, 1831 = 7 ta tinh diroc:

(1 + J2)n+l + (1 - J2)n+1

18nl = 2

Ta co th~ phat bi~u bai toan diroi dang phirc tap han, chKng han:

Bai toan 10 Tim s6 cac da thuc Pn(X) b{j,c n chan thoa man cac di€u ki~n sau: i) Ctic h~ s6 cua Pn(X) thu9C t{j,p {O, ±1}.

ii) T6n tQ,i da thnic Q(x) cung c6 cac h~ s6 thu9C t{j,p {O, ±1} sao cho Pn(X) (x2 - l)Q(x)

Cu5i cling xin neu ra IDQt s5 ba; t~p d~ ban doc ren luyen,

Bai t~p 1 (VMO - 1997) N dUdng iroti chi a miit phdng ra lem baa nhieu phan neu bat cu c(ip hai dUdng trini nao cung cdt nhau tai hai diim pluui bi~t va kh6ng c6 ba dUdng trim. nao c6 giao diim chung.

Bai t~p 2 Cho n la s6 nguyen duotu). D(it 8 - XIYl + X2Y2 + ... + XnYn trong d6

ai, Yi (i = 1,2, , n) nhfj,n qui try 0 hoiic 1. G9i A(n), B(n) Zan luat La s6 2n-b9

(Xl, ... ,Xn, YI, ,Yn) c68 Ze va chan. ChUng minh:

A(n) (2n - 1)

B(n) = (2n+ 1)'

Bai t~p 3 Ky hi~u M = {-I, 0, I}. Tim s6 cdc b9 (al,"" an) thoa: 1. ai thu9C M vai moi i = 1,2, ... , n.

2.' ai - aHI thu9C M vai tnoi i = 1,2, ... ,n - 1.

Bai t~p 4 Cho truac s6 nguyen duatu; n. Xet tat ca aic t6ng

8 = XIYI + X2Y2 + ... + XnYn vai Xi, Yi E {O, I}, Vi = 1, n G9i 8n La s6 t6ng le, Pn La s6 t6ng chan. Chung minh:

68

Y tu'dng giai va sir tu'ong minh 10i

, . ?

giai qua mot s6 hal toan to hop

Le Van Quang

Trong cac ky thi IMO, cac bai toan t6 hQP (Combinatorics) duoc dM trtroc d§.u bai bang chfr "C". Vi du: Bai 6 IMO 2005 do Rumani d~ nghi diroc ghi: C6(ROM).

Khi doc d§.u bai cua cac BTTH (Bai toan t6 hQP)· thi h9C sinh d~u c6 th~ hi~u cac gia thi~t va k~t luan kha de dang, nhung giai diroc chung la di~u kh6 khan. Tir bang k~t qua di~m cho th§,y s6 h9C sinh giai diroc di~m t6i da r§,t it, dieu d6 chirng to day la loai bai toan kho, tham chi c6 trong tay 10i giai cua tac gia ra bai toan d6 thl khong phai h9C sinh nao cling hieu d§,y du va can ke 10i giai. Nhtrng ngiroi tu giai bai toan d6 b~ng m9t each khac thirong hi~u duoc 1ai giai cua tac gia mot each kha de dang. Tai sao lai nhir v~y?

Mot s6 BTTH thuong d~ cap mot s6 y~u t6 rang buoc theo nhtrng quy tttc nao d6.

Yeu c§.u cua bai toan la danh gia mot dai hrong nao do lien quan d~n cac y~u t6 da d~ cap, hoac chirng minh mot quy uk nao d6 luon thuc hien diroc, hoac chimg minh mot quy luat nao d6 nghiem dung.

LUQc d6 tu nhien d~ ti~p can viec giai loai bai toan nay de. diroc hlnh thanh cho hoo sinh tir cac lop dudi gom cac biroc:

1. Chon §.n d~ mo ta cac y~u t6 trong d§.u bai thanh mot phuong trlnh, mot b§,t phuong trinh hoac mot h~ han hop chua §.n de. chon,

2. Xu ly cac dieu vira mo ta theo yeu c§.u cua bai toan bang each giai ra nghiem hoac bien d6i thanh nhirng k~t qua giup cho viec hlnh thanh quy tttc hay quy luat thoa yeu . c§.u bai toano

Tir y tirong giai nhu th~ thi khau then ch6t nhat la th~ hien tucng minh ra mot 101 giai cu th~. Do bai toan kh6, nguoi giai duoc bai toan chac chan phai chi ra diroc rn,6i quan h~ noi tai cua cac y~u t6 trong bai toan thong qua cacky nang bien d6i tinh xao hoac nhimg nhan xet tinh t~, ban chat nhat tir he de. mo ta ducc.

N gay d birdc 1, viec kheo chon §.n, hoac d~t them §.n phu hoac tich hQP cac y~u t6 trong d§,u bai ... 1a sir sang tao r§,t ca biet rieng cua ngirci giai,

Hoan thanh biroc 1 de. la mot thanh cong rna khong phai h9C sinh nao cling lam t6t, nhung di~u c6t y~u 1a xU 1y thanh cong d birdc 2. Trong biroc nay thucng nay sinh mot s6 v§,n d~ 1a cac ket qua thu duoc thirong la do cac phep bien d6i he qua. Viec khao sat ngU<;IC lai 1a c§.n thiet; hoac it ra giai quyet diroc v§,n d~ t6n tai tlnh huong rna de. chi ra. Dua ra mot vi du cu th~ d~ cluing to t6n tai tlnh huong cling khong phai de dang,

69

con tao diroc mot quy trlnh hop Iy, chat che, e6 h~ thong d@ xay dung duoc tlnh huong doi hie lai kh6 hen yeu e§,u cua d§,u bai,

Cac BTTH nay di!u do cacnha toan hoc lirng danh tren th~ gioi sang tac nen trong , lei giai cua ho thuong thong bao mot kham pha moi vi! tri thirc toan, mot "b§,t bien" nao d6, hoac kien thiet mot thuat toan nao d6 ... Doc cac lai giai cua ho.choc sinh hoc t~p diroc nhimg each d~t v§,n d~ mot each sang tao, nhirng ky nang bien d6i dieu luyen bac th§,y, nhtrng hoat dong v~ tich hop cac du kien rieng Ie thanh nhtrng k~t qua sau s[c rna tu d6 c6 th@ dira ranhtrng k@t luan xac dang, cac khang dinh rna ho thuong dM ten la cac "b6 d~".

Cling y tirong giai nhung c6 th@ c6 nhieu each d@ thuc hien su tuong minh loi giai.

Trong d6 c6 lai giai rna hoc sinh cho la kh6 hieu, Viec goi y cho hoc sinh mot y ttrong giai va dong vien hoc sinh no lire thirc hien theo each cua mlnh, d@ ttrong minh ra mot lai giai cu th@ cho BTTH, phai chang la. each hop ly d@ giup cho cac em hoc sinh rndi biit d§,u lam quen voi cac BTTH h6c btia nay.

Sau day la cac bai toan minh hoa eho cac di~u vira d~ cap,

Ba BTTH diroc chon minh hoa nam trong cac ky thi: Chon hoc sinh gioi Qu6c gia 2005 cua Viet Nam, USAMO l§,n thir 30 va IMO 2005. Chung ta se xem xet cluing a hai khia canh: y tuang giai va sir tuong minh loi giai,

Bai toan 1 (HSGQG 2005, Bai 3) Trang m(it philng, cho bat quic l8i AIA2A3A4AsA6A7As ma khong co ba duang cheo nao cua no edt nhau iai mi)t diim. Ta gQi moi giao diim cua hai duang cheo cua bat giac La mot nut. Xet cdc tu giac l8i ma moi tu giac d€u co ca bBn dinh la dinh cua bat quu: da cho. Ta qoi moi tu giac tinu V9,y la tu giac can.

Hay tim. s6 nguyen n nho nhat co tinh chat: co the to mau nut sao cho vai tnoi i, k E {I, 2, 3, 4, 5, 6, 7, 8} va i =I- k, n€u ky hi~u s(i, k) La s6tu giac can nh9,n Ai, Ak lam dinh ua d8ng thai co giao diim hai duang cheo La mot nut da duac to mau thz tat ca cac gia try s(i, k) d€u Mng nhau.

Giai.

GQi n la s6 nguyen nho nh§,t thoa bai toano Ta c6 s(i, k) = s(l,2) voi moi i, k E {I, 2, 3, 4, 5, 6, 7, 8} va i i= k.

Do mot nut tuong irng vdi Cl cap dinh nen:

n.ci = L s(i, j) = C;.s(l, 2) {::} 3n = 14s(1, 2)

i<j

Suy ra n chia h@t cho 14. Tir d6: n ~ 14.

Cach to mau 14 nut thea man bai toan sau:

{1,2,3,4}{1,2,5,6}{1,2, 7,8}{2,3,5,8}{2,3, 7,6}{3,4,7,8}{3,4,5,6}{1,4,8,5}{1,4,7,6}{5,

Nh~n xet.

70

6,

- V6i mot hinh lap phuong co th~ ghi lai moi dinh mot s6 chon trong t~p {I, 2, 3, 4, 5, 6, 7, 8}, hai dinh khac nhau ghi hai s6 khac nhau.

- Moi canh hlnh lap phuong co th~ tuong trng vdi dung 3 canh song song voi no.

- Moi duong cheo ciia mat hinh l~p phuong co th~ tuong irng voi dung 3 dirong cheo

cua mat cung nam trong mat chua no hoac trong mat d6i dien voi no.

- Moi duong cheo (chinh) ciia hlnh l~p phuong co th~ tirong irng voi dung 3 duong cheo (chinh) con lai.

V6i Y tirong tren co th~ hi~u ly do tai sao lai chi ra dU<;1C each t6 mau nhu tren,

Bai toan 2 (USAMO 2001, Bai 1) Co 8 cai hop, m6i h{)p chua 6 trai banh. Tim s6 nh6 nhtU sao cho m6i banh tuy y d€u dU(jc to mot trang n mau thod man d6ng thai hai ditu ki?n sau:

1. Trang m6i hop, khong co hai banh nao duac to cung mot tnau.

2. H ai h{)p Mt ky co chung khog qua mot mau.

Gicii.

+) GQi Xi la s6 rnau xuat hien i 1§.n. i = 1,2, ... ,k, (k:::;; n). Ta co:

n = Xl + X2 + + Xk

48 = lXI + 2X2 + + kXk

(1) (2)

+) GQi y la s6 each chon hai h9P khong co chung mau nao. Do hai h9P b§,t ky co chung khong qua mot mau nen:

(3)

+) V6i i > 1 ta co:

2. 12 (i-2)(i-3)

l-~+~i = 6 .

, 2), 1()

Lay (1) trir 3.(2 roi cong v6i3. 3 ta diroc:

68 1 L (i - 2) (i - 3) 1

n - - = -Xl + X· + -'II >- O.

3 3 6 ~ 3.:1 7

i 2

Tu do:

n ~ 23.

71

+) Cach to sau cua 23 mau thea bai toan (goi ten mau la: 1, 2, ... , 23)

H<)p I 1 3 4 5 6 7
H<)p II 1 8 9 10 11 12
H<)p III 1 13 14 15 16 17
H<)p IV 2 3 8 13 18 19
H<)p V 2 4 9 14 20 21
H<)p VI 2 5 10 15 22 23
H<)p VII 6 11 16 18 20 22
H<)p VIII 7 12 17 19 21 23
Nh~n xet , +) d hinh duoi, moi duong ttrcng tnrng cho moi hop, cac giao di~m d tren dirong tirong trung cho cac banh.

+) C6 dung 8 duong, moi dtrong chua dung 6 giao di~m va c6 t§,t ca 23 giao di~m.

Hai dirong b§,t ky c6 t6i da mot di~m chung.

+) Moi each danh s6 23 giao di~m, tu 1 d~n 23, cho ta mot each to mau tren cac banh d 8 hop thea cac di~u kien bai toano

Bai toan 3 (IMO 2005, Bai 6) Trang mot ky thi hoc sinh gi6iJ ctic thi sinh phdi gidi 6 bdi toano Biet rang udi hai bai to/in Mt ky lu6n co nhi€u han i s6 thi sinh dv: thi, gidi . dU(ic cd hai bai to an nay. Nqoa: ra kh6ng co thi sinh nao gidi duac cd 6 bai toano Chung minh rang c6 it ~hfit 2 thi sinh sao cho m6i nguai trang ho gidi duac dung 5 bai toano

Gicii.

+) GQi n la s6 thf sinh tham gia ky thi va Xk la 86 thf sinh giai diroc dung k bai toan (k = 0, 1,2,3,4,5,6). Ta c6:

(4)

Ta c§.n cluing minh: X5 ~ 2

72

+) Voi i,j va i =I j, goi s(i,j) = s(j,i) la s6 thi sinh giai diroc ca bai i va bai .i.ti.i= 1,2,3,4,5,6).

Theo gia thi~t luon c6: 5s(i,j) > 2n. Do d6: 5s(i,j) ~ 2n + 1. C6t§,t ca C~ = 15 . c~p (i,j) rna i < j nen:

L 5s(i,j) ~ 15(2n + 1).

i<j

Do d6:

S = LS(i,j) ~ 3(2n + 1)

i<j

(5)

+) Ta cling c6:

S = C~X2 + C;X3 + C~X4 + C;xs = X2 + 3X3 + 6X4 + 10xs

(6)

Tu (4), (5), (6):

hay

(7)

Tu d6:

Xs ~ 1.

+) Ta clnrng minh them Xs khong th@ bang 1.

Gia S11 Xs - 1. Luc d6 tir (7) cho Xo= Xl = X2 = X3 = 0, va X4 = n - 1. Tu (6) cho:

S = 6n+4.

(8)

Truong hop nay c6 duy nh§,t mot thi sinh A lam duoc dung 5 bai, con 19-i t§,t ca d~u lam diroc dung 4 bai.

+) GQi bai duy nhat rna thf sinh A khong lam duoc la bai r va k la 86 thf sinh giai diroc bai r. Moi thf sinh trong k thf sinh nay ngoaiviec giai diroc bai r con giai dU<;1C . dung 3 bai nfia trong s6 cac bai toan con lai nen:

6

3k = L' s(r,j)

j=l,j#-r

(*)

+) D~t 2n + 1 = 5a. Ta co: s(i,j) ~ a. Neu a khong phai la s6 nguyen thl:

s(i,j) > a ¢} 5s(i,j) ~ 2n + 2.

Suy ra:

L 5s(i,j) ~ 15(2n + 2) ¢} S ~ 6n + 6.

i<j

73

Trai vdi (8).

+) Neu a la s6 nguyen thl hieu s( i, j) - ala s6 nguyen khong am. Til S = 6n + 4 vi~t Iai Ei<j(s(i,j) - a) = 1, suy ra trong 15 s6 hang s(i,j) vdi i < i, phai co 14. s6 hang co . cling gia tri la a va dung mot s6 hang co gia tri la a + 1. GQi s(p, q) = a + 1. Do do gia tri cua E;=l,j:j:r s(r, j) chi co th@ la 5a hoac 5a + 1 tuy theo r kh6ng thuoc hoac thuoc {p,q}.

K~t hop voi (*), ta co hoac 5a chi a h~t cho 3 hoac 5a + 1 chia h~t cho 3 (**).

+) VI thf sinh A giai duoc 5 bai, nen t6n tai mot bai t khac voi cac bai p, q, r rna thf sinh A giai diroc, GQi h la s6 thf sinh giai duoc bai t. Trong s6 h thf sinh nay, thl thf sinh A gidi diroc bai t va them dung 4 bai mra, va h -"1 thf sinh con lai cling giai diroc bai t va them dung 3 bai nira,

VI v~y:

6

4 + 3(h - 1) = L s(t,j) hay

j=l,j:j:t

6

L s(t,j) = 3h + 1.

j=l,Ht

Do t ~ {p, q} nen:

6

L s(t,j) = 5a.

j=l,Ht

Suy ra 5a = 3h + 1 va 5a + 1 = 3h + 2.

Dieu nay mau thujin vdi (**).

Qua loi giai bai toan tren, toi cho r~ng cac bai toan sau cling co cling y tircng giai va cac em hoc sinh co th@ thuc t~p ttrong minh loi giai.

Boo toan 4 Cho n La s8 nguyen lon. han hotic Mng 3. Treti m6i canh. va m6i duiJng cheo cua n-quic dtu AIA2 ... An nguiJi ta mu8n ghi mot s8 nguyen duanq nho han hoac Mng p, sao cho cdc ditu ki~n sou dtu dU(jc ihoa man:

i) M oi s8 nguyen tit 1 d€n p deu duac ghi.

ii) Vai m6i tam giac AiAjAk tuy y dtu c6 hai canh. dU(jc ghi hai s8 gi8ng nhau va s8 nay Lan han s8 dU(jc ghi tren canli con L(1,i.

Hay xac dimh. s8 nguyen p ldn nhdt dt c6 tht ghi cac s8 thoa man cac ditu ki~n diJ,t

ra.

Vai gia try nay cua p, hoi c6 bao nhieu oicli ghi thoa man ditu ki~n bai toano

Boo toan 5 C6 m + 2 cai hop, m6i hi)p chua m trai banh. Tim s8 n nho nhdt sao cho m6i banh duac to mot trong n mini thoa man d6ng thiJi hai ddu kien. sau:

1. Trong m6i hop, khong c6 hai banh nao dU(jc to cimq mot mini.

2. Hai hi)p Mt ki c6 chung khong qua mot mau.

BAi toan 6 Trong mot kiJ thi hoc sinh gioi, cac thi sinh ph iii giiii 6 bai toano Bitt ding

74

t I

vai hai bai totui Mt ky Luon co nhieu han ~ s6 thi sinh d7,t thi gio'i duac co' hai Mi to an nay. Nqoii; ra khong co thi nao gio'i dU(iC co' 6 bdi toano

a. G9i k La s6 thi sinh gio'i dU(ic dung 5 Mi totui. Tim gia tri nho nhtU cua k. 1

b. Chung minh tBn tg,i ba bdi to an mi: co nhUu lum. 5" s6 thi sinh d7,t thi gio'i duac.

1

C. Chung minh tBn toi b6n bdi to/in ma co nhieu hoti 15 s6 thi sinh d7,t thi gio'i dU(Jc.

Bai toan 7 Cho mot n-puic 18i co di~ tich 8. Chung minh ding t6n iai mot cg,nh AB cua da quic va m9t diim M thu9c mien da giac nay sao cho khoo'ng cadi til diim M

,. 48

den duang thang AB khong nho luni nAB'

Tai lieu tham khao

[1] Toan hQC va Tu6i tre s6 340/2005; 344/2006: Bai cua Thay Vii Dlnh Hoa (DHSP Ha NQi).

[2] Toan hoc va Tu6i tre s6 341/2005: Bai cua Th§,y Hoang Ngoc Canh (THPT chuyen Ha Tlnh).

[3] USA and International Mathematical Olympiads 2001 Titu Andreescu and Zuming Feng.

[4] http://www.mathlinks IMO Shotlist 2005. [5] IMO 2006 Solutions.

75

~

Gidi thieu mot so bai toan dai

•• •

xuat xu til' hinh hoc



~ ~

so co

Nguyen Dang Phat

Bai vi~t nay gidi thieu vci ban doc mot s6 bai toan dai s6 diJ,e this, e6 xutu xU tit hmh. h9C, rna noi dung bao g6m hai th~ loai la h~ phuong trinh. dai s6 va cue try dt;Li s6.

1 MQt 86 h~ plurong trinh dai 86 dac biet co xuat xu tu hinh hoc,

Truce h~t, hay xuat phat til mot s6 h~ phuonq trtnh. dt;Li s6 bg,e hai dan gian (c6 xuat xu tlt hlnh hoc).

Bai toan 1. Giai va bien luan h~ phuong trlnh sau:

x2 - yz y2 - zx Z2 - xy

= b = , (a =I 0, b =I 0, c =I 0);

a c

(1)

Loi giai so hroc

D~t ~ la gia tri chung cua ba ty s6 trong h~ phirong trmh (1) r6i tfnh t (rna ta thirong goi la tin ph,!)) va x, y, z. Ta diroc:

a = t(x2 - yz), b = t(y2 - zx), c = t(Z2 - xy).

Tir d6 suy ra (n~u t < 00):

a2 - be = t2.X(X3 + y3 + Z3 - 3xyz);

(2)

va hai he tlnrc nira tirong tv (d6i vdi b2 - ca va c2 - ab), thu diroc nho hoan vi vong quanh a ~ b ~ c ---+ a va x ~ y ---+ z ~ x.

Cuoi cling ta di d~n k~t qua sau day:

Tra 1Oi.

1°) N~u ba s6 a, b, c khac khong da cho va khac nhau doi mot thi x, y, z cling v~y

76

(x =I 0, y =I 0, z =I ° x, y, z doi mot khac nhau). Khi d6, t cling khac khong va h~ phuatu; trtnh. (1) la va dinh) , c6 bieu thirc nghiem nhu sau:

x

y Z

b2 = 2 b(=,Xtuyy);

-ea e-a

a2 - be

(3)

2°) Neu a = b = e thl h~ phuong trlnh (1) cling 1a va dinh, VI khi d6 x = y = z va l§.y gia tri tuy y, k§ ca 0.

Nhan xet: 1°) Bi§u tlnrc nghiem cua h~ (1) c6 tinh ch§.t dBi xung, duoc suy ra til (1) bang each thay cac hang s6 a, b, e l§.n hrot bdi cac §.n x, y, z, va nguoc lai, thay x, y, Z bdi a, b, e tucng irng, Ta n6i r[ng h~ (1) c6 tfnh ch§.t dBi hop.

2°) Til loi giai cua h~ (1) ta de dang suy ra hai h~ phirong trlnh sau (h~ ba phuong trlnh ba §,n) c6 nghiem duy nhat:

{ X2 - yz _ y2 - ZX _ z2 - xY. a - b - e '

x + y + z = a + b + e; (abc =J 0)

{ X2 - yz _ y2 - zx _ z2 _; xY.

a - b - e '

xyz = abc; (abc =I 0)

BAi toan 2. Giai he phuong trlnh (§.n 1a x, y, z):

x-a y-b z-e

, = -,- = -- = (x - a)2 + (y - b)2 + (z _ e)2.

a-a b-b c'-e

D6ng thai, chirng minh fli.ng: N~u a = b = c = Ova gitra a', b', d c6 h~ thirc

A( a'2 + b'2 + e'2) + 2(pa' + qb' + re') + B = 0,

(1')

(I")

(*)

(i)

(trong d6 A "I 0, B-1 0; a' =I a, b' =I b, e' "I e; P"l 0, q =J 0, r "I 0) .thl gnra x, y, z Cling co h~ thirc sau day:

B(x2 + y2 + z2) + 2(px + qy + rz) + A = 0,

(ii)

Huang d§.n. Dat t la gia tri chung cua ba ti s6 d v~ trai cua (*), cling c6 nghia d~t t = (x - a)2 + (y - b)2 + (z - e)2 1a tin phu. Liru y r[ng dat §.n phu nhu th~ nay cling 1a phuong phap chung d§ giai he phuong trlnh dai s6 c6 dang (*) nhu he phirong trlnh tren day. Sau khi thuc hien mot s6 phep bi@n d5i dai s6 tuong duongta thu duoc dap s6 cua bai toano Phan nay danh cho ban doc tv ki@m nghiem,

Trll loi. Bi§u thuc nghiem cua he c6 th§ dU<;1C vi~t (siipd~t) duoi dang sau day:

a' - a b' - b e' - e

-- = -- = -- = (a' - a)2 + (b' - b)2 + (e' _ e)2;

x-a y-b z-e

77

(**)

Nhan xet, Bieu thirc (**) cua he (*) co tinh chat d6i xirng cling giong nhu bi~u thtrc nghiem (3) cua he (1) diroc chi ra trong bai toan 1 0 tren. Cu th@ la (**) dU<;1G suy fa til (*) bang each giu nguyen cac hang s6 du kien (da cho) a, b, c nhung lai thay cac hang s6 a', b' ,d l§.nlu<;1t boi cac iln x, y, z va ngU<;1C lai, thay x, y, z l§.n hrot boi a', b' , d.

Xuat xu cua bai t.oan 2. Til bai toan dai s6 1 tren day, nhan nhan xet v~ tinh ch§.t II d8i lfJ,p" cua bieu thirc giira nghiem ("iln") va hang s6 dii kien, tac gia bai vi~t nay lien ttrong d~n tinh ch§.t "d6i hop" cua phep bien hlnh nghich dao trong mat phllng ciing nhu trong khong gian. Trude het, vi~t phuong trlnh vecto cua phep nghich dao trong khong gian cue a va phuong tich p = r2 = 1) r5i d~ xuat thanh bai toan 2 tren day.

----+ -----+

----+ ---+ ---+ ----+ -----+ OM ----+ OM'

OM.OM' = 1 (OM' /' /' OM) {::} OM' = OM2 {::} OM = OM'2'

hay la:

)

r (

~ ~ ~, ~

j ~ x - Xo ~ ~ x - Xo

x - x; = (~ ~)2 {::} X - x; = (~' ~)2·

X - Xo x - Xo

Sau do, vi~t phuong trlnh vecto cua mat e§,u (hay duong tron trong m?,t phllng)

~

A 72 + 2 C .:i! + B = 0 r5i d~ xu§.t bai toan chirng minh ti~p theo (Phep nghich dao

bien mat c§.u thanh mat c§,u hay m?,t phllng).

Bai toan 3. Giai va bien luan h~ phucng trinh (iln x, y, z):

(a - x) cos otan'' a + (y - b) cos 13 + (z- c) cos-j = 0, (x - a) cos a + (b - y) cos o tan? 13 + (z - c) cos-y = 0, (x - a) cos o + (y - b) cos 13 + (c - z) cos'Ytan2'Y = O.

IT (cos a =I- 0, cos 13 =I- 0, cOS'Y =I- 0; 0 < a, 13, 'Y < "2)

(***)

- 2 1 2 . 1

Huang dan. Thay tan a = -. -2--1, tan 13 = ~j3 -I,

cos a cos

phuong trlnh (***) thl dua dU<;1C h~ nay v~ dang sau day:

x-a y-b z-c (x - a) cos a + (y - b) cos 13 + (z - c) cOS'Y = -- = -13 = --;

cos a cos cos 'Y

1

tan2'Y = --2- - 1 vao h~ cos 'Y

(***')

Tra loi. 1 0) N~u a, 13 va'Y khong co m6i lien h~ gi vdi nhau, cu th@ la: cos2 a + cos? 13 + eos2'Y =I- 1 thl he phuong trlnh c6 nghi~m duy nhat: x = a, y = b, z = c.

2°) N~u cos? a + cos2 13 + cos2 'Y = I, h~ phuong trinh co vo s6 nghiem, dang:

{X = a + Xcos e, y = b + ACOSj3, z = C + A cos 'Y ,

(trong do A la mot tham s6 tuy y, A E JR)

Xu§.t xu cua bai toan 3. Bai toan tren day diroc phat hien d5ng thai voi bai toan 4 (duoc trlnh bay ngay sau day), nay sinh til viec dat bai toan 4. Bai toan nay c6 nhien

78

de hon nhieu so voi Bai toan 4.

Nhan xet: Chung ta c6 th~ de dang mo rong bai toan cho trirong hop nhieu §.n: n §.n Xl, X2, ... ,Xn, d6ng thoi cho n g6c aI, a2, ... ,an'

Bai toan 4 cling vay,

Bai toan 4. GiiH va bien luan h~ phuong trlnh (§.n la X, y, z):

p(X2 + y2 + z2) + q(x cos a + y cos {3 + z cos 1')2

--=--=--=.-

(p + q) (x cos a + y cos (3 + z cos 1')

a-x b-y c-z

(****)

cos a cos {3 cos l'

1r ,

trong d6 p =t 0, q =t 0, p + q =t 0, 0 < a, (3, l' < 2' va thoa man dieu kien sau day:

Huang d§.n gild. Chung minh riing gia tri cua bi~u thirc



p(x2 + y2 + z2) + q(x cos a + ycos{3 + ZCOS1'? (p + q) (x cos a + y cos (3 + z cos 1')

(ii)

khong phu thuoc vao x, y, Z"

Cu th~, goi -A la gia tri cua bieu thtrc (ii) 0 tren, th~ thl ta ducc: a = z +Xcos o, b = y + Acos{3, c = z + A cos 1'. Tir d6 suy ra:

p(a2 + b2 + 2) + q(acosa + bcos{3 + CCOS1')2

A=--~--~~-------~~-_

(p + q) (a cos a + b cos (3 + c cos 1')

(iii)

Trii Wi. 1 0) Neu a cos a + b cos {3 + c cos l' =I 0, h~ (****) co nghi~m duy nhat, bi~u dien diroc diroi dang:

x-a y-b z-c cos a = cos (3 = cos l'

p(a2 + b2 + c2) + q(a cos a + bcos {3 + ccos 1-)2 (p + q) ( a cos a + b cos (3 + c cos 1')

(iv)

2°) N~u a cos a + b cos [J + c cos l' = 0, he phirong trlnh (****) VD nghi~m.

3°) N~u a = b = c = 0 thl a cos a + b cos{3 + c cos l' = 0, h~ phucng trmh VD dinh.

Nhan xet. 1°) Bi~u thirc (iv) cua nghiem cua h~ phucng trlnh (****) nhan diroc tu cac h~ thirc xu§.t phat cua bai toan bang each thay trong d6 x, y, z l§.n luot bdi cac hang s6 a, b, c da cho, va ngiroc lai. Nhu vay, quan h~ R(x, y, z; a, b, c) c6 tfnh ch§.t d8i hop, nghia la:

R(x, y, z; a, b, c) = R(a, b, c; x, y, z)

2° Ngoai ra, de dang thiet lap them he thirc d6i hop nira sau day:

x2 +y2 + Z2 + (x cos a+y cos{3 +z cos 1')2 = a2 +b2+c2 + (a cos a+b cos{3 +c cos 1'?; (v)

79

(i)

! I

I

I

!

!.

.~

D~ nghi ban doc hay tv kiem tra h~ thtrc (v) nay, xem la mot bai t~p.

Xu§.t xu cua bai t oan 4. Bai toan dai s6 4 tren day diroc tac gia bai vi~t nay phat hien nhan quan tam d~n mot phep bien hlnh d6i hop trong hlnh hoc "phi Euclide" [Can bai toan 3 d tren la mot truong hop rierig cua bai toan 4 nay, khi cho p = OJ.

Chu t hich. Dac biet, n~u chon cac g6c nhon a,j1, l' thoa man dieu kien (i) voi nhirng

3 4 12

gia tri cu th~, ch11ng han cos a = 13' cos j1 = 13' cos l' = 13 thi Bai toan 4 bay gio c6

dang cu th~ sau day:

Bai t.oan 4a. Giai h~ phuong trinh 3 §.n sau:

a-x b-y c-z

169p(x2 + y2 + Z2) + q(3x + 4y + 12z)2

--=-_. =--=

169(p + q)(3x + 4y + 12z)

3

4

12

(trong d6: p oj:. 0, q oj:. 0, p + q oj:. 0).

Sau day tac gia bai vi~t nay gi6i thieu b6 sung mot s6 bai toan mra v~ giai (va bien luan) h~ phucng trinh dai 86 bac hai cling c6 xu§,t xu til hlnh hoc d~ bati doc tu giiii va lam quen han vdi th~ loai toan nay.

Bai toan 5. Gia su (x, y, z); (x', y', Zl) la hai bo ba s6 duong, moi bO d@u thoa man b§,t d11ng thtrc tam giaC(l) , d6ng thai thoa man cac d11ng tlnrc sau day:

{ X2 + y2 + Z2 = X'2 + y'2 + Zl2, x2 + X'2 = y2 + y'2 = Z2 + Z12.

(i) (ii)

Chung minh ding ta cling c6 cac h~ thirc sau day: 1°)

(iii)

(iv)

trong d6 ~(y2 + Z2 - x2) la ky hieu Hit cua tiSng (c6 tfnh ch§,t hoan vi vong quanh):

I;(y2 + Z2 - X2)X2 = (y2 + Z2 _ X2)X2 + (Z2 + x2 _ y2)y2 + (x2 + y2 _ z2)Z2.

Huang dan Gild va Nhan xet. N~u "hlnh.hoc hoa" bai toan dai s6 nay ta duoc va dua v~ giai Bai toan hlnh hQC kh6ng gian sau day.

Tuy nhien, bai toan D9-i s6 nay can c6 y nghia nhu la "lai giiii dq,i s6 cua Bai ioin hinh hoc kh8ng gian" c6 noi dung nhu sau:

Bai t oan 5'. Trong kh6ng gian xet mot tam giac d@u ABC va hai di~m D, D' (n[m ngoai mat ph11ng cua tam giac ABC) d6i xirng vdi nhau qua trong tam 0 cua tam giac d~u da cho.

ITa ky hieu gia thiet nay 1a: 3'Ii(x, y, z) va 72(x', y', z'), trong d6 Ti chi tam giac c6 dQ dai canh 1a x, y, z d6i voi 'Ii va x', y', z' d6i vdi 72.

80

Chung minh r[ng hai tam giac 7i (D A, DB, DC) va 72 (D' A, D' B, D' C) 1a tuaru; duong (cling tire la e6 dien tich. Mng nhau), trong d6 T = T(x, y, z) 1a ky hieu tam giac c6 dO dai cac canh la x, y, z; va Ie dirong nhien 3 s6 dirong x, y, z thea man b§.t d~ng thirc tam giac.

Chu t hich, Su t6n tai cac tam giac 7i(i = 1,2) diroc suy ra tir dinh 1y Pompiu rna tae6 th@ de dang chirng minh. Ngoai ra, ta cling hru y r[ng L:(y2 + z2 _ X2)X2 = -(x4 + y4 + Z4) + 2(X2y2 + y2z2 + Z2X2) -168r, trong d6 81 = s(7i) la dien tfch cua tam giac 7i (x, y, z).

Cling vay, L:(y'2 + zl2 - XI2)X'2 = 168?, trong d6 82 = 8(72) la dien tfeh cua 72 = 72(x', V', z').

Bai toan 6. 1°) Chung minh r[ng cac nghiem dircng x, y, z (n~u e6) cua he ba phucng trinh bac hai:

{X2 + y2 + Z2 = a2 + b2 + c2, (a, b, c > 0); x2 + a2 = y2 + b2 = z2 + c2;

(i) (ii)

thea man he thtrc (iii) sau day:

L:(y2 + Z2 - X2)X2 = L:(b2 + c2 _ a2)a2;

(iii)

Chu t hich: Khong e§,n phai giai he phuong trlnh

2°) Dao lai, n~u x, y, z la cac nghiem duong cua he phuong trlnh (H2) (gom 2 phircng trlnh (ii) va mot phuong trlnh (iii)):

H2 = ((ii) , (iii) r

thl cling thoa man he thirc (i).

T6m 11,Li:

'J-t1{(i) , (ii)} {:} H2{(ii), (iii)}

3°) Chirng minh r[ng cac h~ Hj (j = 1,2) c6 nghiem (dirong] khi va chi khi cac s6 duong . a, b,c da cho thoa man cac b§.t d~ng thirc sau day:

Bai toan 7*. Cho 9 s6 dirong a, b, c; x, y, z; x', V', z' thea man cac d~ng thirc sau day:

{E(b2 + c2 - a2)a2x2 = L:(b2 + c2 _ a2)a2x'2, x2 + X'2 = y2 + y'2 = z2 + Z12.

(1) (2)

Chung minh r[ng ta cling e6 d~ng thirc sau:

L:(b2y2 + C2Z2 - a2x2)a2x2 = L:(b2y'2 + C2Z'2 _ a2xI2)a2x'2.

(3)

81

Chu thich. Bai toan 7 co xu§.t xu tir bai toan hinh hoc khong gian sau day:

Bai toan 7'. Hai tU dien ABCD va ABCD' co day ABC chung va hai dinh D, D' d5i . xirng vdi nhau qua tam 0 cua duong tron ngoai ti~p day ABC. Chung minh rling cac tam giac Ti(BC.DA,CA.DB,AB.DC) va 72(BC.D'A,CA.D'B,AB.D'C) co dien tich bang nhau.

Bai toan 8. Giai va bien luan h~ phuong trlnh sau (§.n x, y, Z, t d§u > 0):

(1-l) {X2 + y2 + z2 + t2 = a2 + b2 + c2 + d2, (a > 0, b > 0, c > 0, d > 0);

x2 + a 2 = y2 + b2 = Z2 + c2 = t2 + d2;

Xu§.t xu cua bai toan 8. Bai toan 8 tren day co xu§.t xu til bai toan hmh hoc khong gian sau day:

Bai toan 8'. Cho tu dien g§_n d§u ABCD (rna tu dien d§u la trl10ng hop d~ biet), nghia la cac canh d6i dien bang nhau: BC = DA, CA = DB, AB = DC.

Ray xac dinh cac khoang each til mot diem P da cho d~n cac dlnh ella tu dien bi~t r~ngdi~m Q, d6i ximg vdi P qua tam 0 ella mat e§.u C(ABCD) ngoai ti~p tu dien da eho co khoang each d~n cac dinh A, B, C, D cua tu dien theo thir tl,t b[ng QA = a,QB = b,QC = c,QD = d.

Huang d§.n. Ap dung dinh 1:5' v§ dirong trung tuyen trong tam giac va tinh eh§.t cua trong tam ella mot he diem, Tuy nhien ciing eo th~ sir dung phucng phap vecto.

(***)

Bai totin lu6n c6 nghi~m (dudng) duy nhat, xac dinh boi cac h~ thtrc (***). Muon vay, hay chimg minh tmh ch§.t sau day, diJ,c ·trung cho mnt tit di~n glln dtu (bao gbm ca tU dien d§u) , tUdng tv voi dinh 1:5' Pompiu trong hinh hoc phang, dac trung cho mot tam giac d§u.

Djnh 1:9' Pompiu (trong hinh hoc khong gian) co noi dung nhu sau:

Blnh phirong cac khoang each til mot diem P b§.t ky trong khong gian d~n cac dinh ella mot tU dien g§_n d@u ABC D da cho luon bi~u thi dien tfch cac mat ella mot tu dien (T) nao do. (Di@u do conghia cu th~ nhu sau: Binh phuong khoang each til P d~n mot dinh nao do cua tU dien g§_n d@u ABC D da cho khong IOn hon t6ng binh phuong cac khoang each tu P den ba dinh con lai).

82

Tu dien (T) nay suy bien thanh mot tU diem phang (tuc la c6 th~ tich v(T) = 0) khi va chi khi di~m P trimg vdi di~m xuyen tam d6i cua mot dinh nao d6 cua tu dien g§,n d~u da cho ABGD tren mat c§,u C(ABCD) ngoai tiep tu dien d6.

Huang d~n chirng minh dirih ly Pompiu trong hlnh hoc khong gian.

SU dung vecto va tfnh ch§.t cua tam ti cu cua mot he diem, chimg minh cac b§.t diing thirc hlnh hoc sau:

PA2 ~ P B2 + PC2 + P D2, va ba B.D.T tuong tu PB2 ~ PG2 + PD2 + PA2,

PG2 ~ PD2 + PA2 + PB2,

PD2 ~ PA2 + PB2 + PC2.

(1) (2) (3) (4)

D§.u dKng thirc xay ra khi va chi khi: P E {A', B', G', D'}, trong d6 A' = CEo(A), B' = CEo(B), C' = CEo(C) va D' = CEo(D), 0 la tam c§,u C(ABGD), tuy theo P tirong irng thuoc VaG b§.t ding thirc (I), (2), (3) hay (4).

Chu t hich b6 sung. Trong m1).C 1.2 tren day, tac gia bai viet nay da gidi thieu them 4 beLi totiti moi cling v~ giai nhtrng h~ phuong trlnh bac hai c6 3 hoac 4 §.n, trong d6 co chi r6 xu§.t xu hinh hoc d6i VC1i ba bai toan 5, 7, va 8. Chinh viec giai cac bai toan nay, cac ban da cho liJi gidi dr;Li s6 cua cac bai toan hinh hoc 5', 7' va 8', la xu§.t xu hlnh hoc cua cac bai toan dai 86 5, 7 va 8. N goai ra, tac gia cac d~ toan nay cling d~ nghi ban doc tim toi them loi giai thuan tuy hlnh hoc cua cac bai toan hlnh hoc xu§.t xu 5', 7' va 8', g6p phan lam phong phu va da dang cho lbi giai cac bai toan d6. D6i VC1i bai toan 6, tac gia cua n6 cling xuat phat tir mot bai toan hlnh hoc rna d~ xuat, nhung lai khong gioi thieu trong bai viet. Tac gia c6 dung y d~ Iai, danh cho ban doc pluisi doom; suy xet v€ ngu8n gac xufit xu nao tu hinh hoc cua bai toan dai 86 (bai 6) nay. Thy nhien, xin hru y ban doc la vi?c ph/in. dodn v€ xufit xu hinh. h9C cua bai toan 6 thirc ra kh6ng c6 gz kh6 khan ldm. Cac ban chi c§,n quan tam d@n y nghia hunh. h9C ciia cac dKng thirc (ii) va (iii) cling nhu y nghia hlnh hoc cua hai bo 86 dircng (a, b, c) va (x, y, z). Tu d6 ban de dang chi ra d6i tirong hlnh hoc khong gian nao §.n d d~ng sau 6 con 86, trong d6 a, b, c la dB. cho con z , y, z la cac §.n 86 dirong c§,n tlm,

Sau cling, xin gioi thieu hai bai to an v€ h? phiamq trinh. dq,i s6 blj,c hai pluic tap luni va chua nhi€u tin (5 hoiic 6 tin).

BM toan 9*. Gidi va bien. lulj,n h? phuong truih. blj,c hai ( sau tin Xi, Yi, i = 1,2,3 lo' nhftng s6 duong):

(1i)

{Xl + Y2 = X2 + Y3 = X3 + YI = d,

xi + YI + kXIYI - x§ + y~ + kX2Y2 = x~ + y~ + kX3Y3 = 2;

trong d6 eva d la nhtrng 86 dirong va k E R; c, d, k cho trudc. a) Loi giai thu nh§.t: Loi giai g6m hai bUC1C sau day:

83

Buac 1,' ss« h~ phuotu; trinli 'H {( *), (**)} co nghi~m thz Xl = X2 = X3 (= X) va Yl = Y2 = Y3 (= y).

Tir (*) thay Yl = d - X3, Y2 = d - Xl va Y3 = d - X2 VaG (**), sau khi thuc hien cac phep tinh r6i rut gon ta diroc h~ ba phuong trinh sau d5i voi ba §.n Xl, X2 va X3:

(i)

va hai phirong trlnh tuong tu bang each hoan vi vong quanh cac chi s5: 1 --+ 2 --+ 3 --+ 1, trong d6 ta da d?,t X = d2 - c2 + 2:;=1 xT.

Tir d6 ta bieu thi duoc d bang ba bi~u thirc khac nhau nhu sau day, trong d6 bi~u thtrc sau diroc suy tu bi~u thirc dtrng trudc n6 nho hoan vi n6i tren

d = x~ + kX2X3 - X kX3 - 2X2

X~ + kX3Xl - X kXl - 2X3

X~ - kXlX2 - X kX2 - 2Xl

(ii)

(voi quy irdc neu mau s5 triet tieu thl tu s5 cling triet tieu).

Sau d6 ap dung tinh ch§.t cua day cac ty s5 bang nhau d~ khu dai hrong X, ta thu duoc ba dl1ng thirc sau:

trong d6

(iii)

va t2 nhan dH<;1C tir tl cling nhu t3 nhan diroc tu t2 bdi hoan vi vong quanh cac chi s5 1 --+ 2 --+ 3 --+ 1. (Le dirong nhien cling de nhan ra r~ng d~n hrot tl thl nhan lai tu t3 cling boi hoan vi vong quanh cac chi s5 nhir da n6i 0 tren). D~n day tu cac cap dl1ng tlnrc ti = tj (i =I- j E {I, 2, 3}) ta d@u thu duoc S\! triet tieu cua mot bi~u thirc c6 dang mot tam thirc bac hai d5i voi k sau day:

(iv)

Chang han, tu t3 = tl thi a, j3 va , l§.n hrot d@u c6 dang la nhirng da thirc bac 3 thuan nhat va d5i xiing d5i vdi cac d5i s5 Xl, X2, X3 theo nghia hoan vi vong quanh cac chi s5 1,2,3 nhu da n6i 0 tren. Cu th~ 1a:

{a = -X2X~ - X3X~ - XlX~ + 3XlX2X3, (v)

j3 = -2(x~ + x~ + x~ - 3XlX2X3) +" . (vi)

I = (x~ + x~ + x~ - X2X~ - X3X~ - XlX~) + 2(x~ + x~ + x~ - X3X~ - XlX~ - X2X~) (vii)

Sau vai phep bien d6i don gian mra, cac h~ s5 a, j3 va , trong phuong trlnh (iv) con diroc bi~u thi duoi dang mot s5 bieu thirc khac mra, thuan tien han cho viec xet d§.u cua chung, That vay, ta c6 cac bi~u tlnrc sau:

I-a = (X2 - Xl)(X3 - Xl)X2 + (X3 - X2)2Xl,

j3 - -(Xl + X2 + X3) [(X2 - X3)2 + (X3 - Xl)2 + (Xl - X2)2] +" ,= (X2 - Xl) [2(X~ - xi) + (x~- xD] + (X3 - x2)(2x~ - XI - x~).

(v') (vi')

(vii')

84

Ngoai ra, cling voi bi~u thirc (v') cua a va bieu thirc (vii') cua , ta con nhan duoc them hai bieu tlnrc nita cua a cling nhir hai bi~u tlnrc nita cua , nho hoan vi vong quanh 1 -+ 2 --+ 3 -+ 1 nhir da n6i d tren. Tir d6 suy ra, ngoai (vi') thlj3 cling con c6 hai bi~u thiic khac mra. D5i vdi viec xet d§.u cua a, j3 va " khong m§.t tfnh t6ng quat chung ta c6 th~ gia dinh quy irdc r~ng: 0 < Xl ::; X2 ::; X3 (hoac 0 < X2 ::; X3 ::; Xl, hoac o < X3 ::; Xl ::; X2)' Chllng han, voi gia dinh 0 < Xl ::; X2 ::; X3 ta th§.y ngay r~ng a ::; 0 va,2 0 con j32 > O. Tir d6 suy ra biet s5 cua (iv) khong am: 6. = j32 - 4a, 2 o. Ngoai ra, voi cac bi~u tlnrc (v') va (vii') cua a va " ta nhan ra ngay r~ng:

Va do d6: a = 0 {::} , = 0, keo theo j3 = O. Mat khac, de th§.y: Tli 6. = j32 + (-4a,)

suy ra:

6. _ 0 {::} [a = 0 va j3 = 0, , = 0 vaj3 = o.

Bdi v~y, ta di d~n k~t luan:

6. = 0 {::} Xl = X2 = X3 (= X) va do d6, keo theo Yi = Y2 = Y3 (= y).

Nhirng 6. = 0 lai la ai€u kien can va au d~ k la nghiem duy nhat cua h~ phucng trlnh H{(*), (**)} diroc xet (n~u"h~ nay c6 nghiem).

Tom.lai la; lap luan tren day chung"to r~g: Neu h~ H sau phtrong trinh {(*), (**)} c6 nghiem thl cac §.n Xl, X2, X3 bang nhau (va d?,t bang X), d6ng thci cac §.n YI, Y2, Y3 cling bang nhau (va d~t bang y). D6 la dpcm. Va do do, bai toan quy v@ viec giai mot he hai phircng trlnh bac hai d6i voi hai §.n X va Y sau day:

Buac 2: Gidi va bien lUQ,n he hai phuong irinh. (an X va Y atu duong)

{X+Y=d,

X2 + y2 + kxy = c2.

(1) (2)

Tli (1) va (2) ta dU<;1C

(2 - k)xy = rP - c2

(3)

Tir d6 de dang suy ra:

- Neu k = 2 va c =J d thl h~ phuong trlnh {(1), (2)} vo nghiem,

- N~u k = 2 va c = d thl h~ phirong trlnh c6 vo s5 nghiem, mien sao thoa man (1).

C1,1 th~ la

{X = ~ + A,

Y= E!. -,\'

2 '

- Neu k =I- 2 thl

rP - c2 XY=

2-k

(4)

85

va do do, x va y la hai nghiem Xi (i = 1,2) cua phuong trlnh bac hai sau:

d2 2

X2 _ dX + - c = 0 2-k

(5)

Muon cho phuong trlnh bac hai (5) co hai nghiem duong (phan biet hoac trung nhau), di~u kien c§,n va du la:

6. = d2 - 4 ~ = ~2 ~ 0 va (2 - k) (d2_ c2) > o.

Cu5i cling, tu cac b§.t diing thirc (6) de dang suy ra:

Phuong trinh bac hai (5) co hai nghiem duong khi va chi khi: 1 0) Ho~c

(6)

(d2 2)

k ~ 2 - 4 ~ < 2, C < d;

(7)

(C2 d2) .

k ~ 2 + 4 d2 > 2, c > d.

Con cac nghiem duong x, y cua h~ {(I), (2)} la hai nghiem cua (5), xac dinh boi:

(8)

(9)

b) Nhan xet, Sau khi thuc hien hai biroc giai nhir da trinh bay d tren, viec giai va bien luan h~ phucng trlnh bac hai vdi 6 lin duong Xl, X2, X3; Yl, Y2, Y3 d~n day da hom t§.t. Day la mot h~ phuong trlnh dai s5 d~c biet, g6m ba phirong trmh bac nhat (*) va ba phucng trinh bac hai (**) d6ng thoi doi hoi t§.t ca cac lin Xi, Yi (i = 1,2, 3) d~u la nhirng s5 thirc duong, Bay gio chung ta hay quan tam d~n qua trinh bien luan v~ viec giai he phuong trmh {(I), (2)}. Tnroc h~t hay d~ y d~n gia tri dac biet k = 2 lam cho he phirong trinh hoac vo nghiem, hoac co vo s5 nghiem. Neu k = 2 va c = d thl (2) duoc vi~t 11;1i la (d5i chieu vdi (1)):

X2 + y2 + 2xy = c2 = d2 hay la X2 + y2 - 2xy cos 7r = c2 (= d2) (10)

Khi do, he tlnrc (10) bi~u thi dinh ly ham s&> eosin d5i voi mot tam giac "suy bign" co. hai canh voi d(> dai x, y con canh Ion nh§.t c bang t6ng hai canh co d(> dai X va y (voi c = d thl tU (10) suy ra: X + y = c (= d). Sau mra, trong hai b§.t diing thirc (7) va (8) bi~u thi di~u kien co hai nghiem dirong (c5 nhien la duy nh§.t) cua phuong trlnh bac hai (5), ta hay d~ y dac biet d~n b§.t diing thirc (7). Do la b§.t diing tlnrc dieu kien quy dinh moi lien h~ gifra hai s5 thirc dirong c, d va s5 thtrc k d~ (5) co nghiem duong, cung tiic la d~ bai toan co loi giai. D5i vdi b§.t diing thirc kep (7) ta d~c biet chu y d~n b§.t diing thirc k < 2 (voi c < d). Khi do ta co th~ d~t k = -2 cos 'Y (voi 0 < 'Y < 7r), 'Y diroc hoan toan xac dinh: 'Y = arccos -;k va do do, (2) duoc vi~t 11;1i dirdi dang:

(2')

86

Luc nay, he thirc (2') bH~u thi dinh ly ham s6 eosin d5i VGi mot tam giac c6 mot canh voido dai c, d5i dien VGi g6c c6 dt) IOn 'Y va hai canh cua g6c 'Y do c6 dt) dai x va y. Ngoai ra, theo (1) thl t6ng dt) dai x + y cua hai canh do bang d va trong cac dai hrong x, y, c, d va 'Y thl cac dt) dai c, d va ot) IOn g6c 'Y la oa cho.

Nhan xet n6i tren cling dap irng hoan toan cho ca ba dKng thirc (**), trong d6 Xi va Yi (i = 1,2,3) con thea man ca ba oKng thircf"). N6i mot each khac, cac dKng thirc dai s6 o@ cap d~n trong h~ phucng trinh H = {( *), (**)} cua bai toan 9 diroc xet a tren ciing phan anh sir kien mang nt)i dung hlnh h9C lien quan d~n ba tam giac nao do c6 mot g6c (ba g6c tircng irng] bang nhau va bang 'Y, canh d6i dien (ba canh d6i dien tuong irng) bang nhau va bang c va caccanh con lai thl thea man ba dKng thirc (*). Vdi y nghia do, ta c6 th~ phat bi@u k~t qua phan chirng minh dU<;IC chi ra trong bUGC 1 cua loi giai bai toan 9-mt)t bai toan dai s5 v~ giiii h~ phuong trinh bac hai-sang ngon ngtr hlnh h9C nhu sau.

Bai toan 9a. Gia sit ba tam giac AiBiCi (i E {1, 2, 3}) thoa man cdc di€u ki~n:

a = c; = Os; AlBl = A2B2 - AgBg va B, Cl + C2A2 = BZC2 + CgAg = BgCg + ClAl

Chung minh rb.ng ba tam giac d6 bting nhau.

Ti~p theo, cling VGi y nghia nhu tren, chung ta cling c6 th@ phien dich sang ngon ngu hinh h9C toan bt) noi dung cua bai toan dai s6 9 v@ giai he phuong trinh. N6i mot each khac, sau day chiing ta se thiet l{jp mi)t m6 hinh hsnh. h9C cua bai totui 9, ciing tuc la hinh. h9C h6a bai toan do.

e) Hmh hoc hoa n9i dung bai toan 9, ciing c6 nghia la: Phat bi@u sang ngon ngii hlnh h9C nol dung bai toan (v~ giai va bien luan mothe phuong trlnh dai s5 bac hai).

Bai toan 9b. Cho mi)t cung trbti A'YB chua g6c 'Y; AB = c. Tim tren. cung d6 (kh6ng ke hai dau mut A va B) tilt ca nhilng bi) ba ddm Cl, C2, Cg sao cho

BCl + AC2 = BC2 + ACg BCg + ACI = d,

(*)

trong d6 d La di) dai cua mi)t doom. thang cho truce. Bien lurm.

d) Loi giai thli hai (Loi giai hinh hoc) cua bai toan 9-Loi giai bai toan 9b. Loi giai gom cac bUGC theo trinh tu sau:

Buac 1. TrUGc h~t ta chirng te r~ng: Neu hai trong ba diem Cl, C2, Cg triLng nhau thi day dang thuc (*) duoc thoa man khi va chi khi ca hai diem d6 triLng nhau.

That vay, d?,t BCi = tu, ACi = b, (i E {1, 2, 3}). Khi do day dKng thtrc (*) duoc vi~t lai la:

al + b2 = a2 + bg = ag + bl (1)

N~u chKng han 01 = O2 thi al = a2 va b; = bz. Do d6 tu (1) ta diroc: b2 = bg va ag = al. Suy ra: al = a2 . ag, d6ng thai bl = b2 = bg; nghia la Og = 01 = C2 (= 0).

Buac 2. Ta chiing minh r[ng: Di~u do cfmg c6 nghia la (theo khKng dinh OU<;IC chi ra a buoc 1):

87

se« bai iotui c6 l(ji giai ihi, truac het ba diem Cl, C2, C3 phai trung nhau. D~ chimg minh khang dinh nay, ta c§,n d@n b6 d~ (bai toan phu tro) sau.

Ba d~. ss« mot tu quic lbi ABCD c6 hai cordi d6i dien AD va BC Mng nhau nhung kh8ng song song thz AB =1= CD; cv. the La AB lati luni hay nh6 lum. CD tiLy thea hai tia . AD va EC cdt nhau hay hai tia DA va CB cdt nhau (0 diim 0).

(C6 th~ chirng minh di~u nay bang cachdvng hlnh blnh hanh BCDE (Hlnh 1) n~u OC ~ OD, (ho~c hlnh binh hanh ADCE neu OD ~ OC) sau d6 S11 dung dinh 1:5' v~ so sanh canh va g6c vao tam giac ABE thl duoc AB > BE = CD, d.p.c.m).

Bay gio ta trCi lai chirng minh di~u khiing dinh tren day bang phtrong phap phan

---

chung. That vay, gia S11 tren cung A,B da cho ta tim diroc ba di~m Cl, C2, C3 d6i mot

phan biet thoa man (*), ciing Wc 1a thoa man day diing thirc (1). Kh6ng m§,t tfnh t6ng quat, ta c6 th~ gia dinh r~ng: al < a2 < a3 va do d6 theo(l) thl dira d~n:

(2)

Sau d6, ta dung mot g6c ;;cy = I rBi l§,y cac di~m Ao, AI, A2, A3 tren Cx va Bo, Bl, B2, B3 tren Cy sao cho CAo = CBo = ~; CAi = bi va CBi = ai (i E {I, 2, 3}). Th@ thl ta dU<;1C: t6AiBiC = t6ABCi (c.g.c); do d6:

(3)

Va tir cac b§,t diing thirc (2) suy ra t6AIBl C nam gon trong ca hai tam giac A2B2C va A3B3C. Ta se cluing to r~ng:

(4)

va

(5)

Tren hlnh 2 ta diroc hai tu giac 1Bi A2B2BIA3 va A3B3B2AI tuong irng thoa man (4) va (5), trong d6 hai cap tia (A2A3, B2BI) va A3AI' B3B2) d~u c~t nhau Ci C. BCii vay, theo b6 d@ Ci tren ta dU<;1C

(6)

va

(7)

Xet hai truong hop c6 th~ xay ra:

------ ----- ----------

1°) N@u al ~ bl (Hlnh 2) thl CAIBI ~ CBIAI suy ra AIA3BI < CAlEI < 900 <

----- , ,

BIAlA3 va do d6, trong t6AIBIA3 ta c6 A3BI > AIBI nen d6i chieu vdi (6) ta diroc:

(8)

88

2°) N~u al 2: bl thl cling lap luan tuong tv (ban doc tv ve hinh tucng trng) ta duoc

---- ----.. '" .;

AIB2BI < 90° < AIBIB2 va do d6, trong l::::.AIBIB2 ta c6 AIB2 > AIBI nen dai chieu

vdi b§.t d§,ng thirc (7) ta diroc:

(9)

Cac b§.t d§,ng thirc (8) hoac (9) thu dU<;1C tu gia dinh c6 cac b§.t d§,ng thirc nghiem ng~t (2), mau thulln vdi gia thiet cua bai toan la c6 day d§,ng thirc (3). Mau thujln d6 chimg te hoac al = a2 = a3 va do d6, bl = b2 = b3;._ [nghia 1a cac di~m C, (i = 1,2,3)

trung nhau 0 mot di~m Co nao d6 tren cung tron AlB], hoac it nh§.t hai trong ba s6 a, b[ng nhau (hay hai trong ba s6 bi bang nhau, i E {I, 2, 3} ).

Bay gib ta chimg minh r[ng: Neu hai trong ba s6 ai (i E {I, 2, 3}) bang nhau thl ca . ba s6 a, d6 bang nhau, do d6 ca ba s6 b, cling b[ng nhau. That vay, ch§,ng han gia S11 a2 = a3, th~ thl tu (1) suy ra b3 = bl va bay gio thl day d§,ng thirc (1) thu v~ chi con mot, d6 la:

(10)

Bai toan quy v~ chi con hai tam giac, trong d6 di~u kien (*) duoc thay bdi (10). Sau d6 cling chtrng minh tuong tv nhu 0 phan tren, S11 dung b6 d~ da neu thl chirng minh dU<;1C r[ng: al = a2 = a3 va do d6 bl = b2 = b3, tadiroc d.p.c.m.

Buac 3. R6t cuoc bai toan quy v~ bai toan dung hinh don gian sau day:

r--.

Bai toan 9c. Tim tren cuiu; tron AlB mot diim C sao cho: BC + CA = d, trong d6

AB = c, c va d la nhftng di) dai cho truac. Bien lu(j,n:

Keo dai tia BC v~ phia C r6i dung di~m D sao cho CD = CA va do d6, BD =

r--.

BC + CD = BC + CA = d. (C6 d§,ng thirc nay la do ta gia S11 r[ng C tren cung AlB

la di~m da Urn duoc, thea mandieu kien d~t ra). Th~ thl l::::.C AD can 0 C c6 g6c 0 day

- -

CDA (= BDA = ~). Tu d6 suy ra D 1a mot trong hai giao di~m DI, D2 cua duong

r--.

tron (B, d) tam B ban kinh d va cung tron AO"B chua g6c 0" = ~ [c6 tam la trung di~m

r--.

Co cua cung AlB da cho] dung tren doan AB va n[m cung phia voi AlB.

- Tu d6 suy ra each dung diem C}__: Di~m C 1a giao di~m cua tia BD va cung A~B, trong d6 D E {Dl' D2} = (B, d) n Ao"B. De th§.y rang BC + CA = d. (Hlnh 3)

- Bi~n lui),n: Bai toan co loi giai (nghi~m h'inh) khi va chi khi:

c I

AB = c < d:::; BDo = 2BCo = ~ = c.cosec-

sm2 2

(11)

K€t lui),n: Bai toan c6 2 hoac 1 nghiem hinh khi va chi khi c < d :::; si:1 = c.cosec ~

2

(khi c = dsin ~ thi bai toan c6 nghiem duy nhat); vo nghiem n~u d :::; c hoac c < dsin~.

D~n day bai toan 9c) va do d6, Mi iotiti d1/ng hinh. 9b) dd dU(jc gidi xong (vl v~ mat

~ ~

hinh hoc, cluing ta da hoan toan xac dinh dU<;1C diem C tren cung tron AlB da cho bdi

cac phep dung hlnh hoc theo trinh tv da diroc chi ra 0 tren). Tuy nhien, lin gidi hinh. hoc cua Mi totui dQ,i s6 9 d€n day chua hoan tat.

89

D~ hoan t§,t Ioi giai hlnh hoc cua bai toan 9 ta c§,n hru y r~ng bai toan 9c) chfnh la noi dung hinh hoc cua bai toan dai s5, phat bieu trong bUGC 2 cua ldi giai bai toan

~

9. Muon vay, sau khi da xac dinh dU<;1C di~m C tren A,B bang nhung phep dung hlnh ,

--

hoc rBi, ta c§,n tfnh dQ dai cac dOC:1n BC = x va AC = y thea AB = c, d va ACB = ,.

(Vi~c lam nay co y nghia tircng dirong VGi viec xac dinh vi trf hinh hoc cua di~m C b~ng phuong phap dung hlnh), Ta chi viec giai va bien luan h~ phircng trlnh (§.n x, y > 0):

{X +y - d

x2 + y2 - 2xy cos , = c2

{x+y = d

{:} xy - /:o~fq > 0 (vl c < d)

Vay x va y la hai nghiem dirong Xl, X2 cua phuong trlnh bac hai:

X2 - dX + ~ - c2 = 0 4cos2,]_

2

(12)

Dieu kien c§,n va du d~phltdng trlnh (12) co nghiem (nghiem duong) la: Biet s5

c2 - d2 sin 2 ']_ .

/:::, = 2 2': 0 {:} c 2': d sin 12 va c < d

cos? ']_

2

{:} d > c > d sin 1

- 2

(13)

D~ Y r~ng /:::, co th~ diroc viet lai nhu sau:

/:::, = 2c2 - (1 - cos,)~ = 4c2 - (2 + k)d2

1 + cos, 2 - k

Tir do ta diroc k~t qua (sau khi da thay -2cos, = k, VGi Ikl < 2)

(14)

{X }={X X}=!(d±V4C2-(2+k)d2)

,y 1, 2 2 2 _ k

(15)

K~t qua (15) nay tim dU<;1C phii hop VGi k~t qua (9) tim ra d bUGC 2, ti~u muc a).

e) Nhan xet va Wi bmh (v~ hai loi giai cua bai toan 9).

Hai bai toan hlnh hoc 9a) va 9c) chinh la noi dung hinh hoc tirong ling VGi hai bai toan dai s5 rna Idi giai diroc d~ cap den trong hai bUGC giai (buoc 1 va buoc 2) lien ti~p cua bai toan 9 v~ giai va bien luan mot h~ phuong trinh dai s5 bac hai. Chung la hai phan c§.u thanh bai toan hlnh hoc 9b - ban phien dich sang ng6n ngir hinh hoc cua bai toan 9 (mot bai toan hoan toan dai s5 v~ h~ phucng trlnh) rna chung ta co th~ cho no mot ten goi la II M6 hinh. hinh. hoc" cua bai toan 9, mot bai toan v~ giai va bien luan mot he phirong trlnh bac hai. No cho ta mot hlnh anh true quan trong hinh hoc v~t ly cua mot h~ phuong trlnh dai s5 bac hai. Tuy nhien, moi loi giai cua bai toan 9 mac du khac nhau v~ phuong phap ti~p can nhung d~uth~ hien s[c thai d~c thu, rieng biet va r§,t §,n tucng cua tung ldi giai, N~u nhu ldi giai 1 (Zai giiii dr;r,i 86) co doi hoi kfj nang bien d6i va tinh toom tinh te va c§,n phai thuc hien nhieu phep toan nhung lai cho duoc chitng minh true tiep dieu khitng dinh quan trong phat bi~u trong biroc 1 cua loi giai

90

bai toan 9 [va di~u do cling c6 nghia la chl ra diroc each chung minh true titp tinh chat hl,nh h9C phat bi~u trong noi dung cua bai toan hlnh hoc 9a)] thl Ibi giai 2 (liJi gidi hinh. h9C) cua bai toan 9 lai khang doi hoi tinh to an nhirng doi hoi thong minh sang tao, dung them duong phu, hlnh phu va chi cAn huy dong von kien thirc it oi thuoc chuang trlnh hlnh hoc 8 cling du dung cho viec clnrng minh tinh ch§.t hlnh hoc n6i tren tuy Ibi giai nay lai khong th~ cho duoc each chirng minh true ti~p rna S11 dung phuong phap phdn chung_mot phuong phap chung minh gian tiep_d~ khfulg dinh tinh ch§.t hlnh hoc nay.

Tren day la phac hoa doi net v~ t6ng the khi d6i chieu, so sanh hai ldi giai cua bai toan 9 v~ phuaru; pluip titp cfj,n cling nhu phuatu; ptuip gidi quyet van de cua tirng loi giai d6 rna bai vi~t nay neu ra nh~m muc dich trao d6i vdi ban doc nhirng suy ngam, nhung y tuong d~ xuat va kinh nghiem xung quanh viec giai va khai thac mot bai toan toan hoc hay (v~ dai s6 cling nhu hlnh hoc) nao d6.

Neu xern xet ky luang, chi ti~t han thl truce het, phai n6i r~ng: Bai toan hlnh hoc 9b) chi la mo hinh. hinh. h9C cua m(jt phan bai toan dai s6 9 rna thoi, That vay, trd lai phAn bien luan cua Ibi ghti thir nhat cua bai toan 9 (trlnh bay d biroc 2) chung ta nhan ra ngay r~ng Ibi giai thtr hai (loi giai hlnh hoc) cua bai toan nay chi d~ cap d~n nhirng gia tri cua tham s6 k thoa man b§.t dl1ng thirc Ikl < 2 sao cho co th~ gan cho k 11 nghia hZnh h9C k = - 2cos'Y, trong d6 0 < 'Y < 7r la do Ian cua mot g6c cua mot tam giac nao do rna canh d6i dien vdi g6c d6 c6 do dai c va hai canh cua g6c 'Y d6 c6 do dai x va y rna t6ng cua hai canh d6 x + y = d, d6ng thoi d > c. Nhirng gia tri cua k d day doi hoi Ikl < 2 chi la mot t~p con nhimg gia tri k thea man b§.t dl1ng thirc (7) chu chua n6i Wi nhimg gia tri cua k thoa man b§.t dl1ng thirc (8) diroc chi ra dAy du trong phAn bien luan thuoc buoc 2 cua ti~u muc a) trlnh bay Ibi giai thir nhat cua bai toan 9. Tac gia bai vi~t nay con d~ trong, chua th§.y diroc y nghia hinh hoc cua nhtrng gia tri k khac, ngoai Ikl < 2.

f) Xu§.t xu cua cac bai toan 9, 9a), 9b) va mot htrdng d~ xuat bai toan mdi khac.

Trong ky thi Olympic Toan quoc t~ IAn thir 46 (46th IMO, 7/2005) t6 chirc d Mexico

c6 hai bai toan hlnh hoc phang, trong d6 c6 bai toan (Bai -1) sau day: .

Tren cac canh. cua mot tam giac deu ABC sau diem dO. dUr;Jc chon lan luat: AI, A2 iren. BC; BI, B2 tren CA; C1, C2 iren. AB. Otic diim nay la cae dinh cua m(jt luc giae 16i A1A2B1B2C1 C2 c6 tat cd cac comh. Mng nhau. Chung minh ding cae duiJng thilng A1B2, B1C2 va C1A2 dang quy.

Chinh tu bai toan hlnh hoc nay cua ky thi IMO, 7/2005 vira qua rna tac gia da d~ xuat bai toan hlnh hoc 9a), 9b) va bai toan dai s6 9 d tren. D~ diroc thuan tien cho viec phat bi~u cac bai toan nay diroi dang nhu da trinh bay trong bai viet, tac gia da d?,t lai ten nhu hinh 4 da ghi: Tam giac d~u xuat phat diroc ky hieu lai la C1 C2C3, con luc giac 16i diroc xet ky hieu la A1B1A2B2A3B3 vdi cac cap dlnh B2, A3; B3, Al va B1, A21An hrot tren C2C3, C3C1 va C1 C2. Ba duong thl1ng duoc xet se la: A1B2, A2B3 va A3B1. Th~ thl, thea gia thiet cua bai toan va voi ky hieu nhu tren hinh 4 chung ta thiet lap diroc ngay day dl1ng thirc (*) neu trong gia thiet cua cac bai toan 9, 9b) va 9a). Tuy nhien, rieng

91

'.

v~ g6c, g6c 60° Ci cac dinh C, (cua 6. d~u C~ C2C3 va cling la cua ba 6.AiBiCi) da dU<;1C thay boi g6cc6 dO Ion I t6ng quat han, mien la 0 < I < tt .

Bai toan 10**.(1) Giiii va bien lUQ,n h? phuotu; irinh. bQ,c hai 5 tin x, y, z, u, v sou day

xz _ y2 a

xv - yu yu - Z2

-

c d

xu-yZ b

zu-yv e

(*)

I

trong d6 a, b, c, d, e, I la aic 86 ihuc khac kh6ng dii cho.

A. Loi giai "so e§.p" cua bai toano

1°) Nghi?m tam thuCJng cua bai toano

a) H~ phuong trlnh ?to (*) c6 th@ vi~t lai dirdi d~ng sau day: N~u dfJ,t A Ia gia tri chung (rna ta thirong goi Ia tin phu cue, h~) cua day 5 ti s6 bang nhau (*) thl ta c6 6 dKng thirc sau:

(1)

Aa = xz _ y2 Ab = xu - yz AC = xv - yu Ad = yu - Z2 Ae = zu - yv AI = zv - u2

b) Nhan th§.y r~ng voi x, y, Z, u, v tiiy y, bao gio ta cling c6 d6ng nh§.t thirc sau day (de dang ki@m nghiern):

(xz - y2)(zv - u2) +. (~u - yz)(zu - yv) + (xv - yu)(yu - z2) = 0, (Vx, y, z, u, v); (*') D6i chieu (I) va (*') ta dU<;1C he thtrc sau day (n~u he (*) da cho c6 nghiem):

A2(al + be + cd) = 0

Tir d6 suy ra:

1 ° Hoac A = 0, va do d6, h~ (*) c6 dang don gian (I') sau:

{xz - y2 = xu - yz = xv - yu = yu - Z2 = ZU - yv = zv '- u2 = O}; (I')

2° Hoac A i= 0 thl h~ (*) cling con nghiem khac mra khi cac hang s6 da cho rang" buoc voi nhau boi dieu kien sau:

a] + be + cd = 0,

(**)

V~y ta c§.n xet hai trirong hop:

Truong hop 1. N~u cac s6 thirc khac khong a, b, c, d, e, I da cho 18. tuy y, kh6ng doi hoi phai thoa man dieu kien (**) thl h~ phuong trinh (*) dU<;1C xet chi c6 nghi?m tam

1 Bai toan 10** nay da dU<;1C dira VaG cuon sach ciia GS. Nguyen Van Mau "Da thirc dai 86 va phan thirc hiru ty", NXB Giao due, 2002 (Bai toan 5, trang 48)

92

thuang, irng voi gia tri A = ° trong (I). Nghiem d6 la nghiem cua he phuong trlnh (1') nhu da chi ra 0 tren,

Tir h~ (1') nay ta de dang thiet lap dU<;:IC day dl1ng tlnrc sau:

(1') =? ~ = ~ = :_ = '!!. ¢:> ~ = :_ = '!!. = ~ (= t E lR) (1)

y z u v x y z u

Tir (1) suy ra h~ (I') nay c6 nghiem sau day:

X : y : z : u: v = 1 : t : t2 : t3 : t4, (voi t tuy y, t E lR);

(2)

Neu d~ Y rling 1 = to thl x, y, z, u, v thea thti tv ty 1~ voi luy thira cua t, tir 0, 1,2,3 d@n 4. Di~u d6 goi Y rling neu ta thay d6i ky hieu cac ~n s6 x, y, z, u, v l§.n hrct boi Xo, Xl, X2, X3, X4 thl bi~u thirc nghiem cua h~ (I) dU<;:IC vi@t gon lai dudi dang:

Xi = pti (p # ° b§.t ky, i E {O, 1,2, 3,4}) hay d~c biet, c6 th~ cho p = 1 thl:

Xi = ti, (iE{0,1,2,3,4});

(3)

TrU'C1ng hop 2. H~ H; diroc b6 sung them di~u kien (**) d~ tro thanh h~ (1-l) = {( *), (**)}. Tu day khong nhiing chi thay d6i ky hieu d6i voi cac ~n s6 nhu tren rna cimg thay d6i ky hieu d6i yoi cac hang s6 da cho; cu th~ la a, b, c, d, e, f l§.n hrot diroc thay boi bo, b1, b2, b3, b4, b5. Viec thay d6i nay ciing con c6 tac dung giup chung ta theo 'a6i viec giai he phucng trlnh phan nao dU<;:IC de dang han. Voi quy tree d6, ta vi@t lai h~ phuong trlnh (1-l) = {(*), (**)} = (1-lo) U (**) sau khi diroc ba sung them di~u kien (**):

(1-l) {xox:o-X? = XOX3b:XIX2 = xox4b;;xIX3 = XIX:3-x~ = X2X3~XIX4 = X2X:5-x5 (= A # 0) (*)

bob5 + blb4 + b2b3 = ° (**)

Con d6ng nhat thirc (*') 0 tren thl bay gio duoc vi@t lai nhu sau:

2 2 ) ( ( , 2) (*')

(XOX2 - X1)(X2X4 - x3) + (XOX3 - XIX2 X2X3 - XIX4) + (XOX4 - XIX3) XIX3 - x2 ;

.2°) Tim nghi?m kh6ng tam thuang cua bai toano

c) Ngoai d6ng nh§.t thtrc (*') lien quan d@n cac ~n s6 Xi (i E {O, 1,2,3, 4}) da diroc phat hien 0 tren, bang nhiing quan sat d~c biet cac nhi thirc thuan nh§.t cua cac ~n co cling bac (d6ng bac) va c6 cling tang cac chi s6, ta con phat hien them 5 d6ng nh§.t thirc mra sau day (rna ban doc ciing de dang kiem nghiem diroc):

(X2X4 - X~)X2 + (X2X3 - XIX4)X3 + (XIX3 - X~)X4 = 0, (i)

-2(X2X4 - X~)Xl - (X2X3 - XIX4)X2 + (XOX4 - 2XIX3 + X~)X3

-(XOX3 - XIX2)X4 =,0, (ii)

(X2X4 - x~)xo - (X2X3 - XIX4)XI - 2(XOX4 - XIX3)X2

+(XOX3 - XIX2)X3 + (XOX2 - Xi}x4 = 0, (iii)

(X2X3 - XIX4)XO + (XOX4 - 2XIX3 + X~)XI + (XOX3 - XIX2)X2 - 2(XOX2 - Xi)X3 = 0, (iv)

(XIX3 - x~)xo - (XOX3 - XIX2)Xl + (XOX2 - Xi)X2 _ 0. (v)

93

Bay gio ta thay cac nhi thirc a trong ngoac bdi nhi thirc tuong ling c6 gia tri Abi (i = 0,1,2,3,4, 5}) rut tit day ti s6 bang nhau (*) a tren, sau d6 khir §,n phu ,\ ta thu diroc mot he moi HI gom 5 phuong trlnh tuyen tfnh thuan nh§.t vdi 5 §,n s6 Xo, Xl, X2, X3, X4 rna dinh thirc /:::., cua he triet tieu (vI/:::" dU<;1C phan tich thanh tich hai thira s6, .trong do mot thira s6 la bob5 + blb4 + b2b3 (= 0); thira s6 con lai, ky hieu <I>(bi) =I o.

b5X2 + b4X3 + b3X4 - 0 (ih
-2b5Xl b4X2 + (b2 - b3)X3 bIX4 - 0 (ii) I
(HI) b5Xo -b4xI 2b2X2 + bIX3 + bOX4 - 0 (iiih
b4Xo + (b2 - b3)XI + blX2 -2boX3 0 (iV)1
b3Xo -bixi + bOX2 - 0 (V)l VI dinh thirc /:::., cua h~ (HI) c6 dang: /:::., = (bob5 + blb4 + b2b3).<I>(bi) = 0 rna <I>(bi) =/:. 0 nen (HI) c6 nghiem khong tam thudng duy nhat, (xac dinh sai khac mot thira s6 =/:. 0). Giai h~ (HI) ta thu dU<;1C nghiern sau day:

AIXO = b~ - bo(b2 + b3), AIXI = bob4 + blb3, AIX2 = b~ - bob5,

AIX3 = -(b1b5 + b3b4) , AIX4 = b~ - b5(b2 +b3).

(4)

[D~ giai he phuong trlnh tuyen tfnh thuan nh§.t HI nay (c6 dinh thtrc /:::., = 0) ta chi viec be di mot trong 5 phirong trlnh cua he d6, chitng han be phuong trlnh thu ba (iii)l sau d6 chi viec giai he {HI \(iii)J} gom 4 phirong trlnh con lai vdi 5 §,n Xo, Xl, X2, X3, X4 thi thu dU<;1C nghiem duy nh§.t c6 bieu thirc (4)].

T6m lai la, sau hai buoc giai ta thu diroc k~t qua:

H~ phirong trinh bac hai (H) {(*), (**)} voi 5 §,n XO,XI,X2,X3,X4 c6 nghiem t§.m thuong (3) va nghiem khong t§.m thuong (4) nhu da chi ra a tren.

B. Co sd ly thuy~t toan hoc d§.n d~n h~ phtrdng trinh tuy~n tinh thu~n nh~t (HI) d~ tim ngh'iem khong tfun thtrong cua h~ (H) neu trong bai toan 3.

Chung ta c§.n tim hi~u ngon nganh d~ ly giiii con duong dan den 10i giai "sa cap" ella bai toan lO** nhir da neu ra trong muc A a tren.

1°) Trude het, di y rdng cac hieu sau day:

la cac nhi thircbac hai thuan nh§.t cua cac §,n Xi (i E {O, 1,2,3, 4}), xuat hien trong day (*) cac ty s6 bang nhau cua h~ (Ho), diroc tao thanh tit cac dinh tlnrc con (minor) c§.p 2 rut til rna tran chit nhat (2 x 4) sau:

II~~

94

Co th~ xem chung la cac tor), d¢ Plucker cua mot duong thang X<;L anh trong khong gian X<;L anh 3 chieu P3 xac dinh bdi hai diem co cac toa dO: di§m A(xo, Xl, X2, X3) va di§m B(XI' X2, X3, X4) trong toa dO X<;L anh thuan nh§.t.

2°) Cdc y tuang co ban trong qua trinh. tim toi liJi giai (nghi~m) cua h~ phuatu; truth. (H) i'J tren.

a) Nhu 0 phan A da chi ra: H~ plnrong trlnh Ho (*) va do do, h~ phirong trinh (H) co nghiem t§.m thirong rna bi§u thiro nghiem co dang (3)

Xi=pti, iE{0,1,2,3,4}, p=l0.

b) H~ phuong trlnh Ho (*) voi cac §n Xi dUQC vi~t lai nhu sau:

(I') {p. bo = XOX2 - xi, pbj = XOX3 - XjX2, pb,_ = XoX, - XjX3, pb3 = XIX3 - X~, pb4 = X2X3 - XIX4, pb5 = X2X4 - x~;

trong do p 1a gia tri chung cua 5 ti s6 bang nhau cua he phucng trlnh (*) rna gia tri p = ° cho ta nghi~m tam thuiJng (3) cua h~ (H) va chua doi hoi d~n di~u kien (**) cua cac h~ng s6 cho truce b, (i E {O, 1, 2, 3,4, 5}).

Cling nhir nhan xet b) phan A da chi ra: Voi Xo, Xl, X2, X3, X4 tuy y bao gio ta cling co d6ng nh§.t tlnrc (*'):

. . 2 2

VXi (z = 0,1,2, ... ,4) : (XOX2 - XI)(X2X4 - x3) + (XOX3 ~ XIX2) (X2X3 - XIX4)+

+ (XOX4 - XIX3)(XIX3 - x~) = 0; (*') Chinh d6ng nh§.t thirc (*') nay cimg voi dl1ng thrrc dieu kien (**) v~ cac hang s6 b, (i = 0,1, ... ,5) 1a co so quyet dinh cho viec tlm nghiem khong t§.m thuong cua h~ (H).

c) Ta chirng minh di~u khl1ng dinh sau:

Neu co h~ thirc (**) nghia la cac h~ng s6 b, (i - 0, 1, ... ,5) rang buoc boi dieu kien

bob5 + bIb4 + b2b3 = °

thl, ngoai nghiem t§.m thtrong (3) [Xi = ti (i E {O, 1, 2, 3, 4})], he phuong trlnh bac hai thu§.n nh§.t (H) = {(*), (**)} con c6 mot nghiem khong t§.m thuong duy nh§.t rna (4) 1a bi~u thirc nghiem da chi ra 0 phan A.

That vay, he thirc (**) diroc viet lai dirdi dang tirong dirong sau:

2p(bob5 + bib« + b2b3) = ° (voi p =I 0),

hay 1a

bo(pb5) + b5(pbo) + bi (pb4) + b4(pbl) + b2(pb3) + b3(pb2) = 0. (*")

Thay cac gia tri cua pbi (i = 0,1, ... ,5) tu (1') VaG (*") ta diroc h~ thirc sau day (sau khi dii nhan hai v~ cua (*") vdi 2):

2[bo(X2X4 - x~) + b5(xOX2 - xD + bl(X2X3 - XIX4) + b4(xOX3 - XIX2)+

+ b2(XIX3 - x~) + b3(xOX4 - XIX3)] = 0. (I")

95

Vdi cac bi (i = 0, I, ... ,5) la da cho thi (r") chinh la phirong trlnh trong toa do xa anh thuan nhat cua mot sieu mat bac hai, suy bienthanh mot sieu non trong khong gian xa anh 4 chieu P4.

Bay gio ta viet lai phucng trlnh (I") duoi dang rna tran xTQx = 0:

4

L qijXiXj = 0, (qij = qji)

i,j=O

(5)

trong do cac h~ s6 qij co cac gia tri cu th§ nhu sau:

qoo = 0, qOI = 0, q02 = bs, q03 = b«, q04 = b3;
qlQ = 0, ql1 = -2bs, q12 = -b4, ql3 = b2 - b3, ql4 - -bl;
q20 = bs, q21 = =b«, q22 = -2b2, q23 = bi, q24 = bo; (5')
q30 - b4, q31 = bz - b3, q32 = bl, q33 = -2bo, q34 = 0;
q40 = b3, q41 = =bi, q42 = bo, q43 = 0, q44 = 0. V~y dirdi dang rna tran thi phirong trlnh sieu quadric (5) co.dang:

° ° bs b4 b3 Xo
° -2bs -b4 b2 - b3 -bl Xl
(XO Xl X2 X3 X4) bs -b4 -2b2 bl bo X2 = 0, (6)
b4 b2 - b3 bl -2bo ° X3
b3 -bl bo ° ° X4 hay vi~t (6) mot each ngan gon han, dirdi dang toan hi:

T _ --+

X Qx = 0, \Ix f. ° = (0,0,0,0,0)

(7)

trong do: xT = (Xo, Xl, X2, X3, X4) 1a vecto hang (chuyen vi cua vecto cot x(xo, ... , X4), va Q = Q( b) la mot rna tran vuong c§.p 5, co cac phan tu qij tao thanh tu cac hhg s6 b, da cho [thea man (**)) theo bang (5') d tren),

Sau khi thuc hien phep tfnh dinh thirc IQI cua rna tran Q(b) trong do

b = (bo,bl,b2,b3,b4,bs) thl diroc: IQ(b)1 = (bobs +b1b4 +b2b3).q,(bi) = ° (vl theo (**) thl bobs + b1b4 + b2b3 = 0), nghia 1a Q(b) la mot rna tr~ suy bien va co hang r = 4. Bci vay, (6) 1a phuong trlnh cua mot sieu mat b~c hai suy bien thanh mot sieu non bQ,c hai co O-phdng_ dinh, tuc phllng_dinh nay la mot diem, diroc xac dinh boi h~ phuong trlnh tuyen tinh thuan nh§.t sau: Diroi dang toan tU, he phuong trlnh nay duoc vi~t gon lai la:

Qx=O.

(8)

VI rna tran vuong c§.p 5 Q co hang r bang 4 nen trong 5 phuong trlnh tuyen tfnh thuan nh§.t cua he (8) chi co 4 phuong trlnh 1a doc 1~p tuyen tfnh. Bdi vay, he phuong trlnh tuyen tfnh thuan nhat (8) xac dinh cho ta mot diim duy nh(U trong khong gian xa anh bon chieu P4. Di@m nay chinh 1a dinh. cua sieu n6n bQ,c hai co phirong trlnh (6) hoac (7) trong khong gian xa anh P4 nay.

96

That vay, VI (*') la mot d6ng nh§,t thtrc, k~t hop vdi diing thirc dieu kien (**) thi (*") va do d6, (6) hay (7) cling la mot d6ng nh§,t tlnrc, Va tlt (7) suy ra (8) la mot phucng trinh toan tu.

N~u vi~t cu th~ ra thl (8) cho ta M 5 phucng trinh tuyen tinh thuan nh§,t (71.1) nhu da viet ra a ti~u muc 2°)c) cua phan A a tren,

Sau cling giai h~ phuong trinh tuyen tfnh (8) cling tuc la giai h~ (71.1) ta dU<;1C nghi~m kh6ng tam thuang (4) cua he (H) nhir da chi ra trong phan A, ti~u muc 2°)c) a tren. Di~u khang dinh tren day da diroc clnrng minh.

d) V6i loi giai tren day cua bai toan dai s6 10 trlnh bay a ti~u muc c) ta th§,y r~ng viec giai bai toan nay c6 lien quan d~n mot ly thuytt hinh. h9C v€ m(it bij,c hai trong kh6ng gian X(L anh, trong d6 c6 m(it bnc hai suy bien thanh. non bij,c hai. V~ mot Y nghia nao do rna n6i thl cling c6 th~ goi loi giai nay la lai giai hinh. h9C cua bdi toom 1 0**. Dieu d6 con n6i len ding bai toan 10** chilc hiin con nhirng loi giai khac nita. Hy vong bai toan nay con nhan diroc loi giiii hay khac a ban dQC.

c. Nguon g6e lich s11 hay xuat xu cua bai toan 10**.

Bai toan 10** xu§,t hien each day da duoc 42 nam, Nam 1965 trong mot bai bao "Sur un espace riemannien a absolus locaux" dang a Tap chi Acta Scient. Viet. (Sectio Math et Phys) Tome II, p. 5-42, nhan mot v§,n d~ nghien ciru dilu tien v~ mot khong gian e6 tuyet d6i dong, giao su Nguyen Canh Toan da d@ xuat va giai bai toan dai s6 rna bai viet nay da dua vao danh muc cac bai toan ciln khao CUu v~ "M9t s6 h~ phuong truili d(Li s6 d(ic bi~t", bai toan d6 dich thirc la bai toan 10. Xuat phat tlt mot v§,n d~ nghien ciru v@ hlnh hoc, tuy c6ng cu su dung lai la dai s6, tac gia bai bao v6n giau tri tuong tuong khong gian da giai bai toan b~ng phuong phap "cdt", "chitu" , ... Va tac gia bai bao da cho dap s6 nhir chiing ta da biet v~ hai c6ng thirc nghiern (3) va (4) tren day; tuy nhien ngiroi rna duong "kham pha" da cho loi giai kha dai (do tinh toan nhieu va c6 phan phirc tap). Nam 1970, 5 nam sau d6 tac gia bai vi~t nay da cho loi giai nhir da trlnh bay d muc B. KhoangSf) nam sau nita, giao sir Nguyen Van Mau da quan tam d~n bai toan 10 nay va da giai n6 bang phtrong phap "hinh. h9C hoo" (vl c6 su dung khai niem va tfnh ch§,t ciia tich vo huong). Tac gia bai vi~t nay hy vong giao sir Nguyen Van M~u khoi phuc lai loi giai d6 d~ b5 sung mot loi giai dep va hay nita cho bai toan 10** nay.

D. B6 sung mot bai toan t irong tV: Bai toan diio (nguoc) cua bai toan 10**. Bai toan 10'. Giai h~ 6 phaumq irinli bij,c hai thuan nhat (tin yo, Yl, Y2, Y3, Y4, Y5) sonu '

YOY5 + YIY4 + Y2Y3 = 0, p'ao = Yt - YO(Y2 + Y3), pial = YOY4 + YIY3,

I 2

P a2 = Y3 - YOY5,

p'a3 = -(YIY5 + Y3Y4) , p'a4 = yl- Y5(Y2 + Y3);

trong do a; la nhilng s6 thuc khac kh6ng da cho (i = 0,1, ... ,4).

97

"---·~lIriIIIII

JO) Chung minh ding h? 'ptvuanq tr'inh ('H2) nay c6 mot nghi?m duy nh6.t, trung VGi
nghi?m ciia h? pliuaru; trinh. tuyen iinh. thuan nhat ('H~) sau day:
0 0 0 Yo 0
a4 a3 a2 0
0 -2al YI
-a4 a3 -a3 -a2 0
Ay= -2a2 0 Y2 ('H~)
a4 a3 -al ao - 0
-2a3 a2 al -al ao 0 Y3
0 0 0 Y4 0
a2 -al ao 0
Ys ~) Giai h? 6 phuanq trinh. tuyen tinh. thuan nhat ('H~) nay ta thu dur;c biiu thtic nghi?m cua h? ('H2) nhu sau: (Tit h? ('H~) suy ra:

2 PYo = aOa2 - aI'

PYI = aOa3 - ala2, fY!j2 = aOa4 - ala3, fY!j3 = ala3 - a~, fY!j4 = a2a3 - ala4, fY!js = a2a4 - a~.

2 MQt 86 bai toan eire tr~ dai 86, giai tfch va hrong giac co xuat xu tu hinh hoc

Trude h~t, xin gidi thieu mot bai toan cuctri giai tich co xu§.t xu tu hlnh hoc, Bai t oan 11. Xet ham ba bien 1 = 1(XI, X2, X3) cho biJi biiu ihnic:

trong do cac bien Xl, X2, X3 rang bu(Jc vai nhau biJi dieu kien [cac diing thnic} sou:

Xl - Cl X2 - C2 X3 - C3

--- = ~ , Vi E IR

UI U2 U3

(1)

con a., b., Gi, u; (i = 1,2,3) la nhftng s6 th7,tc cho truac sao cho coc b(J ba s6 siip thu t7,t (aI, a2, a3), (b1, b2, b3), (CI' C2, C3) doi mot phiin. biet (ciing co nghza La trong ba b(j do khong co hai b(J nao trung nhau) va u~ + u~ + u~ =I- O.

Hay tim cue tiiu l-; cua ham 1. Bier: lutir;

a) Nhan xet , Day la mot bai toan cue tri giiii tich (co rang buoc), cu th~ la tim eire ti~u cua ham (*) ba bien thtrc Xl, X2, X3 rang buoc voi nhau bdi hai d§,ng thirc (1). [Tuy nhien, Cling co th~ phat bi~u khac di d~ diroc xem 1a mot bai toan v@ giiii m(jt h? phuong irinh. dfJ,i s6 (bac nhat, ba lin Xl, X2, X3) nhung giin vdi mot dieu kien khac mra, cu th~ 1a bi~u thirc (*) phai dat gia tri nho nh§.t 1m (thay eho viec bat di mot phuong trinh].

98

You might also like